You are on page 1of 129

UPSC ANALYSIS DOCUMENT FOR

PRELIMS
POLITY ANALYSIS DOCUMENT (2011-23)
(How to read your favourite book M.
Laxmikanth for Polity)
By
ER NEELESH KUMAR SINGH
ALL INDIA RANK 442 UPSC CSE 2021
JOIN ME AT TELEGRAM – UPSC PRELIMS WITH NEELESH

JOIN ME AT TELEGRAM – UPSC PRELIMS WITH NEELESH (AIR 442 UPSC CSE 2021) https://t.me/UPSCPrelimsWithNeelesh
FREE CSAT COURSE ON YOUTUBE – CIVIL SERVICES WITH NEELESH, Best PYQ Document for Prelims – www.neeleshair442.com,
Best test series on – WWW.CSETOPPER.COM ©Copyright Reserved with the Author
https://t.me/UPSCPrelimsWithNeelesh
Polity PYQ ANALYSIS
How to use this pdf
Simply follow the step-by-step procedure
1. Before reading any chapter, first read the Previous Year Questions asked from the chapter. At this stage, do not see the solution for maximum
benefit
2. Read the Instructions related to the chapter before reading the actual chapter which has been prepared by me based on the analysis of 2011-
2023 UPSC Questions. You must read the chapters as per the instructions.
3. Now read the actual chapter from the book keeping in mind the PYQ questions and my instructions
4. Read the additional information supplied
5. Come back to the original questions again and then solve the questions.
6. See the solutions and see how many you solved correctly. Wherever you were not able to solve, search the solution from the chapter and see
if you are able to do it with the information supplied. In case, it was supplied, and your marked it wrong, mark it in your book for further
revision. If it is not there, note the information in the print out of this PYQ or your book.
7. Work on your weak areas if any in the chapter as per your attempt
8. Congratulations, now you are the master of this chapter
Please note – When I say, some part as important, it does not mean that you should completely ignore other things in the chapter. It is just
that, you must not leave those things which I instruct specially. Prepare those things in more details which I instruct.

JOIN ME AT TELEGRAM – UPSC PRELIMS WITH NEELESH (AIR 442 UPSC CSE 2021) https://t.me/UPSCPrelimsWithNeelesh
FREE CSAT COURSE ON YOUTUBE – CIVIL SERVICES WITH NEELESH, Best PYQ Document for Prelims – www.neeleshair442.com,
Best test series on – WWW.CSETOPPER.COM ©Copyright Reserved with the Author
CAUTION: DO NOT TRY TO DIVERSIFY YOUR SOURCES ENDLESSLY. NO ONE IS ABLE TO READ EVERYTHING. SO, IT IS ALWAYS BETTER TO LIMIT
THE SOURCES FOR SELECTION.
INDEX FOR INDIAN POLITY (BASED ON M. LAXMIKANTH 6TH EDITION)

TOPIC Chapter/Topic Chapter Name/Topic Name Number of


NUMBER Question
1 Chapter 1 Historical Background 8
2 Chapter 2 Making of the Constitution 1
3 Chapter 3 Salient Features of the Constitution 4
4 Chapter 4 Preamble of the Constitution 4
5 Chapter 5 Union And Its Territory 0
6 Chapter 6 Citizenship 1
7 Chapter 7 Fundamental Rights 7
8 Chapter 8 Directive Principles of State Policy 14
9 Chapter 9 Fundamental Duties 5
10 Chapter 10 Amendment Of the Constitution 3
11 Chapter 11 Basic Structure of The Constitution 1
12 Chapter 12, 13, 14, 15 Chapter 12 -Parliamentary System, Chapter 13 – Federal System, Chapter 14 – 12
(System of the Centre-State Relation, Chapter 15 – Inter State Relations
Government)
13 Chapter 16 Emergency Provisions 2
14 Chapter 17 President 6
15 Chapter 18 Vice-President 0
16 Chapter 19, 20, 21 Chapter 19 - Prime Minister, Chapter 20 – Central Council of Ministers, Chapter 3
21 – Cabinet Committees
17 Chapter 22, 23 Chapter 22 – Parliament, Chapter 23 – Parliamentary Committees 33
18 Chapter 24 Chapter 24 - Parliamentary Forums, Chapter 25 – Parliamentary Group 0
19 Chapter 26 Supreme Court 10
JOIN ME AT TELEGRAM – UPSC PRELIMS WITH NEELESH (AIR 442 UPSC CSE 2021) https://t.me/UPSCPrelimsWithNeelesh
FREE CSAT COURSE ON YOUTUBE – CIVIL SERVICES WITH NEELESH, Best PYQ Document for Prelims – www.neeleshair442.com,
Best test series on – WWW.CSETOPPER.COM ©Copyright Reserved with the Author
20 Chapter 27, 28, 29 Chapter 27 - Judicial Review, Chapter 28 – Judicial Activism, Chapter 29 – Public 1
Interest Litigation
21 Chapter 30 Governor 3
22 Chapter 31, 32 Chapter 31 - Chief Minister, Chapter 32 – State Council of Ministers 1
23 Chapter 33 State Legislature 2
24 Chapter 34 High Court 2
25 Chapter 35, 36 Chapter 35 -Tribunals, Chapter 36 – Subordinate Courts 3
26 Chapter 37 Special Provisions for Some States 0
27 Chapter 38 Panchayati Raj 6
28 Chapter 39 Municipalities 1
29 Chapter 40 Union Territories 0
30 Chapter 41 Scheduled And Tribal Areas 1
31 Chapter 42, 43, 44, 45, 46, Chapter 42 - Election Commission, Chapter 43 - Union Public Service 5
47, 48, 49, 50, 51, 52, 53 Commission, Chapter 44 – State Public Service Commission, Chapter 45 –
(Constitutional Bodies) Finance Commission, Chapter 46 – GST, Chapter 47 – NCSC, Chapter 48 – NCST,
Chapter 49 – NCBC, Chapter 50 – Special Officer for Linguistic Minorities,
Chapter 51 – CAG, Chapter 52 – Attorney General of India, Chapter 53 –
Advocate General of the State
32 Chapter 54, 55, 56, 57, 58, Chapter 54 - Niti Aayog, Chapter 55 – NHRC, Chapter 56 – SHRC, Chapter 57 – 3
59, 60, 61, 62, 63 CIC, Chapter 58 – SIC, Chapter 59 – CVC, Chapter 60 – CBI, Chapter 61 – Lokpal
(Non-Constitutional and Lokayukt, Chapter 62 – NIA, Chapter 63 - NDMA
Bodies)
33 Chapter 64, 65, 66, 67, 68 Chapter 64 -Co-operative Societies, Chapter 65 – Official Language, Chapter 66 0
(Other Constitutional – Public Services, Chapter 67 – Rights an Liabilities of the Government, Chapter
Bodies) 68 – Special Provisions relating to Certain Classes
34 Chapter 69, 70, 71, 72, 73, Chapter 69 -Political Parties, Chapter 70 – Role of Regional Parties, Chapter 71 7
74, 75, 76, 77, 78, 79 – Elections, Chapter 72 – Election Laws, Chapter 73 – Electoral Reforms,
(Political Dynamics) Chapter 74 – Voting Behaviour, Chapter 75 – Coalition Government, Chapter 76

JOIN ME AT TELEGRAM – UPSC PRELIMS WITH NEELESH (AIR 442 UPSC CSE 2021) https://t.me/UPSCPrelimsWithNeelesh
FREE CSAT COURSE ON YOUTUBE – CIVIL SERVICES WITH NEELESH, Best PYQ Document for Prelims – www.neeleshair442.com,
Best test series on – WWW.CSETOPPER.COM ©Copyright Reserved with the Author
– Anti-Defection Law, Chapter 77 – Pressure Groups, Chapter 78 – National
Integration, Chapter 79 - Foreign Policy
35 Chapter 80 National Commission to Review the Working of the Constitution 0
(Working of the
Constitution)
36 Appendices Appendix I, II, III, IV, V, VI 2
37 Miscellaneous --- 24

JOIN ME AT TELEGRAM – UPSC PRELIMS WITH NEELESH (AIR 442 UPSC CSE 2021) https://t.me/UPSCPrelimsWithNeelesh
FREE CSAT COURSE ON YOUTUBE – CIVIL SERVICES WITH NEELESH, Best PYQ Document for Prelims – www.neeleshair442.com,
Best test series on – WWW.CSETOPPER.COM ©Copyright Reserved with the Author
TOPIC 1 Chapter 1 (Historical Background)
Total Number of Questions Asked from 2011-23 8
My Instructions related to these chapters Although, this chapter is fully important, in case, you have less time,
focus more on Government of India Act provisions of 1909, 1919
and 1935 (very important). Remember Interim Government and
First Cabinet of Free India.

If you have time, read this chapter and esp the features of the given
acts very sincerely.

PREVIOUS YEAR QUESTIONS RELATED TO THESE CHAPTERS OF TOPICS


S.No. Question and Year Answer Explanation
1 Consider the following statements about ‘the Charter a Statement 1 is correct. Charter Act of 1813 abolished the
Act of 1813’ (2019) trade monopoly of East India Company except for trade in tea
1. It ended the trade monopoly of the East India and trade with China. This opened the Indian Market to all
Company in India except the trade in tea and trade with British Merchants.
China
2. It asserted the sovereignty of the British Crown over Statement 2 is correct. The Charter Act of 1813 renewed the
the Indian territories held by the Company Company’s Charter for 20 years. but asserted the sovereignty
3. The revenues of India were now controlled by the of British Crown over the Company’s territories in India
British Parliament
Which of the statements given above are correct? Statement 3 is incorrect. The East India Company was to
a. 1 and 2 only retain the possession of territories and the revenue for 20
b. 2 and 3 only years more. So, 3 automatically becomes incorrect
c. 1 and 3 only
d. 1, 2 and 3 You are advised to read in detail about this ACT from
Laxmikanth.

JOIN ME AT TELEGRAM – UPSC PRELIMS WITH NEELESH (AIR 442 UPSC CSE 2021) https://t.me/UPSCPrelimsWithNeelesh
FREE CSAT COURSE ON YOUTUBE – CIVIL SERVICES WITH NEELESH, Best PYQ Document for Prelims – www.neeleshair442.com,
Best test series on – WWW.CSETOPPER.COM ©Copyright Reserved with the Author
2 Consider the following statements: (2021) b Statement 1 is not correct. Under this act, though women
1. The Montagu – Chelmsford Reforms of 1919 were not granted voting rights, nor the right to stand in
recommended granting voting rights to all the women elections, the Government of India Act 1919 allowed
above the age of 21 Provincial Councils to determine if women could vote,
2. The Government of India Act of 1935 gave women provided they met stringent property, income, or educational
reserved seats in legislature levels.
Which of the statements given above is/are correct?
a. 1 only Statement 2 is correct. Government of India Act 1935 gave
b. 2 only the women reserved seats in the legislature. Seats were
c. Both 1 and 2 reserved for women through communal representation.
d. Neither 1 nor 2
You are advised to read in detail about this ACT from
Laxmikanth.

3 In the Government of India Act 1919, the functions of c Under the Government of India Act 1919, Subjects were
Provincial Government were divided into ‘Reserved’ divided into two lists: ‘reserved’ which included subjects such
and ‘Transferred’ subjects. Which of the following were as law and order, finance, land revenue, irrigation, etc., and
treated as ‘Reserved’ subjects? (2022) ‘transferred’ subjects such as education, health, local
1. Administration of Justice government, industry, agriculture, excise, etc.
2. Local Self Government
The reserved subjects were to be administered by the
3. Land Revenue
governor through his executive council of bureaucrats, and
4. Police
the transferred subjects were to be administered by
Select the correct answer using the code given below:
ministers nominated from among the elected members of
a. 1, 2 and 3
the legislative council.
b. 2, 3 and 4
c. 1, 3 and 4 Option 1 is correct: Law and order was a reserved subject
d. 1, 2 and 4 under the Government of India Act, 1919.

JOIN ME AT TELEGRAM – UPSC PRELIMS WITH NEELESH (AIR 442 UPSC CSE 2021) https://t.me/UPSCPrelimsWithNeelesh
FREE CSAT COURSE ON YOUTUBE – CIVIL SERVICES WITH NEELESH, Best PYQ Document for Prelims – www.neeleshair442.com,
Best test series on – WWW.CSETOPPER.COM ©Copyright Reserved with the Author
Option 2 is incorrect: Local government was a transferred
subject under the Government of India Act, 1919.
Option 3 is correct: Land Revenue was a reserved subject
under the Government of India Act, 1919.
Option 4 is correct: Police was a reserved subject under the
Government of India Act, 1919.
4 The Montague-Chelmsford Proposals were related to d Self – explanatory.
(2016)
a. Social reforms
b. Educational reforms
c. Reforms in police administration
d. Constitutional reforms

5 Which of the following is/are the principal features(s) of c Separate communal electorate for Muslims were introduced
the Government of India Act, 1919? (2012) by the Government of India Act 1909. Rest are as per the
1. Introduction of dyarchy in the executive government provisions of GOI 1919.
of the provinces
2. Introduction of separate communal electorates for You are advised to read in detail about this ACT from
Muslims Laxmikanth
3. Devolution of legislative authority by the centre to the
province
Select the correct answer using the codes given below
a. 1 only
b. 2 and 3 only
c. 1 and 3 only
d. 1, 2 and 3

JOIN ME AT TELEGRAM – UPSC PRELIMS WITH NEELESH (AIR 442 UPSC CSE 2021) https://t.me/UPSCPrelimsWithNeelesh
FREE CSAT COURSE ON YOUTUBE – CIVIL SERVICES WITH NEELESH, Best PYQ Document for Prelims – www.neeleshair442.com,
Best test series on – WWW.CSETOPPER.COM ©Copyright Reserved with the Author
6 The distribution of powers between the Centre and the c The Government of India (GoI) Act of 1935 provided for a three-
States in the Indian Constitutions is based on the fold enumeration, viz., federal, provincial and concurrent. The
scheme provided in the (2012) present Constitution follows the scheme of this act for the
a. Morley – Minto Reforms, 1909 distribution of powers between the Centre and States but with
b. Montague – Chelmsford Act, 1919 one difference, that is, under this act, the residuary powers were
c. Government of India Act 1935 given neither to the federal legislature nor to the provincial
d. Indian Independence Act, 1947 legislature but to the Governor-General of India. In this respect,
India follows the Canadian precedent.

7 In the context of Indian History, the principle of dyarchy d Government of India Act 1919 divided the provincial subjects
refers to (2017) into two parts – transferred and reserved. Transferred
a. Division of the central legislature into two houses subjects were to be administered by the Governor with the
b. Introduction of double government i.e. Central and aid of Ministers responsible to the legislative council. The
State Governments reserved subjects, on the other hand, were to be
c. Having two sets of rulers; one in London and another administered by the Governor and his executive council
in Delhi without being responsible to the legislative council. This dual
d. Division of the subjects delegated to the provinces scheme of governance was known as ‘Dyarchy’
into two categories

8 In the Federation established by the Government of b Clearly given in Laxmikanth


Indian Act of 1935, residuary powers were given to the
(2018)
a. Federal legislature
b. Governor General
c. Provincial Legislature
d. Provincial Governors

Additional Information you should know


JOIN ME AT TELEGRAM – UPSC PRELIMS WITH NEELESH (AIR 442 UPSC CSE 2021) https://t.me/UPSCPrelimsWithNeelesh
FREE CSAT COURSE ON YOUTUBE – CIVIL SERVICES WITH NEELESH, Best PYQ Document for Prelims – www.neeleshair442.com,
Best test series on – WWW.CSETOPPER.COM ©Copyright Reserved with the Author
1. Sir Ilijah Impey was the first Chief Justice of the Supreme Court which was created due to Regulating Act of 1773. Sir Maurice Gwyer
was the first Chief Justice of the Federal Court in India which was established by the Government of India Act 1935. Harilal J Kania was
the first Chief Justice of Independent India.
2. Principle of Constitutional Autocracy was introduced by the Government of India Act, 1935
3. The President’s power to promulgate ordinance (Art 123) is a relic of Government of India Act, 1935 (Though, in Indian Council Act of
1861, Viceroy got the power to issue ordinance)
4. Burma was separated from India according to the Provisions of Government of India Act, 1935
5. Remember – Indian Council Act of 1861 (Ordinance, portfolio system), Indian Council Act of 1892 (budget, election) and Government of
India Act 1919 (bicameralism, direct election)

TOPIC 2 Chapter 2 (Making of The Constitution)


Total Number of Questions Asked from 2011-23 1
My Instructions related to these chapters You need to look into these topics more sincerely.
1. Know about the composition of the constituent assembly.
2. Remember the details mentioned in the first paragraph of the
topic “WORKING OF THE CONSTITUTION”
3. Remember the functions performed by Constituent Assembly (esp
under the topic Other Functions performed)
4. Remember the Major Committees and Minor Committee (esp Flag
Committee)
5. Drafting Committee is Most Important.
6. Remember the date of enactment vs enforcement of the
constitution

JOIN ME AT TELEGRAM – UPSC PRELIMS WITH NEELESH (AIR 442 UPSC CSE 2021) https://t.me/UPSCPrelimsWithNeelesh
FREE CSAT COURSE ON YOUTUBE – CIVIL SERVICES WITH NEELESH, Best PYQ Document for Prelims – www.neeleshair442.com,
Best test series on – WWW.CSETOPPER.COM ©Copyright Reserved with the Author
PREVIOUS YEAR QUESTIONS RELATED TO THESE CHAPTERS OF TOPICS
S.No. Question and Year Answer Explanation
1 Consider the following statements in respect of the c Constitution Day also known as 'Samvidhan Divas', is
Constitution Day: (2023) celebrated in our country on 26th November every year to
Statement-I The Constitution Day is celebrated on 26th commemorate the adoption of the Constitution of India. The
November every year to promote constitutional values Ministry of Social Justice and Empowerment on 19th
among citizens November 2015 notified the decision of Government of India
Statement-II On 26th November, 1949, the Constituent to celebrate the 26th day of November every year as
Assembly of India set up a Drafting Committee under the 'Constitution Day' to promote Constitution values among
Chairmanship of Dr. B. R. Ambedkar to prepare a Draft citizens. Hence statement 1 is correct.
Constitution of India.
Which one of the following is correct in respect of the Among all the committees of the Constituent Assembly, the
above statements? most important committee was the Drafting Committee set up
(a) Both Statement-I and Statement-II are correct and on August 29, 1947. It was this committee that was entrusted
Statement-II is the correct explanation for Statement-I with the task of preparing a draft of the new Constitution. On
(b) Both Statement-I and Statement-II are correct and 26th November, 1949, the Constitution as adopted. Hence
Statement-II is not the correct explanation for statement 2 is not correct. Hence option (c) is the correct
Statement-I answer.
(c) Statement-I is correct but Statement-II is incorrect
(d) Statement-I is incorrect but Statement-II is correct

Additional Information you should know


1. Though self-evident, know the difference between enactment (26 Nov, 1949) and enforcement/commencement (26 Jan 1950) of the
constitution.

JOIN ME AT TELEGRAM – UPSC PRELIMS WITH NEELESH (AIR 442 UPSC CSE 2021) https://t.me/UPSCPrelimsWithNeelesh
FREE CSAT COURSE ON YOUTUBE – CIVIL SERVICES WITH NEELESH, Best PYQ Document for Prelims – www.neeleshair442.com,
Best test series on – WWW.CSETOPPER.COM ©Copyright Reserved with the Author
TOPIC 3 Chapter 3(Salient Features of The Constitution)
Total Number of Questions Asked from 2011-23 4
My Instructions related to these chapters This chapter is very important. You must read this chapter very
seriously.
Must remember or atleast recognize
1. Structural, philosophical, political parts, sources and its
constituents
2. Features of federation, unitary (non-federal features) and features
of parliamentary govt.
3. Parts of the constitution (Table)
4. Schedules (Table -v v imp)
5. Sources of the constitution (Table v v imp)
6. Remember the schedule 2 (e.g. president, governor etc), schedule
3 (e.g. union minister, member of parliament etc) and schedule 6.
Here UPSC may ask a simple question that like Oath is taken by
President etc. So, don’t get confused. Though it is better to
remember all the schedules

PREVIOUS YEAR QUESTIONS RELATED TO THESE CHAPTERS OF TOPICS


S.No. Question and Year Answer Explanation
1 Which of the following provisions of the Constitution d DPSP (Article 45 – Provides for early childhood care and
of India have a bearing on Education? (2012) education below the age of six years).
1. Directive Principles of State Policy. Under Schedule 11 of the constitutions, there are 29
2. Rural and Urban Local Bodies functional items. Education is included in the item no 17.
3. Fifth Schedule Similarly in schedule 12, there are 18 functional items and
4. Sixth Schedule education is a part of it. Hence, statement 2 is correct.

JOIN ME AT TELEGRAM – UPSC PRELIMS WITH NEELESH (AIR 442 UPSC CSE 2021) https://t.me/UPSCPrelimsWithNeelesh
FREE CSAT COURSE ON YOUTUBE – CIVIL SERVICES WITH NEELESH, Best PYQ Document for Prelims – www.neeleshair442.com,
Best test series on – WWW.CSETOPPER.COM ©Copyright Reserved with the Author
5. Seventh Schedule
Select the correct answer using the code given below: Seventh schedule is related to the distribution of power
a. 1 and 2 only between the centre and state via three list which are Union
b. 3,4 and 5 only List, State List and Concurrent List. Education is mentioned in
c. 1,2 and 5 only the Concurrent List. Hence, seventh schedule is also correct.
d. 1,2,3,4 and 5 Fifth and Sixth schedule is related to the administration and
control of schedule areas. And here itself, the education
comes. So, All the given provisions has a bearing on
Education.

2 The provisions in the Fifth Schedule and the Sixth a Fifth schedule is related to the provisions related to
Schedule in the Constitution of India are made in order administration and control of schedule areas and schedule
to: (2015) tribes.
a. Protect the interests of Scheduled Tribes. Sixth schedule is related to the provisions related to the
b. Determine the boundaries between States. administration of tribal areas in the states of Assam,
c. Determine the powers, authority and responsibilities Meghalaya, Tripura and Mizoram.
of Panchayats. Plz must remember all the schedules from Laxmikanth. It is
d. Protect the interests of all the border States. very important

At present, 10 States namely Andhra Pradesh, Chhattisgarh,


Gujarat, Himachal Pradesh, Jharkhand, Madhya Pradesh,
Maharashtra, Odisha, Rajasthan and Telangana have Fifth
Schedule Areas.

The Fifth Schedule of the Constitution deals with the


administration and control of Scheduled Areas as well as of
Scheduled Tribes residing in any State other than the States of
Assam, Meghalaya, Tripura and Mizoram. Tribal habitations
in the states of Kerala, Tamil Nadu, Karnataka, West Bengal,
JOIN ME AT TELEGRAM – UPSC PRELIMS WITH NEELESH (AIR 442 UPSC CSE 2021) https://t.me/UPSCPrelimsWithNeelesh
FREE CSAT COURSE ON YOUTUBE – CIVIL SERVICES WITH NEELESH, Best PYQ Document for Prelims – www.neeleshair442.com,
Best test series on – WWW.CSETOPPER.COM ©Copyright Reserved with the Author
Uttar Pradesh and Jammu & Kashmir have not been brought
under the Fifth or Sixth Schedule.

3 Under which Schedule of the Constitution of India can b Self-explanatory


transfer of tribal land to private parties for mining be
declared null and void? (2019)
a. Third Schedule
b. Fifth Schedule
c. Ninth Schedule
d. Twelfth Schedule

4 If a particular area is brought under the Fifth Schedule a In the Article 244(1) of the Constitution, expression Scheduled
of the Constitution of India, which one of the following Areas means such areas as the President may by order declare
statements best reflects the consequence of it? (2022) to be Scheduled Areas.
a. This would prevent the transfer of land of tribal
people to non-tribal people. The President may at any time by order:
b. This would create a local self-government body in
that area. • direct that the whole or any specified part of a Scheduled
c. This would convert that area into a Union Territory. Area shall cease to be a Scheduled Area or a part of such
d. The State having such areas would be declared a an area;
Special Category state. • increase the area of any Scheduled Area in a State after
consultation with the Governor of that State;
• alter, but only by way of rectification of boundaries, any
Scheduled Area;
• on any alteration of the boundaries of a State on the
admission into the Union or the establishment of a new
State, declare any territory not previously included in any
State to be, or to form part of, a Scheduled Area;

JOIN ME AT TELEGRAM – UPSC PRELIMS WITH NEELESH (AIR 442 UPSC CSE 2021) https://t.me/UPSCPrelimsWithNeelesh
FREE CSAT COURSE ON YOUTUBE – CIVIL SERVICES WITH NEELESH, Best PYQ Document for Prelims – www.neeleshair442.com,
Best test series on – WWW.CSETOPPER.COM ©Copyright Reserved with the Author
• rescind, in relation to any State of States, any order or orders
made under these provisions and in consultation with the
Governor of the State concerned, make fresh orders
redefining the areas which are to be Scheduled Areas.

Criteria for Declaring Schedule Areas

The criteria followed for declaring an area as Scheduled


Area are

• preponderance of tribal population;


• compactness and reasonable size of the area;
• under-developed nature of the area; and
• marked disparity in economic standard of the people.

These criteria are not spelt out in the Constitution of India


but have become well established. They embody
principles followed in declaring 'Excluded' and 'Partially-
Excluded' Areas under the Government of India Act 1935,
Schedule B of recommendations of the Excluded and
Partially Excluded Areas Sub Committee of Constituent
Assembly and the Scheduled Areas and Scheduled Tribes
Commission 1961.

Special Provisions for Fifth Schedule Areas

• The Governor of each State having Scheduled Areas (SA)


shall annually, or whenever so required by the President,

JOIN ME AT TELEGRAM – UPSC PRELIMS WITH NEELESH (AIR 442 UPSC CSE 2021) https://t.me/UPSCPrelimsWithNeelesh
FREE CSAT COURSE ON YOUTUBE – CIVIL SERVICES WITH NEELESH, Best PYQ Document for Prelims – www.neeleshair442.com,
Best test series on – WWW.CSETOPPER.COM ©Copyright Reserved with the Author
make a report to the President regarding the
administration of Scheduled Areas in that State.
• The Union Government shall have executive powers to
give directions to the States as to the administration of the
Scheduled Areas.
• Para 4 of the Fifth Schedule provides for establishment of
a Tribes Advisory Council (TAC) in any State having
Scheduled Areas. If the President so directs, there will be
established a TAC in a State having Scheduled tribes but
not Scheduled Areas therein, consisting of not more than
twenty members of whom, three-fourths shall be the
representatives of the Scheduled Tribes in the Legislative
Assembly of the State. If the number of representatives of
the STs in the Legislative Assembly of the State is less than
the number of seats in the TAC to be filled by such
representatives, the remaining seats shall be filled by
other members of those Tribes.
• The TAC shall advise on such matters pertaining to the
welfare and the advancement of the STs in the State as
may be referred to them by the Governor.
• The Governor may make rules prescribing or regulating
o the number of members of the Council, the mode of their
appointment and the appointment of the Chairman of the
Council and of the officers and servants thereof,
o the conduct of its meetings and its procedure in general;
and
o all other incidental matters.
• The Governor may, by public notification, direct that any
particular Act of Parliament or of the Legislature of the
JOIN ME AT TELEGRAM – UPSC PRELIMS WITH NEELESH (AIR 442 UPSC CSE 2021) https://t.me/UPSCPrelimsWithNeelesh
FREE CSAT COURSE ON YOUTUBE – CIVIL SERVICES WITH NEELESH, Best PYQ Document for Prelims – www.neeleshair442.com,
Best test series on – WWW.CSETOPPER.COM ©Copyright Reserved with the Author
State shall or shall not apply to a SA or any part thereof in
the State, subject to such exceptions and modifications, as
specified. The Governor may make regulations for the
peace and good government of any area in the State which
is for the time being a SA. Such regulations may
o prohibit or restrict the transfer of land by or among
members of the Scheduled tribes in such area;
o regulate the allotment of land to members of the STs in
such area;
o regulate the carrying on of business as money-lender by
persons who lend money to members of the STs in such
area.
• In making such regulations, the Governor may repeal or
amend any Act of Parliament or of Legislature of the State
or any existing law after obtaining assent of the President.
• No regulations shall be made unless the Governor, in case
a TAC exists, consults such TAC.

Additional Information you should know


Short trick for remember states of Schedule VI (MiMeTA – Mizoram, Meghalaya, Tripura and Assam)

JOIN ME AT TELEGRAM – UPSC PRELIMS WITH NEELESH (AIR 442 UPSC CSE 2021) https://t.me/UPSCPrelimsWithNeelesh
FREE CSAT COURSE ON YOUTUBE – CIVIL SERVICES WITH NEELESH, Best PYQ Document for Prelims – www.neeleshair442.com,
Best test series on – WWW.CSETOPPER.COM ©Copyright Reserved with the Author
TOPIC 4 Chapter 4 (Preamble of the Constitution)
Total Number of Questions Asked from 2011-23 4
My Instructions related to these chapters This chapter is also very important. In your reading, you must
focus/remember
1. Key words mentioned in the Preamble. If possible, remember
sequence. Also, number/types of justice, liberty, equality and
fraternity
2. Date of adoption mentioned in the preamble (vvi)
3. Exact constitutional status of the Indian republic on 26-1-1950
(hint – socialist, secular and integrity word added later)
4. Democratic characteristic of Indian Polity (Under point 4 –
Democracy)
5. Remember, Socialist, Secular and Integrity word were added later
through 42nd Amendment.
6. Currently, Preamble is considered as the part of the constitutions
and hence can be amended

PREVIOUS YEAR QUESTIONS RELATED TO THESE CHAPTERS OF TOPICS


S.No. Question and Year Answer Explanation
1 Which one of the following objectives is not b Remember, the keywords related to each term
embodied in the Preamble to the Constitution of Justice - social, economic and political
India? (2017) Liberty - of thought, expression, belief, faith and worship
a. Liberty of thought Equality of status and of opportunity and to promote among
b. Economic liberty them all.
c. Liberty of expression Fraternity assuring the dignity of the individual and the unity
d. Liberty of belief and integrity of the nation.

JOIN ME AT TELEGRAM – UPSC PRELIMS WITH NEELESH (AIR 442 UPSC CSE 2021) https://t.me/UPSCPrelimsWithNeelesh
FREE CSAT COURSE ON YOUTUBE – CIVIL SERVICES WITH NEELESH, Best PYQ Document for Prelims – www.neeleshair442.com,
Best test series on – WWW.CSETOPPER.COM ©Copyright Reserved with the Author
So, economic liberty is not mentioned in the constitution.

2 The mind of the makers of the Constitution of India is a In the Berubari Union Case (1960), the Supreme Court stated
reflected in which of the following? (2017) that the Preamble was the key to the minds of the framers of
a. The Preamble the Constitutions and showed the general purpose for which
b. The Fundamental Rights they made several provisions in the constitution.
c. The Directive Principles of State Policy
d. The Fundamental Duties

3 The Preamble of the Constitution of India is: (2020) d In the famous Keshavananda Bharati Case (1973), the
a. A part of the Constitution but has no legal effect. Supreme court stated that
b. Not a part of the Constitution and has no legal effect a. Preamble is a part of the constitution
either.
c. A part of the Constitution and has the same legal Two things must be known
effect as any other part. a. The Preamble is neither a source of power to the legislature
d. A part of the Constitution but has no legal effect nor a prohibition upon the powers of legislature.
independently of other parts. b. It is non-justiciable, that is, its provisions are not enforceable
in courts of law.

4 What was the exact constitutional status of India on b On 26th January 1950, the Preamble described the status of
26th January,1950? (2021) India as “Sovereign Democratic Republic”
a. A Democratic Republic. Remember – the word socialist, secular and integrity words
b. A Sovereign Democratic Republic. were added later by the 42nd Amendment Act, 1976.
c. A Sovereign Secular Democratic Republic.
d. A Sovereign Socialist Secular Democratic Republic.

Additional Information you should know


1. Preamble is neither justiciable nor enforceable in the court of law. This implies that court cannot pass orders against the Government
of India to implement the ideas in the Preamble
JOIN ME AT TELEGRAM – UPSC PRELIMS WITH NEELESH (AIR 442 UPSC CSE 2021) https://t.me/UPSCPrelimsWithNeelesh
FREE CSAT COURSE ON YOUTUBE – CIVIL SERVICES WITH NEELESH, Best PYQ Document for Prelims – www.neeleshair442.com,
Best test series on – WWW.CSETOPPER.COM ©Copyright Reserved with the Author
2. As per the principle of Universalism, all individuals are fully and equally human
3. Secularism – Correct expression for the term secular in India is – there is no religion of the state in India and state maintains the
principled distance from each religion.

TOPIC 5 Chapter 5 (UNION AND ITS TERRITORY)


Total Number of Questions Asked from 2011-23 0
My Instructions related to these chapters Though Questions has not been asked from this chapter. It is
moderately important. While reading, you must focus mainly on
1. Parliament Power to reorganize the sates
2. The number of states after State Reorganization Act
3. Sequence of the creation of the states and the year

TOPIC 6 Chapter 6 (CITIZENSHIP)


Total Number of Questions Asked from 2011-23 1
My Instructions related to these chapters This chapter is not very important. While reading this chapter,
Focus on basics instead of recalling each dates.

PREVIOUS YEAR QUESTIONS RELATED TO THESE CHAPTERS OF TOPICS


S.No. Question and Year Answer Explanation
1 With reference to India, consider the following a Statement 1 is correct. In India, there is only one citizenship
statements: (2021) and one domicile.
1. There is only one citizenship and one domicile.
2. A citizen by birth only can become the Head of the Statement 2 is incorrect. In India, both a citizen by birth as well
State. as a naturalized citizen are eligible for the office of President.

JOIN ME AT TELEGRAM – UPSC PRELIMS WITH NEELESH (AIR 442 UPSC CSE 2021) https://t.me/UPSCPrelimsWithNeelesh
FREE CSAT COURSE ON YOUTUBE – CIVIL SERVICES WITH NEELESH, Best PYQ Document for Prelims – www.neeleshair442.com,
Best test series on – WWW.CSETOPPER.COM ©Copyright Reserved with the Author
3. A foreigner once granted the citizenship cannot be This is different from USA, where only a citizen by birth and not
deprived of it under any circumstances. a naturalized citizen is eligible for the office of President.
a. 1 only
b. 2 only Statement 3 is incorrect. A foreigner once granted citizenship
c. 1 and 3 can be deprived of it under circumstances prescribed by the
d. 2 and 3 Citizenship Act 1955. The citizenship act 1955 prescribed three
ways of losing citizenship whether acquired under the act or
prior to it under the Constitution by renunciation, by
termination and by deprivation.

TOPIC 7 Chapter 7 (FUNDAMENTAL RIGHTS)


Total Number of Questions Asked from 2011-23 7
My Instructions related to these chapters This chapter is very important chapter. Read it in details. You must
focus especially on
1. Features of Fundamental Rights
2. Table 7.1 (parts and subparts)
3. Article 14, Article 19, Article 21 (and rights under it – list as per
Menaka Case), writs, right to property
4. Right to Constitutional Remedies
5. Writs – Types and scope
6. Also remember the three rights outside part III – they are
constitutional/legal/non-fundamental rights

JOIN ME AT TELEGRAM – UPSC PRELIMS WITH NEELESH (AIR 442 UPSC CSE 2021) https://t.me/UPSCPrelimsWithNeelesh
FREE CSAT COURSE ON YOUTUBE – CIVIL SERVICES WITH NEELESH, Best PYQ Document for Prelims – www.neeleshair442.com,
Best test series on – WWW.CSETOPPER.COM ©Copyright Reserved with the Author
PREVIOUS YEAR QUESTIONS RELATED TO THESE CHAPTERS OF TOPICS
S.No. Question and Year Answer Explanation
1 Right to Privacy is protected as an intrinsic part of c In K S Puttaswamy vs Union of India (2017) case, the Supreme
Right to Life and Personal Liberty. Which of the Court unanimously recognized that ‘right to privacy’ is
following in the Constitution of India correctly and protected as an intrinsic part of the right to life and personal
appropriately imply the above statement? (2018) liberty under Article 21 and as a part of the freedoms
a. Article 14 and the provisions under the 42nd guaranteed by Part III of the constitution.
Amendment to the Constitution.
b. Article 17 and the Directive Principles of the State
Policy in Part IV.
c. Article 21 and the freedoms guaranteed in Part III.
d. Article 24 and the provisions under the 44th
Amendment to the Constitution.

2 Right to Privacy’ is protected under which Article of c Explained above


the Constitution of India? (2021)
a. Article 15
b. Article 19
c. Article 21
d. Article 29

3 Which one of the following categories of Fundamental d The Constitution under Part III (Fundamental Rights) provide
Rights incorporates protection against untouchability for the Right to Equality. It includes Art 14-18.
as a form of discrimination? (2020) Article 17 of the Right to Equality is Abolition of
a. Right against exploitation. Untouchability.
b. Right to Freedom.
c. Right to Constitutional Remedies.
d. Right to Equality.
JOIN ME AT TELEGRAM – UPSC PRELIMS WITH NEELESH (AIR 442 UPSC CSE 2021) https://t.me/UPSCPrelimsWithNeelesh
FREE CSAT COURSE ON YOUTUBE – CIVIL SERVICES WITH NEELESH, Best PYQ Document for Prelims – www.neeleshair442.com,
Best test series on – WWW.CSETOPPER.COM ©Copyright Reserved with the Author
4 Which of the following are envisaged by the Right c The Right against Exploitation, is provided in the Part III
Against Exploitation in the Constitution of India? (Fundamental Rights) of the Constitution. It includes two
(2017) articles:
1. Prohibition of traffic in human beings and forced Article 23 – Prohibition of traffic in human beings and forced
labour labour
2. Abolition of untouchability. Article 24 – Prohibition of employment of children in factories
3. Protection of the interests of minorities. etc
4. Prohibition of employment of children in factories Rest of the options are not included under the Right Against
and mines. Exploitation.
Select the correct answer using the code given below: Abolition of Untouchability (Article 17) is included in the Right
a. 1,2 and 4 only to Equality
b. 2,3 and 4 only Protection of the interest of minorities (Article 29) is included in
c. 1 and 4 only the Cultural and Educational Rights.
d. 1,2,3 and 4
Instruction – You must revisit the Fundamental Rights Table 7.1
and keep revising it (Category and its parts)

5 Which Article of the Constitution of India safeguards b In the Lata Singh vs State of Uttar Pradesh, the Supreme Court
one’s right to marry the person of one’s choice? (2019) viewed the right to marry as a component of Right to Life
a. Article 19 under Article 21 of the Constitution of India.
b. Article 21 Article 19 is Protection of six rights regarding freedom of speech
c. Article 25 and expression, freedom of assembly, freedom of association,
d. Article 29 freedom of movement, freedom of residence and freedom of
profession.
Article 25 is Freedom of Conscience and free profession,
practice and propagation of religion
Article 29 is Protection of language, script and culture of
minorities.

JOIN ME AT TELEGRAM – UPSC PRELIMS WITH NEELESH (AIR 442 UPSC CSE 2021) https://t.me/UPSCPrelimsWithNeelesh
FREE CSAT COURSE ON YOUTUBE – CIVIL SERVICES WITH NEELESH, Best PYQ Document for Prelims – www.neeleshair442.com,
Best test series on – WWW.CSETOPPER.COM ©Copyright Reserved with the Author
6 What is the position of the Right to Property in India? b On the commencement of the constitution, Right to Property
(2021) was one of the seven fundamental rights under Part III of the
a. Legal right available to citizens only. Constitution. It was dealt by Article 19 (1)(f) and Article 31.
b. Legal right available to any person. Later, 44th Amendment Act of 1978 abolished the right to
c. Fundamental Right available to citizens only. property as a Fundamental Right by repealing Article 19(1)(f)
d. Neither Fundamental Right nor legal right. and Article 31 from Part III. Instead, the Act inserted a new
Article 300A in Part XII under the heading ‘Right to Property’. It
provides that no person shall be deprived of his property except
by authority of law. Thus. the right to property still remains a
legal right or a constitutional right, though no longer a
fundamental right. It is not a part of the basic structure of the
Constitution.

7 A legislation which confers on the executive or a Article 14 of Indian Constitution says that the State shall not
administrative authority an unguided and deny to any person equality before the law or the equal
uncontrolled discretionary power in the matter of protection of the laws within the territory of India. The concept
application of law violates which one of the following of ‘equality before law’ is an element of the concept of ‘Rule of
Articles of the Constitution of India? (2021) Law’. This concept has the following three elements or aspects:
a. Article 14
b. Article 28 • Absence of arbitrary power
c. Article 32 • Equality before the law, that is, equal subjection of all citizens
d. Article 44 to the ordinary law of the land
• The constitution is the result of the rights of the individual
The first and the second elements are applicable to India. This
means ‘equality before law’ under Article 14 connotes the
absence of arbitrary power with the authorities. Thus,
when a legislation confers uncontrolled discretionary powers

JOIN ME AT TELEGRAM – UPSC PRELIMS WITH NEELESH (AIR 442 UPSC CSE 2021) https://t.me/UPSCPrelimsWithNeelesh
FREE CSAT COURSE ON YOUTUBE – CIVIL SERVICES WITH NEELESH, Best PYQ Document for Prelims – www.neeleshair442.com,
Best test series on – WWW.CSETOPPER.COM ©Copyright Reserved with the Author
on any authority, it tends to violate the article 14 of the
constitution.
Unguided and uncontrolled discretionary power is arbitrary
power. So, having it will violate Article 14.
Article 28 – Freedom from attending religious instruction or
worship in certain educational institutions.
Article 32 – Right to Constitutional Remedies
Article 44 – Uniform Civil Code for the citizens

. Additional Information you should know


1. The first demand for fundamental rights came in the form of the “Constitution of India Bill, in 1895. Also popularly known as the Swaraj
Bill 1895, it was written during the emergence of Indian nationalism and increasingly vocal demands by Indians for self-government. In 1928,
the Nehru Commission composing of representatives of Indian political parties proposed constitutional reforms for India that apart from
calling for dominion status for India and elections under universal suffrage, would guarantee rights deemed fundamental, representation for
religious and ethnic minorities, and limit the powers of the government. In 1931, the Indian National Congress (the largest Indian political
party of the time) adopted resolutions committing itself to the defence of fundamental civil rights, as well as socio-economic rights such as
the minimum wage and the abolition of untouchability and serfdom

2. Fundamental Rights are guaranteed against state action. These rights are like the Bill of Rights in USA

3. Three generation of human rights are


a. First Generation of Rights – Civil and Political Rights (underlying theme – liberty) (these are negative rights)
b. Second Generation of Rights – Economic, Social and Cultural Rights (underlying theme – equality) (these are positive rights)
c. Third Generation of Rights – Solidarity Rights. These rights are concerned with society as a whole rather than individual. First and
second generation are individual rights. (Underlying theme – fraternity)

4. Freedom of Press in inherent in Article 19(1)(a) as freedom of speech and expression

JOIN ME AT TELEGRAM – UPSC PRELIMS WITH NEELESH (AIR 442 UPSC CSE 2021) https://t.me/UPSCPrelimsWithNeelesh
FREE CSAT COURSE ON YOUTUBE – CIVIL SERVICES WITH NEELESH, Best PYQ Document for Prelims – www.neeleshair442.com,
Best test series on – WWW.CSETOPPER.COM ©Copyright Reserved with the Author
5. Capital Punishment does not fall within the purview of the Article 21. The Supreme Court of India has declared that death by hanging
(death sentence) is not violative of Article 21 of the Indian Constitution. India stood as a retentionist country that has decided to award
the death penalty only in the “rarest of rare cases”.

6. No person can be compelled to sing National Anthem since


a. It will be a violation of the Right to Freedom of Speech and Expression
b. It will be violation of the Right to Freedom of Conscience and Practice and Propagation of Religion
c. There is no legal provision obliging anyone to sing the National Anthem

7. Under Article 25, the religion Hindu includes Sikh, Jain and Buddhism

TOPIC 8 Chapter 8 (DIRECTIVE PRINCIPLES OF STATE POLICY)


Total Number of Questions Asked from 2011-23 14
My Instructions related to these chapters This chapter is very important. Read it in detail. UPSC tends to ask
from its keywords. In this too, special focus should be on.
1. Features of the DPSP
2. Classification of DPSP (VVVVI)
3. New DPSP and related amendments
4. Directives outside part IV
5. You need to remember articles related to DPSP with classification.
UPSC directly ask questions on it.

PREVIOUS YEAR QUESTIONS RELATED TO THESE CHAPTERS OF TOPICS


S.No. Question and Year Answer Explanation
1 Economic Justice as one of the objectives of the Indian b In the Preamble, it is clearly mentioned – Justice (Social,
Constitution has been provided in: (2013) Economic and Political).

JOIN ME AT TELEGRAM – UPSC PRELIMS WITH NEELESH (AIR 442 UPSC CSE 2021) https://t.me/UPSCPrelimsWithNeelesh
FREE CSAT COURSE ON YOUTUBE – CIVIL SERVICES WITH NEELESH, Best PYQ Document for Prelims – www.neeleshair442.com,
Best test series on – WWW.CSETOPPER.COM ©Copyright Reserved with the Author
a. The Preamble and the Fundamental Rights.
b. The Preamble and the Directive Principles of the Article 38 of DPSP – To promote the welfare of the people by
State Policy. securing a social order permeated by justice – social,
c. The Fundamental Rights and the Directive Principles economic and political
of State Policy.
d. None of the Above

2 Which principle among the following was added to b New Directive Principles were added by 42nd Amendment, 44th
the Directive Principles of State Policy by the 42nd Amendment, 86th Amendment and 97th Amendment.
Amendment to the Constitution? (2017)
a. Equal pay for equal work for both men and women. 42nd Amendment Act of 1976 added four new Directive
b. Participation of workers in the management of Principles to the original list.
industries. They require the state to
c. Right to work, education and public assistance. a. To secure opportunities for healthy development of children
d. Securing living wage and human conditions of work (Article 39)
to workers. b. To promote equal justice and for provide free legal aid to the
poor (Article 39A)
c. To take steps to secure the participation of workers in the
management of industries
d. To protect and improve the environment and to safeguard
forests and wild life (Article 48A)

Instruction – You are advised to remember all the new DPSP


as per the amendments as given in Laxmikanth.

3 Consider the following statements regarding the c Clearly mentioned in Laxmikanth in the features of DPSP (Point
Directive Principles of State Policy: (2015) no 3 and 4).
1. The Principles spell out the socio- economic The Directive Principles constitute a very comprehensive
democracy in the country. economic, social and political programme for a modern
JOIN ME AT TELEGRAM – UPSC PRELIMS WITH NEELESH (AIR 442 UPSC CSE 2021) https://t.me/UPSCPrelimsWithNeelesh
FREE CSAT COURSE ON YOUTUBE – CIVIL SERVICES WITH NEELESH, Best PYQ Document for Prelims – www.neeleshair442.com,
Best test series on – WWW.CSETOPPER.COM ©Copyright Reserved with the Author
2. The provisions contained in these Principles are not democratic state. In brief, DPSP seek to establish economic and
enforceable by any court. social democracy in the country.
Which of the statements given above is/are correct?
a. 1 only The DPSP are non-justiciable in nature, that is, they are not
b. 2 only legally enforceable by the courts for their violation.
c. Both 1 and 2
d. Neither 1 nor 2

4 According to the Constitution of India, which of the c If you have read the chapter of DPSC sincerely, this question
following are fundamental for the governance of the should not confuse you. If it is confusing you, read the chapter
country? (2013) again.
a. Fundamental Rights.
b. Fundamental Duties. Read the point 1 and point 2 in the features of DPSP.
c. Directive Principles of State Policy.
d. Fundamental Rights and Fundamental Duties. Also, As per Article 37 (DPSP) – There principles are
fundamental in the governance of the country and it shall be
the duty of the State to apply their principles in making laws.

5 Consider the following provisions under the Directive b There is a clear classification of the socialist principles,
Principles of State Policy as enshrined in the Gandhian principles and liberal intellectual principles given in
Constitution of India: (2012) Laxmikanth (Page 8.2).
1. Securing for citizens of India a uniform civil code
2. Organizing Village Panchayats You are advised to remember them by heart with classification
3. Promoting cottage industries in rural areas from Laxmikanth.
4. Securing for all workers reasonable leisure and
cultural opportunities
Which of the above are the Gandhian Principles that
are reflected in the Directive Principles of State
Policy?
JOIN ME AT TELEGRAM – UPSC PRELIMS WITH NEELESH (AIR 442 UPSC CSE 2021) https://t.me/UPSCPrelimsWithNeelesh
FREE CSAT COURSE ON YOUTUBE – CIVIL SERVICES WITH NEELESH, Best PYQ Document for Prelims – www.neeleshair442.com,
Best test series on – WWW.CSETOPPER.COM ©Copyright Reserved with the Author
a. 1,2 and 4 only
b. 2 and 3 only
c. 1,3 and 4 only
d. 1,2,3 and 4

6 Consider the following statements: (2017) d DPSP are non-justiciable in nature. It means that they are not
With reference to the Constitution of India, the legally enforceable by the court. Therefore, the government
Directive Principles of State Policy constitute cannot be compelled to implement them. They are simply like
limitations upon the ‘instrument of Instructions’ enumerated in the
1. Legislative function Government of India Act 1935 and are affirmative directions.
2. Executive function Thus, they do not limit the legislative or executive functions.
Which of the above statements is/are correct?
a - 1 only
b - 2 only
c - Both 1 and 2
d - Neither 1 nor 2

7 In the Constitution of India, promotion of b Article 51 (DPSP) – To promote international peace and
international peace and security is included in the: security and maintain just and honorable relation between
(2014) nations.
a. Preamble of the Constitution.
b. Directive Principles of the State Policy.
c. Fundamental Duties.
d. Ninth Schedule.

8 Which part of the Constitution of India declares the a The welfare state is a concept of governance in which the state
ideal of Welfare State? (2020) plays a key role in the protection and promotion of the
a. Directive Principles of State Policy. economic and social wellbeing of its citizens. DPSP embodies
JOIN ME AT TELEGRAM – UPSC PRELIMS WITH NEELESH (AIR 442 UPSC CSE 2021) https://t.me/UPSCPrelimsWithNeelesh
FREE CSAT COURSE ON YOUTUBE – CIVIL SERVICES WITH NEELESH, Best PYQ Document for Prelims – www.neeleshair442.com,
Best test series on – WWW.CSETOPPER.COM ©Copyright Reserved with the Author
b. Fundamental Rights. the concept of Welfare State and not of the police state by
c. Preamble. aiming or economic and political democracy.
d. Seventh Schedule.
Please refer to point 3 of features of Directive Principles in
Laxmikanth (Page 8.1).

9 The ideal of ‘Welfare State’ in the Indian Constitution b Refer to the question above. It shows how UPSC repeats the
is enshrined in its: (2015) question directly.
a. Preamble
b. Directive Principles of State Policy
c. Fundamental Rights
d. Seventh Schedule

10 Under the Indian Constitution, concentration of b As per Article 39(c) of DPSP – Prevention of concentration of
wealth violates. (2021) wealth and means of production.
a. The Right to Equality.
b. The Directive Principles of State Policy.
c. The Right to freedom.
d. The Concept of Welfare.

11 In India, separation of Judiciary from the Executive is b As per Article 50 of DPSP – To separate the judiciary from the
enjoined by: (2020) executive in the public services of the State.
a. The Preamble of the Constitution.
b. A Directive Principle of State Policy.
c. The Seventh Schedule
d. The Conventional Practice.
12 With reference to the provisions contained in Part IV d Part IV of the Constitution of India refers to DPSP.
of the Constitution of India, which of the following DPSP are non-justiciable in nature, that is, they are not legally
statements is/are correct? (2020) enforceable by the courts for their violation. But as per Article
JOIN ME AT TELEGRAM – UPSC PRELIMS WITH NEELESH (AIR 442 UPSC CSE 2021) https://t.me/UPSCPrelimsWithNeelesh
FREE CSAT COURSE ON YOUTUBE – CIVIL SERVICES WITH NEELESH, Best PYQ Document for Prelims – www.neeleshair442.com,
Best test series on – WWW.CSETOPPER.COM ©Copyright Reserved with the Author
1. They shall be enforceable by courts. 37, DPSP are fundamental in the governance of the country and
2. They shall not be enforceable by any court. it shall be the duty of the State to apply these principles in
3. The principles laid down in this part are to making laws.
influence the making of laws by the State. Hence, these principles are to influence the making of laws by
Select the correct answer using the code given below: the state.
a. 1 only
b. 2 only
c. 1 and 3 only
d. 2 and 3 only

13 Other than the Fundamental Rights, which of the d Universal Declaration of Human Rights was proclaimed under
following parts of the Constitution of India UNGA Resolutions 217A in Paris.
reflect/reflects the principles and provisions of the It states that
Universal Declaration of Human Rights, 1948: (2020) a. All human beings are born free and equal in dignity and
1. Preamble. rights.
2. Directive Principles of State Policy. b. They are endowed with reason and conscience and should
3. Fundamental Duties. act towards one another in a spirit of brotherhood.
Select the correct answer using the code given below: Article 3 of the Universal Declaration of Human Rights states
a. 1 and 2 only that everyone has the right to life, liberty and security of
b. 2 only person.
c. 1 and 3 only The Preamble of India also speaks about “EQUALITY OF status
d. 1,2 and 3 and of opportunity; assuring the dignity of the individual and
the unity and integrity”. Hence, 1 is correct.

Article 22 of UHDR asserts that economic, social and cultural


rights are indispensable for human dignity and development
of the human personality.
Similar concepts are also present in the DPSP in the Indian
Constitutions. Hence 2 is correct.
JOIN ME AT TELEGRAM – UPSC PRELIMS WITH NEELESH (AIR 442 UPSC CSE 2021) https://t.me/UPSCPrelimsWithNeelesh
FREE CSAT COURSE ON YOUTUBE – CIVIL SERVICES WITH NEELESH, Best PYQ Document for Prelims – www.neeleshair442.com,
Best test series on – WWW.CSETOPPER.COM ©Copyright Reserved with the Author
Article 29 of the Universal Declaration of Human Rights
mentions duties. A similar concept was inserted in the Indian
Constitution by the 42 nd Constitutional Amendment Act, 1976
under Part IV-A of the Constitution (Article 51A). Hence 3 is
correct.

14 In essence what does ‘Due Process of Law’ Mean? c Due process refers to just, rational and fair treatment under
(2023) the regular judicial proceedings, thus it means the fair
(a) The principle of natural justice application of law. It states that a person cannot be denied their
(b) The procedure established by law life, liberty, or property without adhering to the required legal
(c) Fair application of law procedures and receiving the appropriate protections. The due
(d) Equality before law process of law limits the power of the law and protects an
individual’s rights. Thus, due process upholds a person’s
constitutional rights, which is a legal necessity. Due process
safeguards a person’s rights and regulates the power of the law.

Additional Information you should know/focus


1. The idea of Welfare state is found both in Preamble and DPSP
2. Panchayati Raj system in India is laid down under DPSP
3. Fundamental Rights (Political Democracy) vs DPSP (Social and Economic Democracy)

TOPIC 9 Chapter 9 (FUNDAMENTAL DUTIES)


Total Number of Questions Asked from 2011-23 5
My Instructions related to these chapters This chapter is again important. You must read it in detail. While
reading the chapter, you must focus on
JOIN ME AT TELEGRAM – UPSC PRELIMS WITH NEELESH (AIR 442 UPSC CSE 2021) https://t.me/UPSCPrelimsWithNeelesh
FREE CSAT COURSE ON YOUTUBE – CIVIL SERVICES WITH NEELESH, Best PYQ Document for Prelims – www.neeleshair442.com,
Best test series on – WWW.CSETOPPER.COM ©Copyright Reserved with the Author
1. Remember each fundamental duty. It is important. As you can
see, UPSC asks question simply to recognize the Fundamental duty
2. Only one fundamental duty was added later. You should know it
and the related amendment.
3. Know the features of Fundamental Duties

PREVIOUS YEAR QUESTIONS RELATED TO THESE CHAPTERS OF TOPICS


S.No. Question and Year Answer Explanation
1 Which of the following is/are among the Fundamental c As per Article 46 (DPSP), the state shall promote with special
Duties of citizens laid down in the Indian care the education and economic interest of the weaker
Constitution? (2012) sections of the people and in particular of the SC and the ST and
1. To preserve the rich heritage of our composite shall protect them from social justice and all forms of
culture. exploitation.
2. To protect the weaker sections from social injustice.
3. To develop the scientific temper and spirit of inquiry. Rest are the provisions of Fundamental duties.
4. To strive towards excellence in all spheres of
individual and collective activity. Instruction – You must remember the Fundamental duties
Select the correct answer using the codes given below: from Laxmikanth. UPSC has the tendency to ask it
a. 1 and 2 only
b. 2 only
c. 1,3 and 4 only
d. 1,2,3 and 4

2 “To uphold and protect the Sovereignty, Unity and d Article 51A(c) – To uphold and protect the sovereignty, unity
Integrity of India” is a provision made in the: (2015) and integrity of India.
a. Preamble of the Constitution.
b. Directive Principle of State Policy
c. Fundamental Rights
d. Fundamental Duties
JOIN ME AT TELEGRAM – UPSC PRELIMS WITH NEELESH (AIR 442 UPSC CSE 2021) https://t.me/UPSCPrelimsWithNeelesh
FREE CSAT COURSE ON YOUTUBE – CIVIL SERVICES WITH NEELESH, Best PYQ Document for Prelims – www.neeleshair442.com,
Best test series on – WWW.CSETOPPER.COM ©Copyright Reserved with the Author
3 Under The Constitution of India, which one of the a Plz check the list of the fundamental duties. To vote in public
following is not the fundamental duty? (2011) election is not a fundamental duty.
a. To vote in public elections
b. To develop the scientific temper
c. To safeguard the public property
d. To abide by the Constitution and respect its ideals

4 In the context of India, which one of the following is a First line of the chapter Fundamental duties. Rights and duties
the correct relationship between Rights and Duties? of the citizens are correlative and inseparable.
(2017)
a. Rights are correlative with Duties.
b. Rights are personal and hence independent of
society and Duties.
c. Rights, and Duties, are important for the
advancement of the personality of the citizen.
d. Duties, not Rights, are important for the stability of
the State.

5 Which of the following statements is/ are true of the d Statement 1 is incorrect. Since, fundamental duties are non-
Fundamental Duties of an Indian citizen? (2017) justiciable, there is no legal sanction behind them. There is no
1. A legislative process has been provided to enforce legislative process for the enforcement of fundamental duties.
these duties. It means that their violation i.e. non-performance of the
2. They are correlative to legal duties. fundamental duties is non-punishable.
Select the correct answer using the codes given below:
a. 1 only Statement 2 is incorrect. A legal duty is an obligation created
b. 2 only by law. Breach of legal duties leads to punishment. On the other
c. Both 1 and 2 hand, fundamental duties are simply code of conduct and their
d. Neither 1 nor 2 breach is non-punishable. So, fundamental duties are non-
corelative (mutually related) to legal duties.
JOIN ME AT TELEGRAM – UPSC PRELIMS WITH NEELESH (AIR 442 UPSC CSE 2021) https://t.me/UPSCPrelimsWithNeelesh
FREE CSAT COURSE ON YOUTUBE – CIVIL SERVICES WITH NEELESH, Best PYQ Document for Prelims – www.neeleshair442.com,
Best test series on – WWW.CSETOPPER.COM ©Copyright Reserved with the Author
Additional Information you should know
1. Fundamental duties cannot be enforced by writs
2. They can be used for interpreting ambiguous statutes

TOPIC 10 Chapter 10 (AMENDMENT OF THE CONSTITUTION)


Total Number of Questions Asked from 2011-23 3
My Instructions related to these chapters This chapter is also important esp the procedure.
1. Must focus on the procedure. UPSC tend to twist questions based
on procedure. Remember, prior consent of president not required,
joint sitting is not there, president must give his assent.
2. Know the types of amendment and the list. Amendment by
simple majority is outside scope of Art 368. Remember these
provisions. UPSC may ask like, which can be amended as per Article
368 and could include point such as quorum in parliament. Don’t
fall in trap

PREVIOUS YEAR QUESTIONS RELATED TO THESE CHAPTERS OF TOPICS


S.No. Question and Year Answer Explanation
1 Consider the following Statements: (2013) d The only word is problematic. Most of the times, when only, all,
1. An amendment to the Constitution of India can be every etc words are given, be vigilant. In most of the cases, they
initiated by an introduction of a bill in the Lok Sabha are generally wrong. Don’t fall in trap of UPSC examiners.
only.
2. If such an amendment seeks to make changes in Statement 1 is incorrect. A bill for amendment of the
the federal character of the Constitution, the constitution can be introduced in either house of the

JOIN ME AT TELEGRAM – UPSC PRELIMS WITH NEELESH (AIR 442 UPSC CSE 2021) https://t.me/UPSCPrelimsWithNeelesh
FREE CSAT COURSE ON YOUTUBE – CIVIL SERVICES WITH NEELESH, Best PYQ Document for Prelims – www.neeleshair442.com,
Best test series on – WWW.CSETOPPER.COM ©Copyright Reserved with the Author
amendment also requires to be ratified by the Parliament. So, even Rajya Sabha can introduce the
legislature of all the States of India. amendment of the constitution. Please note here that state
Which of the statements given above is/are correct? legislature cannot initiate an amendment bill.
a. 1 only.
b. 2 only. Statement 2 is incorrect. Those provisions of the constitution
c. Both 1 and 2. which are related to the federal character of the polity can be
d. Neither 1 nor 2. amended by the special majority of the parliament and also
with the consent of half of the state legislature by a simple
majority.

2 Consider the following statements: (2019) b The 39th amendment placed the disputes related to the
th
1. The 44 Amendment to the Constitution of India president, vice-president, prime minister and Speaker beyond
introduced an article placing the election of the Prime the scope of the Judiciary. They are to be decided by such
Minister beyond the judicial review. authority as may be determined by the Parliament. So, it is the
2. The Supreme Court of India struck down the 99th 39th amendment and not the 44th amendment.
Amendment to the Constitution of India as being
violative of the independence of judiciary. The 99th Amendment aimed at establishing National Judicial
Which of the statements given above is/are correct? Appointment Commission which was tasked at appointments
(a) 1 only of judges in the higher judiciary. Later the Supreme Court of
(b) 2 only India struck down the 99th Amendment by upholding the
(c) Both 1 and 2 collegium system. As per Supreme Court, NJAC was violative of
(d) Neither 1 nor 2 the independence of the judiciary.

3 Consider the following statements: (2022) b Constitutional Amendment Bill does not require the prior
1. A bill amending the Constitution requires a prior permission of the President.
recommendation of the President of India. Eliminating option 1, automatically, b becomes correct. Rest
2. When a Constitution Amendment Bill is presented provisions are mentioned in the procedure for the amendment
to the President of India, it is obligatory for the of the constitution in Article 368.
President of India to give his/her assent.
JOIN ME AT TELEGRAM – UPSC PRELIMS WITH NEELESH (AIR 442 UPSC CSE 2021) https://t.me/UPSCPrelimsWithNeelesh
FREE CSAT COURSE ON YOUTUBE – CIVIL SERVICES WITH NEELESH, Best PYQ Document for Prelims – www.neeleshair442.com,
Best test series on – WWW.CSETOPPER.COM ©Copyright Reserved with the Author
3. A Constitution Amendment Bill must be passed by
both the Lok Sabha and the Rajya Sabha by a special
majority and there is no provision for joint sitting.
Which of the statements given above are correct?
a. 1 and 2 only
b. 2 and 3 only
c. 1 and 3 only
d. 1,2 and 3
Additional Information you should know
1. The President power to veto a Bill for amendment of the Constitution has been taken away by 24 th Amendment Bill.
2. In the event of the Upper House rejecting the constitutional amendment bill passed by the lower house or vice versa, the bill will have
to be dropped.

TOPIC 11 Chapter 11 (BASIC STRUCTURE OF THE CONSTITUTION)


Total Number of Questions Asked from 2011-23 1
My Instructions related to these chapters This chapter is also important though UPSC has not asked much
question on it. While reading the chapter, focus on
1. Its emergence. Basically, if it is mentioned in the constitution or
not.
2. Try to recall what things are recognized in the basic structure of
the constitution. UPSC may make a question on it.

JOIN ME AT TELEGRAM – UPSC PRELIMS WITH NEELESH (AIR 442 UPSC CSE 2021) https://t.me/UPSCPrelimsWithNeelesh
FREE CSAT COURSE ON YOUTUBE – CIVIL SERVICES WITH NEELESH, Best PYQ Document for Prelims – www.neeleshair442.com,
Best test series on – WWW.CSETOPPER.COM ©Copyright Reserved with the Author
PREVIOUS YEAR QUESTIONS RELATED TO THESE CHAPTERS OF TOPICS
S.No. Question and Year Answer Explanation
1 Consider the following statements: (2020) d The Supreme Court has defined the Basic Structure.
1. The Constitution of India defines its ‘basic The concept of ‘basic structure’ came into existence in the
structure’ in terms of federalism, secularism, landmark judgement in the Kesavananda Bharati vs State of
fundamental rights and democracy. Kerala Case in 1973. Hence, Statement 1 is not correct.
2. The Constitution of India provides for ‘judicial
review’ to safeguard the citizens’ liberties and to Thus, basic structure owes its existence to the Supreme Court
preserve the ideals on which the Constitution is Judgement and not the Constitution.
based.
Which of the statements given above is/are correct? There is no explicit provision of judicial review in the
a. 1 only Constitution. The phrase ‘Judicial Review Review’ has nowhere
b. 2 only been used in the Constitution. However, the provisions of
c. Both 1 and 2 several Articles explicitly confer the power of judicial review on
d. Neither 1 nor 2 the Supreme Court and the High Court. So, statement 2 is also
not correct.

TOPIC 12 SYSTEM OF THE GOVERNMENT


(Chapter 12 -Parliamentary System, Chapter 13 – Federal System,
Chapter 14 – Centre-State Relation, Chapter 15 – Inter State
Relations)
Total Number of Questions Asked from 2011-23 12
My Instructions related to these chapters This unit is very important. So, read it in detail. While reading,
special focus should be made on
1. Features of parliamentary and presidential government
2. Federal and unitary features of the Central Govt

JOIN ME AT TELEGRAM – UPSC PRELIMS WITH NEELESH (AIR 442 UPSC CSE 2021) https://t.me/UPSCPrelimsWithNeelesh
FREE CSAT COURSE ON YOUTUBE – CIVIL SERVICES WITH NEELESH, Best PYQ Document for Prelims – www.neeleshair442.com,
Best test series on – WWW.CSETOPPER.COM ©Copyright Reserved with the Author
3. Centre-state relation and name of committees. In it,
recommendation of Sarkaria committee is important. Remember
the chronology
4. Interstate relation is important – remember the states involved
in Dispute Tribunals and in Zonal Councils
5. Emergency provisions is also very important (it though has been
dealt in the next section)

In short, this unit is fully important. Read it properly

PREVIOUS YEAR QUESTIONS RELATED TO THESE CHAPTERS OF TOPICS


S.No. Question and Year Answer Explanation
1 Which one of the following is not a feature of Indian d Indian federation is not the result of an agreement among
federalism? (2017) states like the American Federation. Further, the states have
a. There is an independent judiciary in India. no right to secede from the federation. The federation is union
b. Powers have been clearly divided between the because it is indestructible. So, option d is wrong. Rest are the
Centre and the States. features of Indian federalism.
c. The federating units have been given unequal
representation in the Rajya Sabha. The federal features of the Indian Federation include
d. It is the result of an agreement among the a. Dual Polity
federating units. b. Written constitution
c. Division of Powers
d. Supremacy of the Constitution
e. Rigid Constitution
f. Independent Judiciary
g. Bicameralism

JOIN ME AT TELEGRAM – UPSC PRELIMS WITH NEELESH (AIR 442 UPSC CSE 2021) https://t.me/UPSCPrelimsWithNeelesh
FREE CSAT COURSE ON YOUTUBE – CIVIL SERVICES WITH NEELESH, Best PYQ Document for Prelims – www.neeleshair442.com,
Best test series on – WWW.CSETOPPER.COM ©Copyright Reserved with the Author
2 Which one of the following in Indian polity is an a See the explanation above
essential feature that indicates that it is federal in
character? (2021)
(a) The independence of judiciary is safeguarded.
(b) The Union Legislature has elected representatives
from constituent units.
(c) The Union Cabinet can have elected
representatives from regional parties.
(d) The Fundamental Rights are enforceable by Courts
of Law.

3 The Parliament can make any law for whole or any d Article 245 to 255 in Part XI of the Constitution deal with the
part of India for implementing international treaties: legislative relations between the Centre and the States. Under
(2013) it, there are five extraordinary circumstances, when the
a. With consent of all the States. constitution empowers the Parliament to make laws on any
b. With consent of the majority of the States. matter enumerated in the State List
c. With consent of the Sates concerned. a. When Rajya Sabha Passes a Resolution
d. Without the consent of any state. b. During a National Emergency
c. When states make a request
d. To implement international agreements
e. During President Rule

4 In the context of India, which of the following a The features of the parliamentary government in India are the
principles is/are implied institutionally in the following:
parliamentary government? (2013) a. Nominal and Real Executive – The President is the head of
1. Members of the cabinet are members of the the state while the Prime Minister is the head of the
Parliament. Government. So, Statement 3 is wrong. Cabinet is headed by
2. Ministers hold the office till they enjoy confidence the Prime Minister who is the head of the Government and not
in the Parliament. the state.
JOIN ME AT TELEGRAM – UPSC PRELIMS WITH NEELESH (AIR 442 UPSC CSE 2021) https://t.me/UPSCPrelimsWithNeelesh
FREE CSAT COURSE ON YOUTUBE – CIVIL SERVICES WITH NEELESH, Best PYQ Document for Prelims – www.neeleshair442.com,
Best test series on – WWW.CSETOPPER.COM ©Copyright Reserved with the Author
3. Cabinet is headed by the Head of the State.
Select the correct answer using the codes given below: b. Majority Party Rule
a. 1 and 2 only c. Collective Responsibility –The ministers are collectively
b. 3 only responsible to the Parliament in general and the Lok Sabha in
c. 2 and 3 only particular (Article 75). The principle of collective responsibility
d. 1,2 and 3 implies that the Lok Sabha can remove the ministry i.e. council
of ministers headed by prime minister from office by passing a
vote of no confidence. So, they hold officer till they enjoy
confidence in the Parliament. So, statement 2 is correct
d. Political Homogeneity
e. Double Membership – The ministers are members of both
the legislature and the executive. So statement 1 is correct.
f. Leadership of the Prime Minister
g. Dissolution of the Lower House
h. Secrecy

5 There is a Parliamentary System of Government in d See the features of the parliamentary government. In the
India because the: (2015) Collective responsibility principle. It is implied that council of
a. Lok Sabha is elected directly by the people. ministers is responsible to the Lok Sabha.
b. Parliament can amend the Constitution.
c. Rajya Sabha cannot be dissolved.
d. Council of Ministers is responsible to the Lok Sabha.

6 A Parliamentary System of Government is one in b From the principle of collective responsibility which is the
which: (2020) feature of the parliamentary government, it is implied that the
(a) All political parties in the Parliament are government is responsible to the parliament and can be
represented in the Government. removed by it. So, b is the correct answer.
(b) The Government is responsible to the Parliament
and can be removed by it.
JOIN ME AT TELEGRAM – UPSC PRELIMS WITH NEELESH (AIR 442 UPSC CSE 2021) https://t.me/UPSCPrelimsWithNeelesh
FREE CSAT COURSE ON YOUTUBE – CIVIL SERVICES WITH NEELESH, Best PYQ Document for Prelims – www.neeleshair442.com,
Best test series on – WWW.CSETOPPER.COM ©Copyright Reserved with the Author
(c) The Government is elected by the people and can Statement a is incorrect since not all political parties in the
be removed by them. Parliament are represented in the government. There is no
(d) The Government is chosen by the Parliament but such rule.
cannot be removed by it before completion of a fixed
term. Statement c is incorrect. People cannot remove the
Government but the Parliament can remove the Government
by passing no confidence motion.

Statement d is incorrect since Lok Sabha can remove the


government by passing a no confidence motion.

7 The main advantage of the parliamentary form of c In the parliamentary form of govt. in India, the executive is
government is that: (2017) collectively responsible to the legislature. Here, Lok Sabha can
a. The executive and legislature work independently remove the ministry i.e. Council of ministers headed by prime
b. It provides continuity of policy and is more efficient minister from office by passing a vote of no confidence.
c. The executive remains responsible to the legislature
d. The head of the government cannot be changed
without election
8 Consider the following Statements: (2014) b Constitution places restrictions on the authority of the state so
A Constitutional Government is one which that it does not become autocratic. That is why we have
1. Places effective restrictions on individual liberty in Fundamental Rights. Rights are given to the citizens.
the interest of State Authority.
2. Places effective restrictions on the Authority of the
State in the interest of individual liberty.
Which of the statements given above is/ are correct?
a. 1 only
b. 2 only
c. Both 1 and 2
d. Neither 1 nor 2
JOIN ME AT TELEGRAM – UPSC PRELIMS WITH NEELESH (AIR 442 UPSC CSE 2021) https://t.me/UPSCPrelimsWithNeelesh
FREE CSAT COURSE ON YOUTUBE – CIVIL SERVICES WITH NEELESH, Best PYQ Document for Prelims – www.neeleshair442.com,
Best test series on – WWW.CSETOPPER.COM ©Copyright Reserved with the Author
9 We adopted parliamentary democracy based on the c Both the statements are true.
British model, but how does our model differ from The British Parliament is based on the doctrine of sovereignty
that model? (2021) of Parliament. In India, the powers of the parliament to
1. As regards legislation, the British Parliament is legislate is limited by the constitution.
supreme or sovereign but in India, the power of the In India, any matter related to the constitutionality of the
Parliament to legislate is limited. Amendment of an act of the parliament is referred to the
2. In India, matters related to the constitutionality of constitution bench by the Supreme Court.
the Amendment of an Act of the Parliament are
referred to the Constitution Bench by the Supreme
Court.
Select the correct answer using the code given below.
a. 1 only
b. 2 only
c. Both 1 and 2
d. Neither 1 nor 2

10 A Constitutional government by definition is a (2020) d A constitutional government is the government whose powers
a. Government of Legislature. flow from the constitution and is defined and hence limited by
b. Popular Government. the constitution. So, it is the limited government.
c. Multi-party Government.
d. Limited Government.

11 Constitutional government means: (2021) d Explained above.


a. A representative government of a nation with
federal structure.
b. A government whose Head enjoys nominal powers.
c. A government whose Head enjoys real powers.
d. A government limited by the terms of the
Constitution.
JOIN ME AT TELEGRAM – UPSC PRELIMS WITH NEELESH (AIR 442 UPSC CSE 2021) https://t.me/UPSCPrelimsWithNeelesh
FREE CSAT COURSE ON YOUTUBE – CIVIL SERVICES WITH NEELESH, Best PYQ Document for Prelims – www.neeleshair442.com,
Best test series on – WWW.CSETOPPER.COM ©Copyright Reserved with the Author
12 Which one of the following suggested that the c Sarkaria Commission was a three-member body appointed by
Governor should be an eminent person from outside the Central government to recommend on the Centre-State
the State and should be a detached figure without Relations. It suggested that the Governor should be an eminent
intense political links or should not have taken part in person from outside the state and should be a detached figure
politics in the recent past? (2019) without intense political links or should not have taken part in
a. First Administrative Reforms Commission (1966) politics in the recent past.
b. Rajamannar Committee (1969)
c. Sarkaria Commission (1983)
d. National Commission to Review the Working of the
Constitution (2000)

TOPIC 13 Chapter 16 (EMERGENCY PROVISIONS)


Total Number of Questions Asked from 2011-23 2
My Instructions related to these chapters This chapter is very important. Read it fully

PREVIOUS YEAR QUESTIONS RELATED TO THESE CHAPTERS OF TOPICS


S.No. Question and Year Answer Explanation
1 Which of the following are not necessarily the b Article 356: Imposition of the President’s Rule in a state when
consequences of the proclamation of the President’s constitutional machinery fails.
rule in a State? (2017) When President’s Rule is imposed, the President dismisses the
1. Dissolution of the State Legislative Assembly. State Council of Ministers headed by the Chief Minister. Hence,
2. Removal of the Council of Ministers in the State. statement 2 is correct.
3. Dissolution of the local bodies.
Select the correct answer using the code given below: The State Legislative Assembly could either be suspended or
a. 1 and 2 only dissolved. The State Legislative Bills and State Budget are
b. 1 and 3 only passed by the Parliament. The local bodies are not dissolved.
JOIN ME AT TELEGRAM – UPSC PRELIMS WITH NEELESH (AIR 442 UPSC CSE 2021) https://t.me/UPSCPrelimsWithNeelesh
FREE CSAT COURSE ON YOUTUBE – CIVIL SERVICES WITH NEELESH, Best PYQ Document for Prelims – www.neeleshair442.com,
Best test series on – WWW.CSETOPPER.COM ©Copyright Reserved with the Author
c. 2 and 3 only Hence, statement 1 and 3 are not correct.
d. 1,2 and 3

2 If the President of India exercises his power as b The State Legislative Assembly could either be suspended or
provided under article 356 of the constitution in dissolved. The State Legislative Bills and State Budget are
respect of a particular state, then: (2018) passed by the Parliament. Hence option (a) is incorrect.
a. The assembly of the state is automatically dissolved.
b. The powers of the legislature of that state shall be The powers of the State Legislature of that state shall be
exercisable by or under the authority of the exercisable by or under the authority of the Parliament. Hence,
Parliament. option (b) is correct.
c. Article 19 is suspended in that state. Article 19 is not suspended. Hence option (c) is incorrect.
d. The President can make laws relating to that state. The Parliament can make laws relating to that state. Hence,
option (d) is incorrect.

Additional Information you should know


1. Chhattisgarh and Telangana are the only states where President rule has not been imposed so far
2. Maximum number of times President rule has been imposed is in Uttar Pradesh.

TOPIC 14 Chapter 17 (PRESIDENT)


Total Number of Questions Asked from 2011-23 6
My Instructions related to these chapters This chapter is very important. Read it full. Focus more on
1. Election of the President (esp electoral college)
2. Chronology of Presidents
3. Term and Impeachment
4. Executive, Legislative, financial, emergency powers
5. Veto power of the President
6. Ordinance
JOIN ME AT TELEGRAM – UPSC PRELIMS WITH NEELESH (AIR 442 UPSC CSE 2021) https://t.me/UPSCPrelimsWithNeelesh
FREE CSAT COURSE ON YOUTUBE – CIVIL SERVICES WITH NEELESH, Best PYQ Document for Prelims – www.neeleshair442.com,
Best test series on – WWW.CSETOPPER.COM ©Copyright Reserved with the Author
PREVIOUS YEAR QUESTIONS RELATED TO THESE CHAPTERS OF TOPICS
S.No. Question and Year Answer Explanation
1 With reference to the election of the President of a The President of India is elected by the members of an electoral
India, consider the following statements: (2018) college consisting of the elected members of both the Houses
1. The value of the vote of each MLA varies from state of parliament and the elected members of the Legislative
to state. Assemblies of States and of UTs of Delhi and Puducherry.
2. The value of the vote of MPs of the Lok Sabha is
more than the value of the vote of MPs of the Rajya The vote value of each MLA differs from State to State based on
Sabha. a calculation that factors in its population vis-a-vis the number
Which of the statements given above is/are correct? of members in its legislative Assembly.
a. 1 only
b. 2 only There is the fixed value of each vote by an MP of the Rajya
c. Both 1 and 2 Sabha and the Lok Sabha. And their vote value is equal.
d. Neither 1 nor 2

2 Consider the following statements in respect of a Statement 1 is not correct: The President is elected by
election to the President of India: (2023) members of electoral college consisting of:
1. The members nominated to either House of the 1. the elected members of both the Houses of Parliament;
Parliament or the Legislative Assemblies of States are 2. the elected members of the legislative assemblies of the
also eligible to be included in the Electoral College. states (MLAs); and
2. Higher the number of elective Assembly seats, 3. the elected members of the legislative assemblies of the
higher is the value of vote of each MLA of that State. Union Territories of Delhi and Puducherry
3. The value of vote of each MLA of Madhya Pradesh is The nominated MPs and MLAs are not included in the electoral
greater than that of Kerala. college.
4. The value of vote of each MLA of Puducherry is
higher than that of Arunachal Pradesh because the Statement 2 is not correct: Every elected member of the
ratio of total population to total number of elective legislative assembly of a state shall have as many votes as there
are multiples of one thousand in the quotient obtained by
JOIN ME AT TELEGRAM – UPSC PRELIMS WITH NEELESH (AIR 442 UPSC CSE 2021) https://t.me/UPSCPrelimsWithNeelesh
FREE CSAT COURSE ON YOUTUBE – CIVIL SERVICES WITH NEELESH, Best PYQ Document for Prelims – www.neeleshair442.com,
Best test series on – WWW.CSETOPPER.COM ©Copyright Reserved with the Author
seats in Puducherry is greater as compared to dividing the population of the state by the total number of the
Arunachal Pradesh elected members of the assembly.
How many of the above statements are correct?
(a) Only one This can be expressed as: Value of the vote of an MLA of a state
(b) Only two = Total population of the state/ (1000 x total number of
(c) All three elected members in the state legislative assembly).
(d) All four From the equation, it is clear that higher the number of seats in
the legislative assembly, lower the value of the vote of an MLA
of that state (as the number of seats forms the denominator)

The Constitution (Eighty-fourth) Amendment Act, 2001


provides that until the relevant population figures for the first
census to be taken after the year 2026 have been published, the
population of the States for the purposes of calculation of value
of votes for the Presidential Election shall mean the population
as ascertained at the 1971-census. On the basis of the above
formula, the value of a vote of an MLA is given for the following
states and union territories: 1. Madhya Pradesh – 131, 2. Kerala
– 152, 3. Arunachal Pradesh – 8, 4. Puducherry – 16. Hence
statement 3 is not correct and statement 4 is correct

3 Consider the following statements: (2023) d As per Article 71 (2), if the election of a person as President or
1. If the election of the President of India is declared Vice President is declared void by the Supreme court, acts done
void by the Supreme Court of India, all acts done by by him in the exercise and performance of the powers and
him/her in the performance of duties of his/her office duties of the office of President or Vice President, as the case
of President before the date of decision become may be, on or before the date of the decision of the Supreme
invalid. Court shall not be invalidated by reason of that declaration.
2. Election for the post of the President of India can be Hence statement 1 is not correct.
postponed on the ground that some Legislative
JOIN ME AT TELEGRAM – UPSC PRELIMS WITH NEELESH (AIR 442 UPSC CSE 2021) https://t.me/UPSCPrelimsWithNeelesh
FREE CSAT COURSE ON YOUTUBE – CIVIL SERVICES WITH NEELESH, Best PYQ Document for Prelims – www.neeleshair442.com,
Best test series on – WWW.CSETOPPER.COM ©Copyright Reserved with the Author
Assemblies have been dissolved and elections are yet As per Article 71 (4), the election of a person as President or
to take place. Vice President shall not be called in question on the ground of
3. When a Bill is presented to the President of India, the the existence of any vacancy for whatever reason among the
Constitution prescribes time limits within which he/she members of the electoral college electing him. Thus,
has to declare his/her assent. postponement is not an option. Hence statement 2 is not
How many of the above statements are correct? correct.
(a) Only one
(b) Only two As regards the Indian President, the Constitution does not
(c) All three prescribe any time-limit within which he has to take the
(d)None decision with respect to a bill presented to him for his assent.
Hence statement 3 is not correct.

4 Consider the following statements: (2014) a Article 77(3) in the Constitution of India says “The President
1. The President shall make rules for the more shall make rules for the more convenient transaction of the
convenient transaction of the business of the business of the Government of India, and for the allocation
Government of India, and for the allocation among among Ministers of the said business”.
Ministers of the said business.
2. All executive actions of the Government of India Article 77(1) says “All executive action of the Government of
shall be expressed to be taken in the name of the India shall be expressed to be taken in the name of the
Prime Minister. President.”
Which of the statements given above is/ are correct?
a. 1 only
b. 2 only
c. Both 1 and 2
d. Neither 1 nor 2

5 Consider the following statements: (2015) d Article 53 of the Indian Constitution states that the executive
1. The Executive Power of the Union of India is vested power of the Union shall be vested in the President and shall be
in the Prime Minister.
JOIN ME AT TELEGRAM – UPSC PRELIMS WITH NEELESH (AIR 442 UPSC CSE 2021) https://t.me/UPSCPrelimsWithNeelesh
FREE CSAT COURSE ON YOUTUBE – CIVIL SERVICES WITH NEELESH, Best PYQ Document for Prelims – www.neeleshair442.com,
Best test series on – WWW.CSETOPPER.COM ©Copyright Reserved with the Author
2. The Prime Minister is the ex officio Chairman of the exercised by him either directly or through officers subordinate
Civil Services Board. to him in accordance with the Constitution.
Which of the statements given above is/are correct?
a. 1 only The Cabinet Secretary is the ex-officio chairman of the Civil
b. 2 only Services Board.
c. Both 1 and 2
d. Neither 1 nor 2

6 According to the Constitution of India, it is the duty of c The recommendation of the Union Finance Commission is
the President of India to cause to be laid before the caused to be laid before the Parliament by the President (Article
Parliament which of the following? (2012) 281). So, 1 is correct.
1. The Recommendations of the Union Finance
Commission. Report of the Public Accounts Committee is not laid before
2. The Report of the Public Accounts Committee. Parliament by the President. It is the committee to examine the
3. The Report of the Comptroller and Auditor annual audit reports of the CAG which are laid before the
General. Parliament by the President (So, it examines those reports
4. The Report of the National Commission for which CAG laid before parliament through President). So, 2 is
Scheduled Castes. wrong.
Select the correct answer using the codes given
below: The Report of the Comptroller and Auditor General of India is
a. 1 only caused to be laid before the Parliament by the President (Article
b. 2 and 4 only 151). So, 3 is correct.
c. 1,3 and 4 only
d. 1,2,3 and 4 The Report of the National Commission for SCs and STs is
caused to be laid before the Parliament by the President (Article
239). So, 4 is correct.
Additional Information you should know
1. Head of Indian Republic is President. President is the executive head of the State while Prime Minister is the executive head of the
Government
JOIN ME AT TELEGRAM – UPSC PRELIMS WITH NEELESH (AIR 442 UPSC CSE 2021) https://t.me/UPSCPrelimsWithNeelesh
FREE CSAT COURSE ON YOUTUBE – CIVIL SERVICES WITH NEELESH, Best PYQ Document for Prelims – www.neeleshair442.com,
Best test series on – WWW.CSETOPPER.COM ©Copyright Reserved with the Author
TOPIC 15 Chapter 18 (VICE PRESIDENT)
Total Number of Questions Asked from 2011-23 0
My Instructions related to these chapters Although no question has been asked from it but, this chapter is
also important. Focus should be given on
1. Election of the vice-president (esp electoral college)
2. Sequence of vice-presidents
3. Removal (Term of office)

TOPIC 16 Chapter 19 - Prime Minister, Chapter 20 – Central Council of


Ministers, Chapter 21 – Cabinet Committees
Total Number of Questions Asked from 2011-23 3
My Instructions related to these chapters This chapter is also important. Focus should be on
1. Appointment of PM
2. Chief Ministers who became prime ministers
3. Composition of Council of ministers
4. Cabinet committee is not very important. Give it light reading.
Remember, they are extra-constitutional and set up by the PM,
and are of two types – standing (permanent) and ad hoc (set up
for specific task and not permanent). Simply read their features

PREVIOUS YEAR QUESTIONS RELATED TO THESE CHAPTERS OF TOPICS


S.No. Question and Year Answer Explanation
1 The Prime Minister of India, at the time of his/her a The Constitution does not contain any specific procedure for
appointment (2012) the selection and appointment of the Prime Minister.

JOIN ME AT TELEGRAM – UPSC PRELIMS WITH NEELESH (AIR 442 UPSC CSE 2021) https://t.me/UPSCPrelimsWithNeelesh
FREE CSAT COURSE ON YOUTUBE – CIVIL SERVICES WITH NEELESH, Best PYQ Document for Prelims – www.neeleshair442.com,
Best test series on – WWW.CSETOPPER.COM ©Copyright Reserved with the Author
a. Need not necessarily be a member of one of the Article 75 says only that the Prime Minister shall be appointed
Houses of the Parliament but must become a member by the President.
of one of the Houses within six months.
b. Need not necessarily be a member of one of the In 1997, the Supreme court held that a person who is not a
Houses of the Parliament but must become a member member of either House of Parliament can be appointed ad
of the Lok Sabha within six months. Prime Minister for six months, within which, he should become
c. Must be a member of one of the Houses of the a member of either House of Parliament; otherwise he ceases
Parliament. to be the Prime Minister. So, option a is correct.
d. Must be a member of the Lok Sabha.

2 Out of the following statements, choose the one that c Parliamentary Democracy envisages a Cabinet form of
brings out the principle underlying the Cabinet form Government with a President as a titular head (and the
of Government: (2017) Governor in the case of states) with responsibility of the
a. An arrangement for minimizing the criticism against Ministers to the Parliament (or the state legislature). The
the Government whose responsibilities are complex Parliamentary system is also known as the ‘Westminster’ model
and hard to carry out the satisfaction of all. of government, responsible government and Cabinet form of
b. A mechanism for speeding up the activities of the Government. As per Article 75(3), the council of Ministers shall
Government whose responsibilities are increasing day be collectively responsible to the House of the People
by day.
c. A mechanism of parliamentary democracy for
ensuring collective responsibility of the Government to
the people.
d. A device for strengthening the hands of the head of
the Government whose hold over the people is in a
state of decline

3 Consider the following statements: (2022) b Constitution provides that there shall be a Council of Ministers
1. The Constitution of India classifies the ministers with the Prime Minister as head to aid and advise the President
into four ranks viz. Cabinet Minister, Minister of state in exercising his function. The Union Council of Ministers
JOIN ME AT TELEGRAM – UPSC PRELIMS WITH NEELESH (AIR 442 UPSC CSE 2021) https://t.me/UPSCPrelimsWithNeelesh
FREE CSAT COURSE ON YOUTUBE – CIVIL SERVICES WITH NEELESH, Best PYQ Document for Prelims – www.neeleshair442.com,
Best test series on – WWW.CSETOPPER.COM ©Copyright Reserved with the Author
with Independent Charge, Minister of State and comprises three categories of ministers- Cabinet, Ministers of
Deputy Minister. State and Deputy Ministers. The difference between these
2. The total number of ministers in the Union three categories of ministers lies in their respective ranks,
Government, including the Prime Minister, shall not political importance and emoluments. Hence, statement 1 is
exceed 15 percent of the total number of members in not correct.
the Lok Sabha.
Which of the statements given above is/are correct? The Constitution (Ninety-first Amendment) Act in 2003 added
a. 1 only a clause in Article 72 which made a specific provision that, the
b. 2 only total number of Ministers, including Prime Minister, in no case
c. Both 1 and 2 can exceed 15 percent of the total number of Lok Sabha
d. Neither 1 nor 2 members. Hence, statement 2 is correct.

Additional Information you should know


1. The PM of India is appointed (not elected, not selected, not nominated)
2. PM is the president of the Council of Scientific and Industrial Research (CSIR)
3. The office of the Deputy PM is an extra constitutional growth.
4. The Union Council of the Ministers is collectively responsible to LOK SABHA and not RAJYA SABHA
5. The resignation of the PM on losing majority in the Lower house is not mentioned in the constitution. But conventionally he resigns as
he is the head of Council of Ministers which is collectively responsible to the House of the People according to Article 75(3)
6. No confidence Motion can only be introduced in the Lok Sabha and not Rajya Sabha. However, Council of Ministers can move Confidence
Motion to prove their majority in Lok Sabha
7. There is no mention of a ‘no-Confidence Motion’ in the constitution of India
8. Officer of Profit is decided by the Union Parliament for the Union under article 102(1)(a) and by the state legislature for the states under
Article 191(1)(a)
9. Till now, total 6 previous CM finally became the PM. Their names are Morarji Desai, Charan Singh, VP Singh, PV Narasimha Rao, HD
Dewe Gowda and Narendra Modi
10. The word Cabinet (mantrimandal) is mentioned only once in the Constitution of India in Article 352(3)
11. The number of Ministries at the Centre on 15th August 1947 was 18
JOIN ME AT TELEGRAM – UPSC PRELIMS WITH NEELESH (AIR 442 UPSC CSE 2021) https://t.me/UPSCPrelimsWithNeelesh
FREE CSAT COURSE ON YOUTUBE – CIVIL SERVICES WITH NEELESH, Best PYQ Document for Prelims – www.neeleshair442.com,
Best test series on – WWW.CSETOPPER.COM ©Copyright Reserved with the Author
TOPIC 17 Chapter 22 – Parliament, Chapter 23 – Parliamentary Committees
Total Number of Questions Asked from 2011-23 33
My Instructions related to these chapters Parliament chapter is fully important and most important chapter.
Read it in detail and keep revising it regularly. Everything is important
in it.
Also, do not ignore the tables
a.Allocation of seats in PARLIAMENT for States an UTs
b.Seats reserved for SCs and STs in the Lok Sabha (Remember, there
is no reservation for seats in Rajya Sabha)

Parliament Committee chapter has limited importance. Focus more


on
1. Standing Committees and their composition
2. Departmental Standing Committee (its composition – need not
remember all 24 committee. Just its general composition)

PREVIOUS YEAR QUESTIONS RELATED TO THESE CHAPTERS OF TOPICS


S.No. Question and Year Answer Explanation
1 Consider the following statements: (2012) d Union Territory of Delhi and Puducherry are represented in the
1. Union Territories are not represented in the Rajya Rajya Sabha. Rest UTs are not represented. So, 1 is incorrect.
Sabha.
2. It is within the purview of the Chief Election It is not within the purview of the Election Commission of India
Commissioner to adjudicate the election disputes. to adjudicate the election disputes. It is the High Court or the
3. According to the Constitution of India, the Supreme Court which looks into such matter. So, 2 is incorrect.
Parliament consists of the Lok Sabha and the Rajya
Sabha only.

JOIN ME AT TELEGRAM – UPSC PRELIMS WITH NEELESH (AIR 442 UPSC CSE 2021) https://t.me/UPSCPrelimsWithNeelesh
FREE CSAT COURSE ON YOUTUBE – CIVIL SERVICES WITH NEELESH, Best PYQ Document for Prelims – www.neeleshair442.com,
Best test series on – WWW.CSETOPPER.COM ©Copyright Reserved with the Author
Which of the statements given above is/are correct? According to the Constitution of India, the Parliament consists
a. 1 only of the President, the Lok Sabha and the Rajya Sabha. So, 3 is
b. 2 and 3 only incorrect.
c. 1 and 3 only
d. None

2 Regarding the office of the Speaker of Lok Sabha, b The speaker is elected generally in the first meeting of the Lok
consider the following statements: Sabha following general elections. Serving for a term of five
1. He/She holds the office during the pleasure of the years, the speaker is chosen from sitting members of the Lok
President. Sabha (House of the People) and is by convention a member of
2. He/She need not be a member of the House at the the ruling party or alliance.
time of his/her election but has to become a member At the time of the election of the speaker, he necessarily should
of the House within six months from the date of be a member of the Lok Sabha. So, statement 2 is wrong.
his/her election.
3. If he/she intends to resign, the letter of his/her If the Lok Sabha Speaker wants to resign, the letter of his / her
resignation has to be addressed to the Deputy resignation has to be addressed to the Deputy Speaker. So,
Speaker. statement 3 is correct.
Which of the statements given above is/are correct?
a. 1 and 2 only The doctrine of Pleasure of the President is not applicable to
b. 3 only the speaker of Lok Sabha. So, statement 1 is wrong
c. 1,2 and 3
d. None

3 Consider the following statements: (2018) a Usually, the speaker remains in office during the life of the
1. The Speaker of the Legislative Assembly shall assembly. However, he vacates his office earlier in any of the
vacate his/her office if he/she ceases to be member following cases:
of the Assembly. o If he ceases to be a member of the assembly
2. Whenever the Legislative Assembly is dissolved, o If he resigns by writing to the deputy speaker and;
the speaker shall vacate his/her office immediately.
JOIN ME AT TELEGRAM – UPSC PRELIMS WITH NEELESH (AIR 442 UPSC CSE 2021) https://t.me/UPSCPrelimsWithNeelesh
FREE CSAT COURSE ON YOUTUBE – CIVIL SERVICES WITH NEELESH, Best PYQ Document for Prelims – www.neeleshair442.com,
Best test series on – WWW.CSETOPPER.COM ©Copyright Reserved with the Author
Which of the statements given above is/ are correct? o If he is removed by a resolution passed by a majority of all the
a. 1 only then members of the assembly.
b. 2 only o Such a resolution can be moved only after giving 14 days
c. Both 1 and 2 advance notice So, statement 1 is correct.
d. Neither 1 nor 2
Speaker holds office from the date of her election till
immediately before the first meeting of the Legislative
assembly after the dissolution of the one to which he/she was
elected. On the dissolution of the Legislative assembly,
although the Speaker ceases to be a member of the House,
he/she does not vacate his/her office. So, statement 2 is
incorrect.

4 With reference to Deputy Speaker of Lok Sabha, a • The date of election of the Deputy Speaker is fixed by the
consider the following statements: (2022) Speaker. While the date of election of the Speaker is fixed by
1. As per the Rules of Procedure and Conduct of the President. So, statement 1 is correct.
Business in Lok Sabha, the election of Deputy Speaker
shall be held on such date as the Speaker may fix. • It is only by convention that the position of Deputy Speaker is
2. There is a mandatory provision that the election of offered to the opposition party in India. So, statement 2 is
a candidate as Deputy Speaker of Lok Sabha shall be incorrect.
from either the principal opposition party or the
ruling party. • In case of the absence of the Speaker, the Deputy Speaker
3. The Deputy Speaker has the same power as of the presides over the sessions of the Lok Sabha and conducts the
Speaker when presiding over the sitting of the House business in the house. The Deputy Speaker has the same power
and no appeal lies against his rulings. as the Speaker when presiding over the sitting of the House. So,
4. The well-established parliamentary practice statement 3 is correct.
regarding the appointment of Deputy Speaker is that
the motion is moved by the Speaker and duly • The Deputy Speaker is chosen by the Lok Sabha from amongst
seconded by the Prime Minister. its members right after the election of the Speaker has taken
JOIN ME AT TELEGRAM – UPSC PRELIMS WITH NEELESH (AIR 442 UPSC CSE 2021) https://t.me/UPSCPrelimsWithNeelesh
FREE CSAT COURSE ON YOUTUBE – CIVIL SERVICES WITH NEELESH, Best PYQ Document for Prelims – www.neeleshair442.com,
Best test series on – WWW.CSETOPPER.COM ©Copyright Reserved with the Author
Which of the statements above given are correct? place. There is no provision and or established practice of
a. 1 and 3 moving the motion for his election by the speaker and it is
b. 1,2 and 3 seconded by the prime minister. So. statement 4 is incorrect.
c. 3 and 4 only
d. 2 and 4 only

5 Consider the following statements: (2013) b Deputy Chairman of the Rajya Sabha is a member of Rajya
1. The Chairman and the Deputy Chairman of the Sabha whereas the Chairman of the Rajya Sabha is not a
Rajya Sabha are not the members of that House. member. So, option 1 is incorrect.
2. While the nominated members of the two Houses
of the Parliament have no voting right in the Electoral college of Vice President (The Chairman of Rajya
presidential election, they have the right to vote in Sabha) consists of both elected and nominated members of
the election of the Vice- President. parliament where in case of Presidential electoral college, only
Which of the statements given above is/are correct? elected members can vote (Article 63). So, statement 2 is
a. 1 only correct.
b. 2 only
c. Both 1 and 2
d. Neither 1 nor 2

6 For election to the Lok Sabha, a nomination paper can c One of the basic qualifications for candidature at an election to
be filed by: (2017) Parliament or the Legislature of a State is that the person
a. Anyone residing in India. concerned should be an elector. For contesting an election as a
b. A resident of the constituency from which the candidate, a person must be registered as a voter. Sec 4 (d) of
election is to be contested. Representation People Act, 1951 precludes a person from
c. Any citizen of India whose name appears in the contesting unless he is an elector in any parliamentary
electoral roll of a constituency. constituency. Section 5 (c) of R. P. Act, 1951 has a similar
d. Any citizen of India. provision for Assembly Constituencies. So, statement c is
correct

JOIN ME AT TELEGRAM – UPSC PRELIMS WITH NEELESH (AIR 442 UPSC CSE 2021) https://t.me/UPSCPrelimsWithNeelesh
FREE CSAT COURSE ON YOUTUBE – CIVIL SERVICES WITH NEELESH, Best PYQ Document for Prelims – www.neeleshair442.com,
Best test series on – WWW.CSETOPPER.COM ©Copyright Reserved with the Author
7 Consider the following statements: (2017) d The Lok Sabha is the Lower House of the Indian Parliament.
1. In the election for Lok Sabha or State Assembly, the Members of the Lok Sabha are elected by Universal Adult
winning candidate must get at least 50 percent of the Suffrage and a first past the post system to represent their
votes polled, to be declared elected. respective constituencies. There is no such provision where the
2. According to the provisions laid down in the winning candidate must get at least 50 percent of the votes
Constitution of India, in Lok Sabha, the Speaker’s post polled, to be declared elected. But, he must get more votes
goes to the majority party and the Deputy Speaker’s than others. Even if NOTA option is preferred, even then, the
to the Opposition. person with the next highest votes will win. So, statement 1 is
Which of the statements given above is/are correct? incorrect.
a. 1 only
b. 2 only Speaker and Deputy Speaker are elected by the House. This is a
c. Both 1 and 2 convention (not constitutional provision) that usually speaker's
d. Neither 1 nor 2 post goes to the majority party whereas deputy speaker's post
goes to the opposition. So, statement 2 is incorrect.

8 Which of the following is/are the exclusive powers of b Exclusive powers of Lok Sabha
Lok Sabha? (2022) • Only the Lok Sabha can introduce a money bill under Article
1. To ratify the declaration of Emergency. 110.
2. To pass a motion of no-confidence against the • Only the Lok Sabha can introduce a financial bill under
Council of Ministers. Article 110 (1).
3. To impeach the President of India. • Speaker of the Lok Sabha determines whether bills are
Select the correct answer using the code given below: Money Bills, whereas the Chairman of the Rajya Sabha does
a. 1 and 2 not have this authority.
b. 2 only • Union Budget-> the Rajya Sabha can only debate it and not
c. 1 and 3 vote on grant requests.
d. 3 only • A resolution for discontinuance of national emergency can
be passed only by LS.
• To pass a motion of no-confidence against the council of
Ministers
JOIN ME AT TELEGRAM – UPSC PRELIMS WITH NEELESH (AIR 442 UPSC CSE 2021) https://t.me/UPSCPrelimsWithNeelesh
FREE CSAT COURSE ON YOUTUBE – CIVIL SERVICES WITH NEELESH, Best PYQ Document for Prelims – www.neeleshair442.com,
Best test series on – WWW.CSETOPPER.COM ©Copyright Reserved with the Author
Rajya Sabha has the role to play in -
1. To ratify the declaration of Emergency.
2. To impeach the President of India

So, only statement 2 is correct.

Refer to the topic – Position of Rajya Sabha in the chapter.

9 Which of the following special powers have been b The correct option is to pass a resolution empowering the
conferred on the Rajya Sabha by the Constitution of Parliament to make laws in the State List and to create one or
India? (2012) more All India Services.
a. To change the existing territory of a State and to Article 249 allows Rajya Sabha to pass a resolution to empower
change the name of a State. the Parliament to legislate on matters of state list. Similarly,
b. To pass a resolution empowering the Parliament to Article 312 allows similar power to the Rajya Sabha for the
make laws in the State list and to create one or more creation of All India Service.
All India Services.
c. To amend the election procedure of the President
and to determine the pension of the President after
his/her retirement.
d. To determine the functions of the Election
Commission and to determine the number of Election
Commissioners.

10 Rajya Sabha has equal powers with Lok Sabha in: b Rajya Sabha has equal powers with Lok Sabha in Important
(2020) matters like the impeachment of the President, removal of the
a. The matter of creating new All India Services. Vice-President, Constitutional amendments, and removal of
b. Amending the Constitution. the Judges of the Supreme Court and the High Courts.
c. The removal of the Government
JOIN ME AT TELEGRAM – UPSC PRELIMS WITH NEELESH (AIR 442 UPSC CSE 2021) https://t.me/UPSCPrelimsWithNeelesh
FREE CSAT COURSE ON YOUTUBE – CIVIL SERVICES WITH NEELESH, Best PYQ Document for Prelims – www.neeleshair442.com,
Best test series on – WWW.CSETOPPER.COM ©Copyright Reserved with the Author
d. Making cut motions. The two houses of Parliament enjoy co-equal power and status
in all spheres except in financial matters and in regard to the
responsibility of the Council of Ministers which are exclusively
in the domain of Lok Sabha.

REVISIT THE TOPIC – POSITION OF RAJYA SABHA IN


LAXMIKANTH

11 The Parliament of India acquires the power to d Legislation on item in state list in national interest
legislate on any item in the State List in the national
interest If a resolution to that effect is passed by the: • If Rajya Sabha passes a resolution by a majority of not less
(2016) than two-thirds of members present and voting saying that it
a. Lok Sabha by a simple majority of its total is “necessary or expedient in the national interest” that
membership. Parliament should make a law on a matter enumerated in the
b. Lok Sabha by a majority of not less than two-thirds State List, Parliament becomes empowered to make a law on
of its total membership. the subject specified in the resolution, for the whole or any part
c. Rajya Sabha by a simple majority of its total of the territory of India.
membership. • Such a resolution remains in force for a maximum period of
d. Rajya Sabha by a majority of not less than two-thirds one year but this period can be extended by one year at a time
of its members present and voting by passing a similar resolution further.

So, statement d is correct

12 Consider the following statements: (2020) c Article 85(1) of the Constitution empowers the President to
1. The President of India can summon a session of the summon each House of the Parliament to meet at such time
Parliament at such a place as he/she thinks fit. and place as he thinks fit, but six months shall not intervene
2. The Constitution of India provides for three between its last sitting in one session and the date decided for
sessions of the Parliament in a year, but is not its first sitting in the next session. So, statement 1 is correct.
mandatory to conduct all three sessions.
JOIN ME AT TELEGRAM – UPSC PRELIMS WITH NEELESH (AIR 442 UPSC CSE 2021) https://t.me/UPSCPrelimsWithNeelesh
FREE CSAT COURSE ON YOUTUBE – CIVIL SERVICES WITH NEELESH, Best PYQ Document for Prelims – www.neeleshair442.com,
Best test series on – WWW.CSETOPPER.COM ©Copyright Reserved with the Author
3. There is no minimum number of days that the There is no such provision in the Constitution of India for three
Parliament is required to meet in a year. sessions of the Parliament in a year. However, by convention,
Which of the statements given above is/are correct? Parliament meets for three sessions in a year. The longest is the
a. 1 only Budget session, followed by Monsoon session and winter
b. 2 only session. So statement 2 is incorrect.
c. 1 and 3 only
d. 2 and 3 only There is no minimum number of days that Parliament is
required to meet in a year. So, statement 3 is correct.

13 Which of the following are the methods of a A periodic or at least a mid-year review of the programme of
Parliamentary control over public finance in India? the Government against macroeconomic forecasts is not a
(2012) method of Parliamentary control over public finance in India.
1. Placing Annual Financial Statement before the
Parliament. Constitutional Provision for Parliamentary Control over Public
2. Withdrawal of moneys from Consolidated Fund of Purse:-
India only after passing the Appropriation Bill. 1. Article 266
3. Provisions of Supplementary grants and vote-on- All revenues and receipts of government are to go to a
account. 'Consolidated Fund' and moneys can be withdrawn from the
4. A periodic or at least a mis-year review of the 'Fund' only in accordance with laws passed by Parliament.
programme of the Government against
macroeconomic forecasts and expenditure by a 2. Article 112
Parliamentary Budget Office. President to place Annual Financial statement before
5. Introducing Finance Bill in the Parliament. Parliament every financial year.
Select the correct answer using the codes given Charged and voted estimates of expenditure to be shown
below: separately.
a. 1,2,3 and 5 only Expenditure on revenue account to be shown as distinct from
b. 1,2 and 4 only other expenditure.
c. 3,4 and 5 only
d. 1,2,3,4 and 5 3. Article 113
JOIN ME AT TELEGRAM – UPSC PRELIMS WITH NEELESH (AIR 442 UPSC CSE 2021) https://t.me/UPSCPrelimsWithNeelesh
FREE CSAT COURSE ON YOUTUBE – CIVIL SERVICES WITH NEELESH, Best PYQ Document for Prelims – www.neeleshair442.com,
Best test series on – WWW.CSETOPPER.COM ©Copyright Reserved with the Author
Charged expenditure not to be submitted to the vote of
Parliament.
Expenditure of estimates to be submitted in the form of
demands for Grants to Parliament for its assessment.
Prior recommendation of President necessary for placing
demand for Grants.

4. Article 114
Withdrawal of moneys from the Consolidated Fund only after
passing of the Appropriation Bill.

5. Article 115
Provision of supplementary, additional or excess grants.

6. Article 116
Provision of vote on account, vote on credit and exceptional
grants.

7. Article 117
Finance Bill to be introduced in the Parliament with the
recommendation of the President of India

So, only 1, 2, 3 and 5 are correct.

14 A deadlock between the Lok Sabha and the Rajya a To resolve a deadlock between the two Houses, in case of an
Sabha calls for a joint sitting of the Parliament during Ordinary Bill, the Constitution provides for the joint sitting of
the Passage of: (2012) both Houses. Issues in joint sitting are decided by a majority of
1. Ordinary Bill the total number of members of both Houses present and
2. Money Bill
JOIN ME AT TELEGRAM – UPSC PRELIMS WITH NEELESH (AIR 442 UPSC CSE 2021) https://t.me/UPSCPrelimsWithNeelesh
FREE CSAT COURSE ON YOUTUBE – CIVIL SERVICES WITH NEELESH, Best PYQ Document for Prelims – www.neeleshair442.com,
Best test series on – WWW.CSETOPPER.COM ©Copyright Reserved with the Author
3. Constitution Amendment Bill voting. The joint sitting is held in the Central Hall of Parliament
Select the correct answer using the codes given House presided over by the Speaker, Lok Sabha. So, 1 is correct.
below:
a. 1 only However, in the case of a Money Bill, there is no provision in the
b. 2 and 3 only Constitution for a joint sitting of both Houses as Lok Sabha
c. 1 and 3 only clearly enjoys pre-eminence over Rajya Sabha in financial
d. 1,2 and 3 matters. So, 2 is wrong.

As regards a Constitutional Amendment Bill, it has been


provided in the Constitution that such a Bill has to be passed by
the specific majority, as prescribed under article 368 of the
Constitution, by both Houses. There is, therefore, no provision
for resolving a deadlock through joint sitting between the two
Houses in regard to a Constitution Amendment Bill. Once a
Constitutional Amendment Bill is rejected by the other house,
it has to be dropped. So, 3 is wrong.

15 Regarding Money Bill, which of the following c Article 110 – Definition of Money bill (1) For the purposes of
statements is not correct? (2018) this Chapter, a Bill shall be deemed to be a Money Bill if it
a. A bill shall be deemed to be a Money Bill if it contains contains only provisions dealing with all or any of the following
only provisions relating to imposition, abolition, matters, namely
remission, alteration or regulation of any tax. (a) the imposition, abolition, remission, alteration or regulation
b. Money Bill has provisions for the custody of the of any tax;
consolidated fund of India or the contingency fund of (b) the regulation of the borrowing of money or the giving of
India. any guarantee by the Government of India, or the amendment
c. A Money Bill is concerned with the appropriation of of the law with respect to any financial obligations undertaken
money out of the contingency fund of India. or to be undertaken by the Government of India;

JOIN ME AT TELEGRAM – UPSC PRELIMS WITH NEELESH (AIR 442 UPSC CSE 2021) https://t.me/UPSCPrelimsWithNeelesh
FREE CSAT COURSE ON YOUTUBE – CIVIL SERVICES WITH NEELESH, Best PYQ Document for Prelims – www.neeleshair442.com,
Best test series on – WWW.CSETOPPER.COM ©Copyright Reserved with the Author
d. A Money Bill deals with the Regulation of borrowing (c) the custody of the consolidated Fund or the Contingency
of money or giving any guarantee by the government Fund of India, the payment of moneys into or the withdrawal of
of India moneys from any such Fund;
(d) the appropriation of moneys out of the consolidated Fund
of India (not Contingency Fund of India)
(e) the declaring of any expenditure to be expenditure charged
on the Consolidated Fund of India or the increasing of the
amount of any such expenditure
(f) the receipt of money on account of the Consolidated Fund
of India or the Public Account of India or the custody or issue of
such money or the audit of the accounts of the Union or of a
State; or
(g) any matter incidental to any of the matters specified in the
sub-clauses (a) to (f)

A Money Bill is not concerned with the appropriation of


money out of the Contingency Fund of India.
So, c is wrong.

16 When a bill is referred to a joint sitting of both the a A bill which is referred to the joint sitting, is presided by the
Houses of the Parliament, it has to be passed by: Speaker and needs to be passed by a simple majority of the
(2015) members present and voting. So, option a is correct.
a. A simple majority of members present and voting.
b. Three-fourths majority of members present and
voting.
c. Two-thirds majority of the Houses.
d. Absolute majority of the Houses

JOIN ME AT TELEGRAM – UPSC PRELIMS WITH NEELESH (AIR 442 UPSC CSE 2021) https://t.me/UPSCPrelimsWithNeelesh
FREE CSAT COURSE ON YOUTUBE – CIVIL SERVICES WITH NEELESH, Best PYQ Document for Prelims – www.neeleshair442.com,
Best test series on – WWW.CSETOPPER.COM ©Copyright Reserved with the Author
17 Which of the following statements is/are correct? b Lapsing of Bills
(2016) • When the Lok Sabha is dissolved, all business including bills,
1. A Bill pending in the Lok Sabha lapses on its motions, resolutions, notices, petitions and so on pending
prorogation. before it or its committee lapse.
2. A Bill pending in the Rajya Sabha, which has not
been passed by the Lok Sabha, shall not lapse on •The position with respect to lapsing of bills is as follows:
dissolution of the Lok Sabha. o A bill pending in the Lok Sabha lapses (whether originating in
Select the correct answer using the code given below: the Lok Sabha or transmitted to it by the Rajya Sabha).
a. 1 only o A bill passed by the Lok Sabha but pending in the Rajya Sabha
b. 2 only lapses.
c. Both 1 and 2 o A bill not passed by the two Houses due to disagreement and
d. Neither 1 nor 2 if the president has notified the holding of a joint sitting before
the dissolution of Lok Sabha, does not lapse.
o A bill pending in the Rajya Sabha but not passed by the Lok
Sabha does not lapse.
o A bill passed by both Houses but pending assent of the
president does not lapse.
o A bill passes by both Houses but returned by the president for
reconsideration of Houses does not lapse.
A bill pending in the Lok Sabha does not lapse on its
prorogation. Prorogation does not affect any the bills or any
other business pending before the house. So, only option 2 is
correct.

18 What will follow if a Money Bill is substantially a If a Money Bill is substantially amended by the Rajya Sabha, the
amended by the Rajya Sabha? (2013) Lok Sabha may still proceed with the Bill, accepting or not
a. The Lok Sabha may still proceed with the Bill, accepting the recommendations of the Rajya Sabha. Rajya
accepting or not accepting the recommendations of Sabha has only limited power with respect to the money bill. It
the Rajya Sabha.
JOIN ME AT TELEGRAM – UPSC PRELIMS WITH NEELESH (AIR 442 UPSC CSE 2021) https://t.me/UPSCPrelimsWithNeelesh
FREE CSAT COURSE ON YOUTUBE – CIVIL SERVICES WITH NEELESH, Best PYQ Document for Prelims – www.neeleshair442.com,
Best test series on – WWW.CSETOPPER.COM ©Copyright Reserved with the Author
b. The Lok Sabha cannot consider the Bill further. cannot reject or amend a money bill. It can only make the
c. The Lok Sabha may send the Bill to the Rajya Sabha recommendations. So, option a is correct.
for reconsideration.
d. The President may call a joint sitting for passing the
Bill.

19 Consider the following statements: (2015) b Powers which Rajya Sabha has –
1. The Rajya Sabha has no power either to reject or to • The Rajya Sabha has no power either to reject or to amend a
amend a Money Bill. Money Bill. It only can give recommendations which Lok Sabha
2. The Rajya Sabha cannot vote on the Demands for may accept or reject.
Grants. • Rajya Sabha cannot vote on the demand for grants. Only the
3. The Rajya Sabha cannot discuss the Annual Lok Sabha can vote on the demand for grants.
Financial Statement. • Every year the budget (annual financial statement as
Which of the statements given above is /are correct? mentioned in the constitution) is discussed by the Rajya Sabha.
a. 1 only So, discussion is allowed. So, only b is correct.
b. 1 and 2 only
c. 2 and 3 only
d. 1,2 and 3

20 With reference to the Parliament of India, consider d A private member bill is a bill introduced by any member of
the following statements: (2017) Parliament other than a minister. Here, elected or nominated is
1. A private member’s bill is a bill presented by a irrelevant.
Member of Parliament who is not elected but only Till 26-08-2023, a total of 14 private member bills have been
nominated by the President of India. passed. The first private member bill to become a law was the
2. Recently, a private member’s bill has been passed Muslim Wakfs Bill, 1952. The last time a private member bill
in the Parliament of India for the first time in history. was passed was in 1970 and it was the Supreme Court
Which of the statements given above is/are correct? (Enlargement of Criminal Appellate Jurisdiction), Bill, 1968. So,
a. 1 only 1 is correct.
b. 2 only
JOIN ME AT TELEGRAM – UPSC PRELIMS WITH NEELESH (AIR 442 UPSC CSE 2021) https://t.me/UPSCPrelimsWithNeelesh
FREE CSAT COURSE ON YOUTUBE – CIVIL SERVICES WITH NEELESH, Best PYQ Document for Prelims – www.neeleshair442.com,
Best test series on – WWW.CSETOPPER.COM ©Copyright Reserved with the Author
c. Both 1 and 2
d. Neither 1 nor 2

21 Consider the following statements regarding a No- c Constitution of India does not mention about either
Confidence Motion in India: (2014) Confidence or No Confidence Motion. A motion of “No
1. There is no mention of a No- Confidence Motion in Confidence Motion” against the Government can be introduced
the Constitution of India. only in the Lok Sabha only and not in Rajya Sabha. So,
2. A Motion of No- Confidence can be introduced in statement 1 and 2 are correct.
the Lok Sabha only
Which of the statements given above is/ are correct?
a. 1 only
b. 2 only
c. Both 1 and 2
d. Neither 1 nor 2

22 With reference to the Union Government, consider c The Department of Economic Affairs, Ministry of Finance, is
the following statements: (2015) responsible for the preparation of the Union Budget annually.
1. The Department of Revenue is responsible for the So, statement 1 is incorrect.
preparation of the Union Budget that is presented to The Constitution clearly states that ‘No money shall be
the Parliament. withdrawn from the Consolidated Fund of India except under
2. No amount can be withdrawn from the appropriation made by law’. This authorization is given by
Consolidated Fund of India without the authorization Parliament. So, statement 2 is correct.
from the Parliament of India.
3. All the disbursements made from the Public The receipts under Public Account do not constitute normal
Account also need the authorization from the receipts of Government. Parliamentary authorization for
Parliament of India. payments from the Public Account is therefore not required.
Which of the statements given above is/ are correct? So, statement 3 is not correct.
a. 1 and 2 only
b. 2 and 3 only
JOIN ME AT TELEGRAM – UPSC PRELIMS WITH NEELESH (AIR 442 UPSC CSE 2021) https://t.me/UPSCPrelimsWithNeelesh
FREE CSAT COURSE ON YOUTUBE – CIVIL SERVICES WITH NEELESH, Best PYQ Document for Prelims – www.neeleshair442.com,
Best test series on – WWW.CSETOPPER.COM ©Copyright Reserved with the Author
c. 2 only
d. 1,2 and 3

23 In the Parliament of India, the purpose of an a Adjournment motion is introduced in the Parliament to draw
adjournment motion is (2012) the attention of the House to a definite matter of urgent public
a. To allow a discussion on a definite matter of urgent importance and needs the support of 50 members to be
public importance admitted. So, statement 1 is correct.
b. To let opposition members, collect information from
the ministers
c. To allow a reduction of specific amount in demand
for grant
d. To postpone the proceedings to check the
inappropriate or violent behaviour on the part of some
members

24 What is the difference between “vote-on-account” b Every year on the first working day of February, the
and “interim budget”? (2011) Government presents the annual Budget for the next financial
1. The provision of vote-on-account is used by a year. However, if it is an election year, the government would
regular Government, while an interim budget is a instead present a vote on account (VOA) or an interim budget.
provision used by a caretaker Government. During an election year it is not practical for the ruling
2. A vote-on-account only deals with the expenditure government or for the new government taking charge after the
in Government’s budget while an interim budget elections to prepare or debate on the full budget and pass it
includes both expenditure and receipts. before the new financial year begins.
Which of the statements given above is/are correct? Hence, the outgoing government (and not the caretaker
a. 1 only Government) would announce an interim budget or vote on
b. 2 only account in February which will be followed by a full budget by
c. Both 1 and 2 the new government in a few months' time after the election.
d. Neither 1 nor 2 Vote on Account contains just the government's expenses,
whereas the interim Budget deals with receipts and payments.
JOIN ME AT TELEGRAM – UPSC PRELIMS WITH NEELESH (AIR 442 UPSC CSE 2021) https://t.me/UPSCPrelimsWithNeelesh
FREE CSAT COURSE ON YOUTUBE – CIVIL SERVICES WITH NEELESH, Best PYQ Document for Prelims – www.neeleshair442.com,
Best test series on – WWW.CSETOPPER.COM ©Copyright Reserved with the Author
So, 1 is incorrect while 2 is correct.

25 When the Annual Budget is not passed by the Lok d When a policy statement like the Annual Budget is not passed
Sabha, (2011) by the Lok Sabha, then it is equivalent to losing vote of
a. The Budget is modified and presented again. confidence of the ruling party or it is same as passing on no-
b. The Budget is referred to the Rajya Sabha for confidence motion. In this case, the government i.e. Prime
suggestions. Minister should resign along with his Council of Minister. So,
c. The Union Finance Minister is asked to resign. statement d is correct.
d. The Prime Minister submits the resignation of
Council of Ministers

26 All revenues received by the Union Government by c All revenues received by Union Government by way of taxes
way of taxes and other receipts for the conduct of and other receipts for the conduct of Government businesses
Government business are credited to the (2011) are credited to the Consolidated Fund of India. So, option a is
a. Contingency Fund of India correct.
b. Public Account
c. Consolidated Fund of India
d. Deposits and Advances Fund

27 The Authorization for the withdrawal of funds from b As per Article 283, the authorization for the withdrawl of funds
the Consolidated Fund of India must come from from the Consolidated fund of India must come from the
(2011) Parliament of India. For the states, the authorization comes
a. The President of India from the state legislature. So, option b is correct.
b. The Parliament of India
c. The Prime Minister of India
d. The Union Finance Minister

JOIN ME AT TELEGRAM – UPSC PRELIMS WITH NEELESH (AIR 442 UPSC CSE 2021) https://t.me/UPSCPrelimsWithNeelesh
FREE CSAT COURSE ON YOUTUBE – CIVIL SERVICES WITH NEELESH, Best PYQ Document for Prelims – www.neeleshair442.com,
Best test series on – WWW.CSETOPPER.COM ©Copyright Reserved with the Author
28 Consider the following statements: (2013) b Parliamentary Committee on Public accounts consists of 22
The Parliamentary Committee on Public Accounts members (15 from Lok Sabha and 7 from Rajya Sabha). Rest of
1. Consists of not more than 25 members of the Lok the options are correct.
Sabha.
2. Scrutinizes appropriation and finance accounts of
the Government.
3. Examines the report of the Comptroller and
Auditor General of India.
Which of the statements given above is/ are correct?
a. 1 only
b. 2 and 3 only
c. 3 only
d. 1,2 and 3

29 Which one of the following is the largest Committee B - Committee of Public Accounts has 22 members (15 from Lok
of the Parliament? (2014) Sabha + 7 from Rajya Sabha)
a. The Committee on Public Accounts. - Committee on Estimates has 30 members (All 30 from Lok Sabha
b. The Committee on Estimates. only)
c. The Committee on Public Undertakings. - Committee on Public Undertaking has 22 members (15 from Lok
d. The Committee on Petitions. Sabha + 7 from Rajya Sabha)
- Committee on Petitions: Lok Sabha Petition Committee has 15
members while Rajya Sabha Petition Committee has 10
members So, b is correct.

30 With reference to the Parliament of India which of the b Committee on Subordinate Legislation scrutinizes and reports
following Parliamentary Committees scrutinizes and to the House to know whether the powers of making
reports to the House whether the powers to make regulations, rules, sub-rules, by-laws etc. conferred by the
regulations, rule, sub-rules, by-laws, etc. conferred by Constitution or delegated by the Parliament are being properly
the Constitution or delegated by the Parliament are exercised by the executive within the scope of such delegation.
JOIN ME AT TELEGRAM – UPSC PRELIMS WITH NEELESH (AIR 442 UPSC CSE 2021) https://t.me/UPSCPrelimsWithNeelesh
FREE CSAT COURSE ON YOUTUBE – CIVIL SERVICES WITH NEELESH, Best PYQ Document for Prelims – www.neeleshair442.com,
Best test series on – WWW.CSETOPPER.COM ©Copyright Reserved with the Author
being properly exercised by the Executive within the The committee in both the Houses consists of 15 members. So,
scope of such delegation? (2018) statement b is correct.
a. Committee on Government Assurances
b. Committee on Subordinate Legislation Instruction – Read the committees functions from Laxmikanth.
c. Rules Committee Focus more on Financial Committee and Departmental
d. Business Advisory Committee Standing Committee (general composition).

Rest try to link the explanation of each committee with the


committee name. This will help you recall their functions.

31 Consider the following statements: a The Ninth Schedule was added to the Constitution by the First
1. The Parliament of India can place a particular law Amendment Act 1951, to protect land reform and other laws
in the Ninth Schedule of the Constitution of India. included in it from judicial scrutiny on the ground of violation
2. The validity of a law placed in the Ninth Schedule of Fundamental Rights. However, in 2007, the Supreme Court
cannot be examined by any court and no judgement ruled that the laws included in this schedule after April 24,
can be made on it. 1973, are now open to judicial review. Also, the parliament of
Which of the statements given above is/are correct? India is empowered to place a particular law in the ninth
a. 1 only schedule of the Constitution of India but it is open to judicial
b. 2 only review as per the current position. So, statement 1 is correct
c. Both 1 and 2 while 2 is incorrect.
d. Neither 1 nor 2

32 The Parliament of India exercises control over the d Parliament of India exercises the control over the functioning of
functions of the Council of Ministers through: (2017) the Council of Ministers by various measures like committee,
1. Adjournment motion question hours, zero hour etc. All the given options are true in
2. Question Hour this regard
3. Supplementary questions
Select the correct answer using the code given below:
a. 1 only
JOIN ME AT TELEGRAM – UPSC PRELIMS WITH NEELESH (AIR 442 UPSC CSE 2021) https://t.me/UPSCPrelimsWithNeelesh
FREE CSAT COURSE ON YOUTUBE – CIVIL SERVICES WITH NEELESH, Best PYQ Document for Prelims – www.neeleshair442.com,
Best test series on – WWW.CSETOPPER.COM ©Copyright Reserved with the Author
b. 2 and 3 only
c. 1 and 3 only
d. 1, 2 and 3

33 With reference to Finance Bill and Money Bill in the a A Finance Bill is a type Money Bill as defined in Article 110 (a)
Indian Parliament consider the following statements: of the Constitution.
(2023) ● Finance Bill deals with the proposals of the government for
1. When the Lok Sabha transmits Finance Bill to the levy of new taxes, modification of the existing tax structure or
Rajya Sabha, it can amend or reject the Bill. continuance of the existing tax structure beyond the period
2. When the Lok Sabha transmits Money Bill to the approved by Parliament are submitted to Parliament through
Rajya Sabha, it cannot amend or reject the Bill, it can this bill. So, it is introduced as a part of the Annual Financial
only make recommendations. Statement (i.e. Budget) under Article 112. The Finance Bill is
3. In the case of disagreement between the Lok Sabha accompanied by a Memorandum containing explanations of
and the Rajya Sabha, there is no joint sitting for Money the provisions included in it. The Finance Bill can be introduced
Bill, but a joint sitting becomes necessary for Finance only in Lok Sabha. However, the Rajya Sabha can only
Bill. recommend amendments in the Bill, it can not amend or reject.
How many of the above statements are correct? The bill has to be passed by the Parliament within 75 days of its
(a) Only one introduction. Hence statement 1 is not correct and statement
(b) Only two 2 is correct.
(c) All three
(d) None As a finance bill is a money bill so no joint sitting of the two
houses is allowed with regard to a finance bill under Article 108.
Hence statement 3 is not correct

Additional Information you should know


1 In the first Lok Sabha, the party with the highest seat after Congress was Communist Party of India with 16 seats.
2 There is no reservation of seat in Rajya Sabha
3 Session of Lok Sabha is called for at-least two times in a year
4 The Union Territories get representation in – both houses of the parliament
JOIN ME AT TELEGRAM – UPSC PRELIMS WITH NEELESH (AIR 442 UPSC CSE 2021) https://t.me/UPSCPrelimsWithNeelesh
FREE CSAT COURSE ON YOUTUBE – CIVIL SERVICES WITH NEELESH, Best PYQ Document for Prelims – www.neeleshair442.com,
Best test series on – WWW.CSETOPPER.COM ©Copyright Reserved with the Author
5 The tenure of 5th Lok Sabha was about 6 years
6 Uttar Pradesh has the highest number of seats both in Lok Sabha and Rajya Sabha
7 In India, Lok Sabha speaker and Deputy Speaker is chosen (not nominated, not elected, not appointed)
8 First tribal speaker of Lok Sabha was P A Sangma
9 The LOK SABHA secretariat comes under the direct control of the Speaker of the LOK SABHA. It is independent of the Government.
10 Attorney General of India is appointed (not elected, not selected, not chosen etc)
11 Rajya Sabha is called Permanent House because it cannot be dissolved
12 Right to dissolve the RAJYA SABHA is vested in - (NONE – IT CANNOT BE DISSOLVED)
13 A member is not to cross the floor. It means passing between the chair and another member while the latter is speaking.
14 Interpellation is a parliamentary procedure of demanding that a government official explains some act or policy
15 The speaker can stop a MP from speaking and let another member speak. This phenomenon is called YIELDING THE FLOOR
16 Zero Hour is an Indian Innovation and is not mentioned in Constitution. It is not mentioned in the Rules of Procedure too
17 Provisions of Imposition of fines and penalties is not a part of money Bill
18 If budget is disclosed before introducing in the Legislative Assembly, the finance minister will have to resign
19 Right to Information is a legal right (not constitutional right, not political right, not social right)

TOPIC 18 Chapter 24 - Parliamentary Forums, Chapter 25 – Parliamentary


Group
Total Number of Questions Asked from 2011-23
My Instructions related to these chapters 0
These chapters are not important. Just have a brief reading esp.
their composition and move forward

TOPIC 19 Chapter 26 (SUPREME COURT)


Total Number of Questions Asked from 2011-23 10
My Instructions related to these chapters It is also very important chapter. Read it in detail

JOIN ME AT TELEGRAM – UPSC PRELIMS WITH NEELESH (AIR 442 UPSC CSE 2021) https://t.me/UPSCPrelimsWithNeelesh
FREE CSAT COURSE ON YOUTUBE – CIVIL SERVICES WITH NEELESH, Best PYQ Document for Prelims – www.neeleshair442.com,
Best test series on – WWW.CSETOPPER.COM ©Copyright Reserved with the Author
PREVIOUS YEAR QUESTIONS RELATED TO THESE CHAPTERS OF TOPICS
S.No. Question and Year Answer Explanation
1 Who/ which of the following is the custodian of the d Custodian of the Constitution of India
Constitution of India? (2015) Supreme Court of India is the primary custodian of the Indian
a. The President of India. Constitution, while also being its interpreter and guardian.
b. The Prime Minister of India. Parliament enjoys the authority to amend the Constitution; the
c. The Lok Sabha secretariat. Supreme Court has the authority to examine the validity of
d. The Supreme Court of India constitutional amendments. The Supreme Court ensures that
the other branches of government perform their
responsibilities in accordance with the Constitution. So, a is
correct.
2 What is the provision to safeguard the autonomy of a Under Article 124(2) while appointing a Supreme Court Judge,
the Supreme Court of India? (2012) the President of India has to consult the Chief Justice of India.
1. While appointing the Supreme Court Judges, the So, statement 1 is correct.
President of India has to consult the Chief Justice of
India. The Supreme Court Judges are not removed by the Chief Justice
2. The Supreme Court Judges can be removed by the rather they are removed by the order of the President.
Chief Justice of India only. However, the President can issue the removal order only after
3. The salaries of the Judges are charged on the an address by Parliament has been presented to him in the
Consolidated Fund of India to which the legislature same session for such remove. The address must be supported
does not have to vote. by a special majority of each House of Parliament. The grounds
4. All appointments of officers and staffs of the for removal are two – proved misbehaviour or incapacity. So,
Supreme Court of India are made by the Government statement 2 is incorrect.
only after consulting the Chief Justice of India.
Which of the statements given above is/are correct? The salaries, allowances and pensions payable to the judges of
a. 1 and 3 only the Supreme Court are charged on the Consolidated Fund of
b. 3 and 4 only India as mandated by the Article 112(d)(1). This is charged on
c. 4 only
JOIN ME AT TELEGRAM – UPSC PRELIMS WITH NEELESH (AIR 442 UPSC CSE 2021) https://t.me/UPSCPrelimsWithNeelesh
FREE CSAT COURSE ON YOUTUBE – CIVIL SERVICES WITH NEELESH, Best PYQ Document for Prelims – www.neeleshair442.com,
Best test series on – WWW.CSETOPPER.COM ©Copyright Reserved with the Author
d. 1,2,3 and 4 the Consolidated fund of India and hence are non-votable by
the Parliament. So, statement 3 is correct.

Chief Justice of India can appoint officers and staff of the


Supreme Court without any interference from the Executive or
Legislature. So, statement 4 is incorrect.

Refer to the topic – Independence of Supreme Court (in the


chapter Supreme Court) and remember the provisions for
independence.

3 Which of the following are included in the original c Jurisdiction of the Supreme Court is threefold:
jurisdiction of the Supreme Court? (2012) (i)Original, (ii) Appellate and (iii) Advisory.
1. A dispute between the Government of India and
one or more States. As a federal court, the Supreme Court decides the disputes
2. A dispute regarding elections to either House of the between different units of Indian Federation. The Supreme
Parliament or that of a Legislature of a State. Court has exclusive original jurisdiction for disputes
3. A dispute between the government of India and a a. Between the Centre and one or more states; or
Union Territory b. Between the Centre and any state of states on one side and
4. A dispute between two or more States. one or more other states on the other side; or
Select the correct answer using the codes given below: c. Between two or more states
a. 1 and 2 So, 1 and 4 are correct.
b. 2 and 3
c. 1 and 4
d. 3 and 4

4 The Power of the Supreme Court of India to decide c Refer explanation above.
disputes between the Centre and the State falls under
its: (2014)
JOIN ME AT TELEGRAM – UPSC PRELIMS WITH NEELESH (AIR 442 UPSC CSE 2021) https://t.me/UPSCPrelimsWithNeelesh
FREE CSAT COURSE ON YOUTUBE – CIVIL SERVICES WITH NEELESH, Best PYQ Document for Prelims – www.neeleshair442.com,
Best test series on – WWW.CSETOPPER.COM ©Copyright Reserved with the Author
a. Advisory jurisdiction.
b. Appellate Jurisdiction
c. Original jurisdiction
d. Writ Jurisdiction

5 The power to increase the number of judges in the b Power to increase number of judges in Supreme court is vested
Supreme Court of India is vested in: (2014) in the Parliament. The Parliament has increased the number of
a. The President of India other judges progressively to ten in 1956, to thirteen in 1960,
b. The Parliament to seventeen in 1977, to twenty-five in 1986, to thirty in 2008
c. The Chief Justice of India and to thirty-three in 2019. So, a is correct.
d. The Law Commission

6 Consider the following statements: (2022) b In 1961, a committee headed by H N Sanyal was appointed to
1. Pursuant to the report of H.N. Sanyal Committee, examine the application of contempt laws in India. Pursuant to
the Contempt of Courts Act, 1971 was passed. this report, the Contempt of Courts Act, 1971 was passed. So,
2. The Constitution of India empowers the Supreme statement 1 is correct.
Court of India and the High Courts to punish for
contempt of themselves. Article 129 of the Constitution of India makes the Supreme
3. The Constitution of India defines Civil Contempt Court a court of record and confers power to punish for
and Criminal Contempt. contempt of itself. Similarly, Article 215 makes the High Court a
4. In India, the Parliament is vested with the powers court of record and confers power to punish for contempt of
to make laws on Contempt of Court. itself.
Which of the statements given above is/are correct?
a. 1 and 2 only The expression ‘contempt of court’ has not been defined in the
b. 1,2 and 4 constitution. The definition of the Civil Contempt and Criminal
c. 3 and 4 only Contempt has been given in the Contempt of Courts Act, 1971.
d. 3 only
In India, the Parliament is vested with the powers to make laws
on contempt of court. Accordingly, the Parliament enacted
JOIN ME AT TELEGRAM – UPSC PRELIMS WITH NEELESH (AIR 442 UPSC CSE 2021) https://t.me/UPSCPrelimsWithNeelesh
FREE CSAT COURSE ON YOUTUBE – CIVIL SERVICES WITH NEELESH, Best PYQ Document for Prelims – www.neeleshair442.com,
Best test series on – WWW.CSETOPPER.COM ©Copyright Reserved with the Author
Contempt of Court Act, 1971. So, statement 1, 2 and 4 only are
correct.

7 With reference to the Constitution of India, b Article 142 of the Constitution provides that “Supreme Court in
prohibitions or limitations or provisions contained in the exercise of its jurisdiction may pass such decree or make
ordinary laws cannot act as prohibitions or limitations such order as is necessary for doing complete justice in any
on the constitutional powers under Article 142. It cause or matter pending before it…”
could mean which one of the following? (2019)
a. The decision taken by the Election commission of In the Union Carbide case, the Supreme court stated that
India while discharging its duties cannot be challenged ‘Prohibitions or limitations or provisions contained in ordinary
in any court of law. laws cannot, ipso facto, act as prohibitions or limitations on the
b. The Supreme Court of India is not Constrained in the constitutional power under Article 142.” It implies that to do
exercise of its powers by laws made by the Parliament. complete justice, the Supreme Court can override the laws
c. In the event of grave financial crisis in the country, made by Parliament. So, statement 2 is correct.
the President of India can declare Financial Emergency
without the counsel from the Cabinet.
d. State legislatures cannot make law on certain
matters without the concurrence of Union Legislature.

8 Consider the following statements: (2019) c Impeachment of Judges of Supreme Court:


1. The motion to impeach a Judge of the Supreme • Judges Enquiry Act (1968) regulates the procedure relating to
Court of India cannot be rejected by the Speaker of the removal of a judge of the Supreme Court by the process of
the Lok Sabha as per the Judges (Inquiry) Act, 1968. impeachment. So, 3 is correct.
2. The Constitution of India defines and gives details o A removal motion signed by 100 members (in the case of Lok
of what constitutes ‘incapacity and proved Sabha) or 50 members (in the case of Rajya Sabha) is to be given
misbehavior’ of the Judges of the Supreme Court of to the Speaker/Chairman.
India. o The Speaker/Chairman may admit the motion to impeach a
Judge of the Supreme Court of India. So, it can be rejected by
the Speaker. So, option 1 is wrong.
JOIN ME AT TELEGRAM – UPSC PRELIMS WITH NEELESH (AIR 442 UPSC CSE 2021) https://t.me/UPSCPrelimsWithNeelesh
FREE CSAT COURSE ON YOUTUBE – CIVIL SERVICES WITH NEELESH, Best PYQ Document for Prelims – www.neeleshair442.com,
Best test series on – WWW.CSETOPPER.COM ©Copyright Reserved with the Author
3. The details of the process of impeachment of the o If it is admitted, then the Speaker/Chairman is to constitute a
Judges of the Supreme Court of India are given in the three-member committee to investigate into the charges.
Judges (Inquiry) Act, 1968. o The committee should consist of (a) the chief justice or a
4. If the motion for the impeachment of a Judge is judge of the Supreme Court, (b) a chief justice of a high court,
taken up for voting, the law requires the motion to be and (c) a distinguished jurist.
backed by each House of the Parliament and o If the committee finds the judge to be guilty of misbehaviour
supported by a majority of total membership of that or suffering from an incapacity, the House can take up the
House and by not less than two-thirds of total consideration of the motion. This motion has to be passed by a
members of that House present and voting. special majority of each House of Parliament i.e. a majority of
Which of the statements given above is/are correct? the total membership of that House and a majority of not less
a. 1 and 2 only than two-third of the members of that House present and
b. 3 only voting. So, 4 is correct.
c. 3 and 4 only o After the motion is passed by each House of Parliament by
d. 1, 3 and 4 special majority, an address is presented to the president for
removal of the judge.
o Finally, the president passes an order removing the judge.

The constitution nowhere defines the term "incapacity and


proved misbehaviour". So, option 2 is wrong.

9 With reference to the Indian Judiciary, consider the a Under Article 128 of the Constitution, the Chief Justice of India
following statements: (2021) may, at any time, with the previous consent of the President,
1. Any retired judge of the Supreme Court of India can request any person who has held the office of a Judge of the
be called back to sit and act as Supreme Court judge Supreme Court to sit and act as a Judge of the Supreme Court.
by the Chief Justice of India with prior permission of So, statement 1 is correct.
the President of India.
2. A High Court in India has the power to review its Article 137 of the Constitution of India grants the Supreme
own judgement as the Supreme Court does. Court the power to review any of its judgments or orders. This
Which of the statements given above is/are correct?
JOIN ME AT TELEGRAM – UPSC PRELIMS WITH NEELESH (AIR 442 UPSC CSE 2021) https://t.me/UPSCPrelimsWithNeelesh
FREE CSAT COURSE ON YOUTUBE – CIVIL SERVICES WITH NEELESH, Best PYQ Document for Prelims – www.neeleshair442.com,
Best test series on – WWW.CSETOPPER.COM ©Copyright Reserved with the Author
a. 1 only is not available to High Court as per the constitution. So,
b. 2 only statement 2 is incorrect.
c. Both 1 and 2
d. Neither 1 nor 2

10 With reference to the writs issued by the Courts in c Mandamus literally means ‘We command’. This writ is issued
India, consider the following statements: (2022) when the court finds that a person holding a public office or a
1. Mandamus will not lie against a private public authority (including the Government) is not doing his
organization unless it is entrusted with a public duty. legal duty and thereby is infringing on the right of an individual.
2. Mandamus will not lie against a Company even Statement 2 is not correct.
though it may be a Government Company.
3. Any public minded person can be a petitioner to The writ of mandamus is also available against any public body,
move the Court to obtain the writ of Quo Warranto. a corporation(company) etc. for the same purpose.
Which of the statements given above are correct?
a. 1 and 2 only Statement 1 is correct. It may not be granted against the private
b. 2 and 3 only organization unless it is entrusted with a public duty.
c. 1 and 3 only
d. 1,2 and 3 Statement 3 is correct. Quo Warranto means “What is your
authority”. A writ of Quo Warranto is issued against the holder
of a public office to show to the court under what authority he
holds the office. Any public minded person can be a petitioner
to move the court in this case.
Additional Information you should know
1 Ad Hoc Judges are appointed in the Supreme Court when there is no quorum of Judges available to hold or continue any session of the court.
2 There is no ad hoc judge for High Court. This word is not used for high court judges
Your own remark finally
3 Judicial Review in India is based on PROCEDURE ESTABLISHED BY LAW (not due process of law, not rule of law, not precedents and convention)
4 The Guardianship of the Indian Constitution is vested in SUPREME COURT (and NOT PRESIDENT)
5 The Parliament can enlarge the Jurisdiction of the Supreme Court of India
JOIN ME AT TELEGRAM – UPSC PRELIMS WITH NEELESH (AIR 442 UPSC CSE 2021) https://t.me/UPSCPrelimsWithNeelesh
FREE CSAT COURSE ON YOUTUBE – CIVIL SERVICES WITH NEELESH, Best PYQ Document for Prelims – www.neeleshair442.com,
Best test series on – WWW.CSETOPPER.COM ©Copyright Reserved with the Author
TOPIC 20 Chapter 27 - Judicial Review, Chapter 28 – Judicial Activism, Chapter
29 – Public Interest Litigation
Total Number of Questions Asked from 2011-23 1
My Instructions related to these chapters These chapters have limited importance. Know their basics.

PREVIOUS YEAR QUESTIONS RELATED TO THESE CHAPTERS OF TOPICS


S.No. Question and Year Answer Explanation
1 In India, Judicial Review implies: (2017) a Judicial Review is the power of judiciary to examine the
a. The power of the Judiciary to pronounce upon the constitutionality of legislative enactments and executive orders
constitutionality of laws and executive orders. of both the Central and state governments. On examination, if
b. The power of the Judiciary to question the wisdom they are found to be violative of the Constitution (ultra-vires),
of the laws enacted by the Legislatures. they can be declared as illegal, unconstitutional and invalid (null
c. The power of the Judiciary to review all the and void). Consequently, they cannot be enforced by the
legislative enactments before they are assented by the government. So, statement a is correct.
President.
d. The power of the Judiciary to review its own
judgements given earlier in similar or different cases.

Additional Information you should know


1. The term Judicial Review has nowhere been used in the Constitution

TOPIC 21 Chapter 30 (GOVERNOR)


Total Number of Questions Asked from 2011-23 3
My Instructions related to these chapters This chapter is important. Read it fully

JOIN ME AT TELEGRAM – UPSC PRELIMS WITH NEELESH (AIR 442 UPSC CSE 2021) https://t.me/UPSCPrelimsWithNeelesh
FREE CSAT COURSE ON YOUTUBE – CIVIL SERVICES WITH NEELESH, Best PYQ Document for Prelims – www.neeleshair442.com,
Best test series on – WWW.CSETOPPER.COM ©Copyright Reserved with the Author
PREVIOUS YEAR QUESTIONS RELATED TO THESE CHAPTERS OF TOPICS
S.No. Question and Year Answer Explanation
1 Which one of the following statements is correct? c Article 153 of the Constitution requires that there shall be
(2013) Governor for every state. One person can be appointed as
a. In India, the same person cannot be appointed as Governor for two or more states. So, a is incorrect.
Governor for two or more States at the same time.
b. The Judges of the High Court of the States in India Judges in a High Court are appointed by the President of India
are appointed by the Governor of the State just as the and not the Governor. So, b is incorrect.
Judges of the Supreme Court are appointed by the
President. The constitution does not lay does not lay down any ground
c. No procedure has been laid down in the Constitution upon which a governor may be removed by the President. So, c
of India for the removal of a Governor from his/her is correct.
post.
d. In the case of a Union Territory having a legislative In the case of UT having a legislative setup, the Chief Minister is
setup, the Chief Minister is appointed by the Lt. appointed by the President and not Lt Governor on the basis of
Governor on the basis of majority support. majority support. So, d is incorrect.

2 Which of the following are the discretionary powers b Appointing the Ministers and making the rules to conduct the
given to the Governor of a State? (2014) business of the State Government are not the discretionary
1. Sending a report to the President of India for powers of Governor. They are executive powers. Rest are the
imposing the President’s rule. discretionary powers.
2. Appointing the Ministers. So, b is correct.
3. Reserving Certain Bills passed by the State
Legislature for the consideration of the President of Discretionary powers of the Governor are clearly mentioned on
India. page 30.10 (topic – Constitutional Position of Governor). Please
4. Making the rules to conduct the business of the go through it and remember it.
State Government.
Select the correct answer using the code given below:
JOIN ME AT TELEGRAM – UPSC PRELIMS WITH NEELESH (AIR 442 UPSC CSE 2021) https://t.me/UPSCPrelimsWithNeelesh
FREE CSAT COURSE ON YOUTUBE – CIVIL SERVICES WITH NEELESH, Best PYQ Document for Prelims – www.neeleshair442.com,
Best test series on – WWW.CSETOPPER.COM ©Copyright Reserved with the Author
a. 1 and 2 only
b. 1 and 3 only
c. 2,3 and 4 only
d. 1,2,3 and 4

3 Consider the following statements: c According to Article 361: No criminal proceedings whatsoever
1. No criminal proceedings shall be instituted against shall be instituted or continued against the President, or
the Governor of a State in any court during his term the Governor of a State, in any court during his
of office. term of office. So, 1 is correct.
2. The emoluments and allowances of the Governor
of a State shall not be diminished during his term of According to Article 158:
office. The Governor shall not be a member of either House of
Which of the statements given above is/are correct? Parliament or of a House of the Legislature of any State
a. 1 only specified in the First Schedule, and if a member of either House
b. 2 only of Parliament or of a House of the Legislature of any such State
c. Both 1 and 2 is appointed Governor, he shall be deemed to have vacated his
d. Neither 1 nor 2 seat in that House on the date on which he enters upon his
office as Governor.
o The Governor shall not hold any other office of profit.
o The Governor shall be entitled without payment of rent to
the use of his official residences and shall be also entitled to
such emoluments, allowances and privileges as may be
determined by Parliament by law and, until provision in that
behalf is so made, such emoluments, allowances and privileges
as are specified in the Second Schedule.
o The emoluments and allowances of the Governor shall not be
diminished during his term of office. So, 2 is correct.

Additional Information you should know


JOIN ME AT TELEGRAM – UPSC PRELIMS WITH NEELESH (AIR 442 UPSC CSE 2021) https://t.me/UPSCPrelimsWithNeelesh
FREE CSAT COURSE ON YOUTUBE – CIVIL SERVICES WITH NEELESH, Best PYQ Document for Prelims – www.neeleshair442.com,
Best test series on – WWW.CSETOPPER.COM ©Copyright Reserved with the Author
1. No procedure has been laid down in the Constitution of India for the removal of a Governor from his/her post.
2. When the same person is appointed as Governor of two or more states, the emolument and allowances payable to the Governor shall
be allocated among the states is such proportion as the President may by order determine. (Inference – so allocation may not be equal
for two states)

TOPIC 22 Chapter 31 - Chief Minister, Chapter 32 – State Council of Ministers


Total Number of Questions Asked from 2011-23 1
My Instructions related to these chapters These chapters are also important. Prepare it in detail

PREVIOUS YEAR QUESTIONS RELATED TO THESE CHAPTERS OF TOPICS


S.No. Question and Year Answer Explanation
1 Consider the following statements: (2013) b Article 75(3) of the Constitution states that: The Council of
1. The Council of Ministers in the Centre shall be Ministers shall be collectively responsible to the House of the
collectively responsible to the Parliament. People (Lok Sabha). So, 1 is incorrect.
2. The Union Ministers shall hold the office during the
pleasure of the President of India. Article 75(2) The Ministers shall hold office during the pleasure
3. The Prime Minister shall communicate to the of the President. So, 2 is correct
President about the proposals for legislation.
Which of the statements given above is/are correct? It is the duty of the Prime Minister to communicate to the
a. 1 only president all decisions of councils of Minister relating to the
b. 2 and 3 only administration of the affairs of union and proposal for
c. 1 and 3 only legislation. So, 3 is correct.
d. 1,2 and 3

JOIN ME AT TELEGRAM – UPSC PRELIMS WITH NEELESH (AIR 442 UPSC CSE 2021) https://t.me/UPSCPrelimsWithNeelesh
FREE CSAT COURSE ON YOUTUBE – CIVIL SERVICES WITH NEELESH, Best PYQ Document for Prelims – www.neeleshair442.com,
Best test series on – WWW.CSETOPPER.COM ©Copyright Reserved with the Author
Additional Information you should know
1. As per article 192, if any question arises as to whether a member of a House of the Legislature of a State has become subject to any of the
disqualification mentioned in article 191, the question shall be referred for the decision of the GOVERNOR and his decision shall be final.
Before giving any decision on any such question, the Governor shall obtain the opinion of the Election Commission and shall act according to
such opinion

TOPIC 23 Chapter 33 (STATE LEGISLATURE)


Total Number of Questions Asked from 2011-23 2
My Instructions related to these chapters This chapter is also important. Read it full

PREVIOUS YEAR QUESTIONS RELATED TO THESE CHAPTERS OF TOPICS


S.No. Question and Year Answer Explanation
1 Consider the following statement: (2015) d State Legislative Council
1.The Legislative Council of a State in India can be • The maximum strength of the council is fixed at one-third of
larger in size than half of the Legislative Assembly of the total strength of the assembly and the minimum strength is
that particular State. fixed at 40 (with some exceptions). So, 1 is incorrect.
2. The Governor of a State nominates the Chairman of
Legislative Council of that particular State. • The Chairman of the Legislative Council is elected by the
council itself from amongst its members. So, 2 is incorrect.
Which of the statements given above is/ are correct?
a. 1 only
b. 2 only
c. Both 1 and 2
d. Neither 1 nor 2

JOIN ME AT TELEGRAM – UPSC PRELIMS WITH NEELESH (AIR 442 UPSC CSE 2021) https://t.me/UPSCPrelimsWithNeelesh
FREE CSAT COURSE ON YOUTUBE – CIVIL SERVICES WITH NEELESH, Best PYQ Document for Prelims – www.neeleshair442.com,
Best test series on – WWW.CSETOPPER.COM ©Copyright Reserved with the Author
2 With reference to the Legislative Assembly of a State c Article 176(1) of the Indian Constitution enjoins that the
in India, consider the following statements: (2019) Governor shall address both the Houses assembled together at
1. The Governor makes a customary address to the commencement of the first Session after each general
members of the house at the commencement of the election to the Assembly and at the commencement of the first
first session of the year. session of each year and inform the Legislature of the causes of
2. When a State Legislature does not have a rule on a its Summons. So, statement 1 is correct.
particular matter, it follows the Lok Sabha rule on that
matter. According to Article 208 of the Indian Constitution, if there is no
Which of the statements given above is/are correct? rule on a particular matter then the rules prevalent immediately
(a) 1 only before the commencement of the Constitution in the
(b) 2 only corresponding province shall be applicable to the State
(c) Both 1 and 2 Legislature. However, the Speaker of the Legislative Assembly
(d) Neither 1 nor 2 can make modifications to such rules. The State Assembly 'may'
follow the rules of the Lok Sabha, if decided by the Speaker of
the Assembly. So, 2 is correct

TOPIC 24 Chapter 34 (HIGH COURT)


Total Number of Questions Asked from 2011-23 1
My Instructions related to these chapters This chapter is also important. Read it full

PREVIOUS YEAR QUESTIONS RELATED TO THESE CHAPTERS OF TOPICS


S.No. Question and Year Answer Explanation
1 With reference to the Constitution of India, consider d Both the SC and HCs have the jurisdiction to question the
the following Statements: (2019) validity of a constitutional amendment or a central law. So, 1 is
1. No High Court shall have the jurisdiction to declare wrong.
any central law to be constitutionally invalid.
2. An amendment to the Constitution of India cannot As the Supreme Court is the guardian of the Constitution and
be called into question by the Supreme Court of India. so any constitutional amendment that is against the basic
JOIN ME AT TELEGRAM – UPSC PRELIMS WITH NEELESH (AIR 442 UPSC CSE 2021) https://t.me/UPSCPrelimsWithNeelesh
FREE CSAT COURSE ON YOUTUBE – CIVIL SERVICES WITH NEELESH, Best PYQ Document for Prelims – www.neeleshair442.com,
Best test series on – WWW.CSETOPPER.COM ©Copyright Reserved with the Author
Which of the statements given above is/are correct? structure of the constitution which was derived in
a. 1 only Keshavananda Bharati case, 1973 can be nullified by the
b. 2 only Supreme Court. So, 2 is wrong.
c. Both 1 and 2
d. Neither 1 nor 2

Additional Information you should know


1. The pension of a retired High Court Judge is charged on the Consolidated Fund of India and (not the consolidated fund of the state)
2. When the Chief Justice of a High Court acts in an administrative capacity, he is subjected to – the writ jurisdiction of any of the other
judges of the High Court

TOPIC 25 Chapter 35 -Tribunals, Chapter 36 – Subordinate Courts


Total Number of Questions Asked from 2011-23 3
My Instructions related to these chapters This chapter is important. Focus more on
1 Administrative Tribunal (CAT – COMPOSITION etc)
2 National Legal Services Authority
3 Lok Adalats
4 Gram Nyayalayas

PREVIOUS YEAR QUESTIONS RELATED TO THESE CHAPTERS OF TOPICS


S.No. Question and Year Answer Explanation
1 With reference to the National Legal Services c The National Legal Services Authority (NALSA) has been
Authority, consider the following statements: (2013) constituted under the Legal Services Authorities Act, 1987 to
1. Its objective is to provide free and competent legal provide free Legal Services to the weaker sections of the society
services to the weaker sections of the society on the and to organize Lok Adalats for amicable settlement of disputes.
basis of equal opportunity. So, 1 is correct.

JOIN ME AT TELEGRAM – UPSC PRELIMS WITH NEELESH (AIR 442 UPSC CSE 2021) https://t.me/UPSCPrelimsWithNeelesh
FREE CSAT COURSE ON YOUTUBE – CIVIL SERVICES WITH NEELESH, Best PYQ Document for Prelims – www.neeleshair442.com,
Best test series on – WWW.CSETOPPER.COM ©Copyright Reserved with the Author
2. It issues guidelines for the State Legal Services In every state, State Legal Services Authority has been
Authorities to implement the legal programmes and constituted to give effect to the policies and directions of the
schemes throughout the country. NALSA and to give free legal services to the people and conduct
Which of the given statements above is/are correct? Lok Adalats in the State. So, 2 is correct.
a. 1 only
b. 2 only
c. Both 1 and 2
d. Neither 1 nor 2

2 In India, Legal Services Authority provide free legal a The National Legal Service Authority (NALSA) has been
services to which of the following type of citizens? constituted under the Legal Service Authorities Act, 1987 to
(2020) provide free legal service to the weaker sections of the society
1. Person with annual income of less than Rs. to organize Lok Adalats for amicable settlement of disputes.
1,00,000.
2. Transgender with an annual income of less than Rs. The sections of the society as enlisted under Section 12 of the
2,00,000. Legal Services Authorities Act are entitled to free legal services
3. Member of Other Backward Classes (OBC) with an including a member of SC or ST (and not OBC), a victim of
annual income of less than Rs.3,00,000. trafficking in human beings or beggar, a women or a child; a
4. All Senior Citizens. mentally ill or otherwise disabled person; an industrial
Select the correct answer using the code given below: workman or a person in receipt of annual income less than the
a. 1 and 2 only amount as may be prescribed by the State Governments etc.
b. 3 and 4 only
c. 2 and 3 only The provision of free legal services to Senior Citizens is
d. 1 and 4 only dependent on the rules framed by the respective state
governments who can prescribe income limits. Therefore, not
all senior citizens are eligible for free legal services.

JOIN ME AT TELEGRAM – UPSC PRELIMS WITH NEELESH (AIR 442 UPSC CSE 2021) https://t.me/UPSCPrelimsWithNeelesh
FREE CSAT COURSE ON YOUTUBE – CIVIL SERVICES WITH NEELESH, Best PYQ Document for Prelims – www.neeleshair442.com,
Best test series on – WWW.CSETOPPER.COM ©Copyright Reserved with the Author
Further, there is a provision of free legal aid to transgenders
having an annual income of upto Rs 2 lakhs in Delhi, but it is yet
to be implemented all over India.
So, only 1 and 2 are correct.

3 With reference to the ‘Gram Nyayalaya Act’, which of b Gram Nyayalayas Act, 2008 is an Act of Parliament of
the following statements is/are correct? (2016) India enacted for establishment of Gram Nyayalayas or village
1. As per the Act, Gram Nyayalayas can hear only civil courts for speedy and easy access to justice system in the rural
cases and not criminal cases. areas of India. The Act came into force from 2 October
2. The Act allows local social activists as mediators/ 2009. However, the Act has not been enforced properly, with
reconciliators. only 256 functional Gram Nyayalayas in the country (as on 17
Select the correct answer using the code given below: December 2021) against a target of 5000 such courts. The major
a. 1 only reasons behind the non-enforcement includes financial
b. 2 only constraints, reluctance of lawyers, police and other
c. Both 1 and 2 government officials.
d. Neither 1 nor 2
Features:
Gram Nyayalaya are established generally at headquarter of
every Panchayat at intermediate level or a group of contiguous
panchayat in a district where there is no panchayat at
intermediate level.

• The Gram Nyayalayas are presided over by a Nyayadhikari,


who will have the same power, enjoy same salary and benefits
of a Judicial Magistrate of First Class. Such Nyayadhikari are to
be appointed by the State Government in consultation with the
respective High Court.
• A Gram Nyayalaya have jurisdiction over an area specified by
a notification by the State Government in consultation with the
JOIN ME AT TELEGRAM – UPSC PRELIMS WITH NEELESH (AIR 442 UPSC CSE 2021) https://t.me/UPSCPrelimsWithNeelesh
FREE CSAT COURSE ON YOUTUBE – CIVIL SERVICES WITH NEELESH, Best PYQ Document for Prelims – www.neeleshair442.com,
Best test series on – WWW.CSETOPPER.COM ©Copyright Reserved with the Author
respective High Court. The Court can function as a mobile court
at any place within the jurisdiction of such Gram Nyayalaya,
after giving wide publicity to that regards.
• The Gram Nyayalayas have both civil and criminal jurisdiction
over the offences and nature of suits specified in the First and
Second schedules of the Act. The pecuniary jurisdiction of the
Nyayalayas are fixed by the respective High Courts. So, 1 is
correct.
• The fees charged in civil suits shall not exceed Rs.100
irrespective of the value of property in dispute.
• Both the Central and the State Government can add or
remove items in the Schedule. While the Central Government
can amend the list in Schedule I and II, by notifying them and
thereafter laying it in the Parliament, the State Government can
amend the items in Part III of Schedule I or II, in the areas of law
which the state is competent to enact law after due
consultation with the respective High Court and notifying it.
Such notification has to be laid in the State Legislature.
• Offences are to be tried in a summary manner in accordance
with Chapter XXI of Code of Criminal Procedure.
• The Act allows plea bargaining in accordance with Chapter
XXIA of Code of Criminal Procedure.
• Gram Nyayalayas can follow special procedures in civil
matters, in a manner it deems just and reasonable in the
interest of justice.
• Civil suits are proceeded on a day-to-day basis, with limited
adjournments and are to be disposed of within a period of six
months from the date of institution of the suit.

JOIN ME AT TELEGRAM – UPSC PRELIMS WITH NEELESH (AIR 442 UPSC CSE 2021) https://t.me/UPSCPrelimsWithNeelesh
FREE CSAT COURSE ON YOUTUBE – CIVIL SERVICES WITH NEELESH, Best PYQ Document for Prelims – www.neeleshair442.com,
Best test series on – WWW.CSETOPPER.COM ©Copyright Reserved with the Author
• In execution of a decree, the Court can allow special
procedures following rules of natural justice.
• Gram Nyayalayas allow for conciliation of the dispute and
settlement of the same in the first instance.
• Gram Nyayalayas has been given power to accept certain
evidences which would otherwise not be acceptable under
Indian Evidence Act.
• Appeals in criminal matter can be made to the Sessions Court
in the respective jurisdiction and in civil matters to the District
Court within a period of one month from the date of judgment.

The act allows local social activist as mediators /reconciliators.


So, 2 is correct.

TOPIC 26 Chapter 37 (SPECIAL PROVISION FOR SOME STATES)


Total Number of Questions Asked from 2011-23 0
My Instructions related to these chapters Remember the article and related state from this chapter ARTICLE
371 – 371J

TOPIC 27 Chapter 38 (PANCHAYATI RAJ)


Total Number of Questions Asked from 2011-23 6
My Instructions related to these chapters Very important chapter. Read it full. Remember the sequence of the
committees.
Also, you must prepare the articles related to the Panchayats at a
Glance (Table 38.2)
JOIN ME AT TELEGRAM – UPSC PRELIMS WITH NEELESH (AIR 442 UPSC CSE 2021) https://t.me/UPSCPrelimsWithNeelesh
FREE CSAT COURSE ON YOUTUBE – CIVIL SERVICES WITH NEELESH, Best PYQ Document for Prelims – www.neeleshair442.com,
Best test series on – WWW.CSETOPPER.COM ©Copyright Reserved with the Author
PREVIOUS YEAR QUESTIONS RELATED TO THESE CHAPTERS OF TOPICS
S.No. Question and Year Answer Explanation
1 The fundamental object of Panchayati Raj system is to c Panchayati Raj System is an institution of administration which
ensure which among the following? (2015) seeks to involve the people at the grass-root level in planning
1. People’s participation in development. and administration. People's participation at the local level in
2. Political accountability. both planning and implementation is essential for the success
3. Democratic decentralization. of decentralization and democracy. So, 1 is correct.
4. Financial mobilization. The basic objective of Panchayati Raj is to evolve a system of
Select the correct answer using the code given below. democratic decentralization and devolution with a view to
a. 1,2 and 3 only ensuring rapid socio-economic progress So, 3 is correct.
b. 2 and 4 only
c. 1 and 3 only Rest options 2 and 4 are not the fundamental objective of
d. 1,2,3 and 4 Panchayati Raj System.

2 Local self-government can be best explained as an b The political ideal that exists in democratic decentralization is
exercise in: (2017) reflected in the local self-government. So, b is correct.
a. Federalism
b. Democratic decentralization
c. Administrative delegation
d. Direct democracy

3 The Constitution (Seventy Third Amendment) c The 73rd Constitutional Amendment made radical provisions
Act,1992, which aims at promoting the Panchayati Raj with respect to revitalizing Panchayati Raj Institutions. It added
Institutions in the country, provides for which of the various provisions in Articles 243 to 243 (0) in part IX of the
following? (2011) Constitution. The Act also added Eleventh Schedule in the
1. Constitution of District Planning Committees. Constitution, which includes 29 subjects assigned to
2. State Election Commissions to conduct all Panchayats.
panchayat elections.
JOIN ME AT TELEGRAM – UPSC PRELIMS WITH NEELESH (AIR 442 UPSC CSE 2021) https://t.me/UPSCPrelimsWithNeelesh
FREE CSAT COURSE ON YOUTUBE – CIVIL SERVICES WITH NEELESH, Best PYQ Document for Prelims – www.neeleshair442.com,
Best test series on – WWW.CSETOPPER.COM ©Copyright Reserved with the Author
3. Establishment of State Finance Commissions. In order to ensure free, timely and fair elections of Panchayats
Select the correct answer using the codes given at all levels, the Act provides for the establishment of a separate
below: Election Commission in each State, which shall be independent
a. 1 only of the control of the state government. So, 2 is correct.
b. 1 and 2 only
c. 2 and 3 only In order to ensure the availability of adequate financial
d. 1,2 and 3 resources to the Panchayats, the Act makes the provision for
the establishment of State Finance Commission in each State.
The members of the Commission shall be appointed by the
Governor of the state concerned. The Commission shall
recommend the principles which shall govern the distribution
of proceeds of taxes and duties between the States and the
Panchayats and grant-in-aid to the Panchayats from the
Consolidated Fund of the State. So, 3 is correct.

District Planning Committee (DPC) was inserted by the 74th


Amendment Act and is the committee created as per article
243ZD of the Constitution of India at the district level for
planning at the district and below. The Committee in each
district should consolidate the plans prepared by the
Panchayats and the Municipalities in the district and prepare a
draft development plan for the district. So, 1 is incorrect.

4 Consider the following statements: (2016) b The minimum age for contesting elections to panchayats is 21
1. The minimum age prescribed for any person to be years. So, 1 is wrong.
a member of Panchayat is 25 years.
2. A Panchayat reconstituted after premature In case of dissolution of Panchayats before expiry of its term, it
dissolution continues only for the remainder period. is mandatory to hold elections within 6 months of its
Which of the statements given above is/are correct? dissolution. Every Panchayat shall continue for full term of 5
JOIN ME AT TELEGRAM – UPSC PRELIMS WITH NEELESH (AIR 442 UPSC CSE 2021) https://t.me/UPSCPrelimsWithNeelesh
FREE CSAT COURSE ON YOUTUBE – CIVIL SERVICES WITH NEELESH, Best PYQ Document for Prelims – www.neeleshair442.com,
Best test series on – WWW.CSETOPPER.COM ©Copyright Reserved with the Author
a. 1 only years. However, a Panchayat reconstituted after premature
b. 2 only dissolution shall continue for only the reminder of the period.
c. Both 1 and 2 If the period is less than 6 months, then it is not mandatory to
d. Neither 1 nor 2 hold the elections. So, 2 is correct.

5 The Government enacted the Panchayat Extension to c To create autonomous regions in tribal areas is not the
Scheduled Areas (PESA) Act in 1996. Which one of the Objectives of Panchayat Extension to Scheduled Areas (PESA)
following is not identified as its objective? Act. So, c is incorrect
a. To provide self- governance.
b. To recognize traditional rights. Objectives of Panchayat Extension to Scheduled Areas (PESA)
c. To create autonomous regions in tribal areas. Act:
d. To free tribal people from exploitation. - To extend the provisions of Part IX of the Constitution to
Scheduled Areas, identified by the Fifth Schedule of the
Constitution of India.
- To guarantee self-governance through traditional Gram Sabhas
for people living in the Scheduled Areas of India.
- To provide tribals control and rights over the natural resources
and conserves their identity and culture.

The act brought a positive development in the lives of tribal


communities who have earlier suffered tremendously.
So, rest are correct.

6 In the areas covered under the Panchayat (Extension b In 1996, Panchayat Extension to Schedule Areas (PESA) Act was
to the Scheduled Areas) Act,1996, what is the passed extending the provisions of the Panchayat System to
role/power of the Gram Sabha? (2012) Fifth Schedule Areas.
1. Gram Sabha has the power to prevent alienation of
land in the Scheduled Areas. Under section 4(i) of the Act, Gram Sabha or the Panchayats at
the appropriate level shall be consulted before making the
JOIN ME AT TELEGRAM – UPSC PRELIMS WITH NEELESH (AIR 442 UPSC CSE 2021) https://t.me/UPSCPrelimsWithNeelesh
FREE CSAT COURSE ON YOUTUBE – CIVIL SERVICES WITH NEELESH, Best PYQ Document for Prelims – www.neeleshair442.com,
Best test series on – WWW.CSETOPPER.COM ©Copyright Reserved with the Author
2. Gram Sabha has the ownership of minor forest acquisition of land in the Scheduled Areas for development
produce. projects and before resettling or rehabilitating persons affected
3. Recommendation of Gram Sabha is required for by such projects in the Scheduled Areas. So, statement 1 is
granting prospecting license or mining lease for any correct.
mineral in the Scheduled Ares.
Which of the following statements given above is/are Under section 4(m)(2) of the Act, Gram Sabha is endowed
correct? specifically with the ownership of minor forest produce. So,
a. 1 only statement 2 is correct
b. 1 and 2 only
c. 2 and 3 only Under section 4(k) of the Act, the recommendations of the
d. 1,2 and 3 Gram Sabha or the Panchayats at the appropriate level shall be
made mandatory prior to grant of prospecting license or mining
lease for minor minerals (not all minerals) in the Scheduled
Areas. So, statement 3 is wrong

Additional Information you should know


1. Legislature of the state is authorized to make provisions with respect to the composition of Panchayats as per article 243(c) (AND NOT
PRESIDENT, NOT PARLIAMENT, NOT GOVERNOR)
.

TOPIC 28 Chapter 39 (MUNICIPALITIES)


Total Number of Questions Asked from 2011-23 1
My Instructions related to these chapters This chapter has limited importance. Have a brief reading while
recalling the main provisions.
You must read the Table 39.3 (articles related to municipalities at a
glance)

JOIN ME AT TELEGRAM – UPSC PRELIMS WITH NEELESH (AIR 442 UPSC CSE 2021) https://t.me/UPSCPrelimsWithNeelesh
FREE CSAT COURSE ON YOUTUBE – CIVIL SERVICES WITH NEELESH, Best PYQ Document for Prelims – www.neeleshair442.com,
Best test series on – WWW.CSETOPPER.COM ©Copyright Reserved with the Author
PREVIOUS YEAR QUESTIONS RELATED TO THESE CHAPTERS OF TOPICS
S.No. Question and Year Answer Explanation
1 Consider the following statements: (2011) a As per the Constitution 74th Amendment Act, 1992, there is a
In India, a Metropolitan Planning Committee mandatory provision for constitution of Metropolitan Planning
1. Is constituted under the provisions of the Committee in all metropolitan areas by State governments. So,
Constitution of India. 1 is correct.
2. Prepares the draft development plans for
metropolitan area. As per Article 243ZE, there shall be constituted in every
3. Has the sole responsibility for implementing Metropolitan area a Metropolitan Planning Committee to
Government sponsored schemes in the metropolitan prepare a draft development plan for the Metropolitan area as
area. a whole. However, it is not the sole authority for implementing
Which of the Statements given above is/ are correct? government sponsored schemes in the metropolitan area. For
a. 1 and 2 only example, in Delhi, there are various other agencies as well for
b. 2 only implementing government sponsored schemes such as New
c. 1 and 3 only Delhi Municipal Corporation, Delhi Development Authority. So,
d. 1,2 and 3 2 is incorrect.

TOPIC 29 Chapter 40 (UNION TERRITORIES)


Total Number of Questions Asked from 2011-23 0
My Instructions related to these chapters Know the special provisions for Delhi. And Lt Governor vs
Administrator (who for which UT). Also see the comparison of
states and UTs

JOIN ME AT TELEGRAM – UPSC PRELIMS WITH NEELESH (AIR 442 UPSC CSE 2021) https://t.me/UPSCPrelimsWithNeelesh
FREE CSAT COURSE ON YOUTUBE – CIVIL SERVICES WITH NEELESH, Best PYQ Document for Prelims – www.neeleshair442.com,
Best test series on – WWW.CSETOPPER.COM ©Copyright Reserved with the Author
TOPIC 30 Chapter 41 (SCHEDULED AND TRIBAL AREAS)
Total Number of Questions Asked from 2011-23 1
My Instructions related to these chapters Know the basics and read this chapter. This chapter is gaining
importance now. So, you must not ignore it.

PREVIOUS YEAR QUESTIONS RELATED TO THESE CHAPTERS OF TOPICS


S.No. Question and Year Answer Explanation
1 With reference to 'Scheduled Areas' in India, b Statement 1 is correct: As per the Indian Constitution (Part C of
consider the following statements: (2023) the Fifth Schedule), the expression Scheduled Areas means
1. Within a State, the notification of an area as such areas as the President may by order declare to be
Scheduled Area takes place through an Order of the Scheduled Areas.
President.
2. The largest administrative unit forming the Statement 2 is correct: The largest administrative unit forming
Scheduled Area is the District and the lowest is the the scheduled areas has been the district and the lowest the
cluster of villages in the Block. cluster of villages in the block.
3. The Chief Ministers of the concerned States are Statement 3 is not correct: The governor of the state has a
required to submit annual reports to the Union special responsibility regarding such scheduled areas. He has to
Home Ministry on the administration of Scheduled submit a report to the president regarding the administration
Areas in the States. of such areas, annually or whenever so required by the
How many of the above statements are correct? president. Thus, only two statements are correct. Hence option
(a) Only one (b) is the correct answer.
(b) Only two
(c) All three
(d) None

JOIN ME AT TELEGRAM – UPSC PRELIMS WITH NEELESH (AIR 442 UPSC CSE 2021) https://t.me/UPSCPrelimsWithNeelesh
FREE CSAT COURSE ON YOUTUBE – CIVIL SERVICES WITH NEELESH, Best PYQ Document for Prelims – www.neeleshair442.com,
Best test series on – WWW.CSETOPPER.COM ©Copyright Reserved with the Author
TOPIC 31 CONSTITUTIONAL BODIES (Chapter 42 – Election Commission,
Chapter 43 – Union Public Service Commission, Chapter 44 – State
Public Service Commission, Chapter 45 – Finance Commission,
Chapter 46 – GST, Chapter 47 – NCSC, Chapter 48 – NCST, Chapter 49
– NCBC, Chapter 50 – Special Officer for Linguistic Minorities, Chapter
51 – CAG, Chapter 52 – Attorney General of India, Chapter 53 –
Advocate General of the State)
Total Number of Questions Asked from 2011-23 5
My Instructions related to these chapters This Section is particularly important. While reading, focus more on
composition, independence, removal and functions.

More important Constitutional bodies are – Election Commission,


UPSC, Finance Commission, GST, CAG, Attorney General.

Also, you must know what are the constitutional bodies and what are
the non-constitutional bodies.

PREVIOUS YEAR QUESTIONS RELATED TO THESE CHAPTERS OF TOPICS


S.No. Question and Year Answer Explanation
1 In India, other than ensuring that public funds are c CAG derives his authority and functions mainly from the
used efficiently and for intended purpose, what is the provisions of Article 149 to 151 of the Constitution of India and
importance of the office of the Comptroller and Comptroller and Auditor General’s (Duties, Powers and
Auditor General (CAG)? (2012) Conditions of Service (DPC)) Act in 1971.
1. CAG exercises exchequer control on behalf of the CAG does not exercise exchequer control under any
Parliament when the President of India declares circumstances including emergency. So, statement 1 is wrong.
national emergency/ financial emergency.
CAG does not have judicial powers for the prosecution of
violation of law. So, statement 4 is wrong.
JOIN ME AT TELEGRAM – UPSC PRELIMS WITH NEELESH (AIR 442 UPSC CSE 2021) https://t.me/UPSCPrelimsWithNeelesh
FREE CSAT COURSE ON YOUTUBE – CIVIL SERVICES WITH NEELESH, Best PYQ Document for Prelims – www.neeleshair442.com,
Best test series on – WWW.CSETOPPER.COM ©Copyright Reserved with the Author
2. CAG reports on the execution of projects or
programmes by the ministries are discussed by the Rest statements are correct.
Public Accounts Committee.
3. Information from CAG reports can be used by
investigating agencies to press charges against those
who have violated the law while managing public
finances.
4. While dealing with the audit and accounting of
government companies, CAG has certain judicial
powers for prosecuting those who violate the law.
Which of the statements given above is/are correct?
a. 1,3 and 4 only
b. 2 only
c. 2 and 3 only
d. 1,2,3 and 4

2 Consider the following statements: c Attorney General of India does not have the right to vote in Lok
Attorney General of India can: (2013) Sabha since he is not the member of the Parliament. Only the
1. Take Part in the proceeding of the Lok Sabha. members can vote. Hence, 4 is incorrect.
2. Be a member of a committee of the Lok Sabha.
3. Speak in the Lok Sabha. Rest all statements are correct.
4. Vote in the Lok Sabha.
Which of the statements given above is/are correct?
a. 1 only
b. 2 and 4 only
c. 1,2 and 3 only
d. 1 and 3 only

JOIN ME AT TELEGRAM – UPSC PRELIMS WITH NEELESH (AIR 442 UPSC CSE 2021) https://t.me/UPSCPrelimsWithNeelesh
FREE CSAT COURSE ON YOUTUBE – CIVIL SERVICES WITH NEELESH, Best PYQ Document for Prelims – www.neeleshair442.com,
Best test series on – WWW.CSETOPPER.COM ©Copyright Reserved with the Author
3 Consider the following statements: (2022) d Statement 1 is not correct. According to the Article 88 of the
1. Attorney General of India and the Solicitor General constitution of India, the Attorney General of India has the right
of India are the only officers of the Government who to speak and take part in the proceedings of either House of the
are allowed to participate in the meetings of the Parliament without having the right to vote. However, the
Parliament of India. Solicitor General of India doesn’t enjoy such right.
2. According to the Constitution of India, the Attorney
general of India submits his resignation when the Statement 2 is not correct. According to Article 76(4), the
Government which appointed him resigns. Attorney General shall hold office during the pleasure of the
Which of the statements given above is/are correct? President. The term of office of the Attorney-General is not
a. 1 only fixed under the Constitution of India. Moreover, the
b. 2 only Constitution doesn’t contain any provisions for procedure and
c. Both 1 and 2 grounds for his removal. However, by convention, he submits
d. Neither 1 nor 2 his resignation when the Government which appointed him
resigns.

4 With reference to the Finance Commission of India, d The Finance Commission is constituted by the President under
which of the following Statements is correct? (2011) article 280 of the Constitution. Its main function is to give
a. It encourages the inflow of foreign capital for recommendations on distribution of tax revenues between the
infrastructure development. Union and the States amongst the States themselves. Refer to
b. It facilitates the proper distribution of finances its functioning from Laxmikanth.
among the Public Sector Undertakings.
c. It ensures transparency in financial administration. Among the given options, none are true since they do not
d. None of the statements (a), (b) and (c) given above come within the ambit of Finance Commission
is correct in this context.

5 Consider the following organizations/bodies in India: a The National Commission for Backward Classes is a
(2023) constitutional body. It was established through a Constitution
1. The National Commission for Backward Classes (Amendment) Act, 2018 (also called, 102nd Amendment Act,
2. The National Human Rights Commission 2018). This amendment act in the constitution to make it a
JOIN ME AT TELEGRAM – UPSC PRELIMS WITH NEELESH (AIR 442 UPSC CSE 2021) https://t.me/UPSCPrelimsWithNeelesh
FREE CSAT COURSE ON YOUTUBE – CIVIL SERVICES WITH NEELESH, Best PYQ Document for Prelims – www.neeleshair442.com,
Best test series on – WWW.CSETOPPER.COM ©Copyright Reserved with the Author
3. The National Law Commission constitutional body under Article 338B of the Indian
4. The National Consumer Disputes Redressal Constitution. Hence option 1 is correct.
Commission
How many of the above are constitutional bodies? The National Human Rights Commission: It is a statutory body
(a) Only one established under the Protection of Human Rights Act, 1993.
(b) Only two Hence option 2 is not correct.
(c) Only three
(d) All four National Law Commission: The Law Commission of India is an
executive body whose chief function is legal reforms in the
country. Hence option 3 is not correct.

The National Consumer Disputes Redressal Commission: The


National Consumer Disputes Redressal Commission was
established in 1988 under the Consumer Protection Act of
1986. It is a quasi-judicial body and statutory in nature. Hence
option 4 is not correct.

TOPIC 32 NON-CONSTITUTIONAL BODIES (Chapter 54 - Niti Aayog, Chapter


55 – NHRC, Chapter 56 – SHRC, Chapter 57 – CIC, Chapter 58 – SIC,
Chapter 59 – CVC, Chapter 60 – CBI, Chapter 61 – Lokpal and
Lokayukt, Chapter 62 – NIA, Chapter 63 - NDMA)

Total Number of Questions Asked from 2011-23 3


My Instructions related to these chapters Know what are the non-constitutional bodies.
Give special focus to
a) Niti Aayog
b) NHRC

JOIN ME AT TELEGRAM – UPSC PRELIMS WITH NEELESH (AIR 442 UPSC CSE 2021) https://t.me/UPSCPrelimsWithNeelesh
FREE CSAT COURSE ON YOUTUBE – CIVIL SERVICES WITH NEELESH, Best PYQ Document for Prelims – www.neeleshair442.com,
Best test series on – WWW.CSETOPPER.COM ©Copyright Reserved with the Author
PREVIOUS YEAR QUESTIONS RELATED TO THESE CHAPTERS OF TOPICS
S.No. Question and Year Answer Explanation
1 Who among the following constitute the National b National Development Council:
Development Council? (2013) The National Development Council (NDC) or Rashtriya Vikas
1. The Prime Minister Parishad is the apex body for decision creating and
2. The Chairman, Finance Commission. deliberations on development matters in India, presided over
3. Ministers of the Union Cabinet. by the Prime Minister. It was set up on 6 August 1952 to
4. Chief Ministers of the States strengthen and mobilize the effort and resources of the nation
Select the correct answer using the codes given below: in support of the Five Year Plans made by Planning Commission,
a. 1,2 and 3 only to promote common economic policies in all vital spheres, and
b. 1,3 and 4 only to ensure the balanced and rapid development of all parts of
c. 2 and 4 only the country. The Council comprises the Prime Minister,
d. 1,2,3 and 4 the Union Cabinet Ministers, Chief Ministers of all States or
their substitutes, representatives of the Union Territories and
the members of the NITI Aayog (erstwhile Planning
Commission).
NDC (National Development Council) has been proposed to be
abolished. But till date no resolution has been passed to abolish
it. Since the inception of NITI Aayog's Governing Council (which
has almost the same composition and roles as NDC), the NDC
has had no work assigned to it nor did it have any meetings.
So, only 1, 3 and 4 are correct and hence the b option.

2 Which of the following bodies does not/do not find d National Development council is not a constitutional body. It is
mention in the Constitution? (2013) an extra-constitutional body. So, 1 is incorrect.
1. National Development Council.

JOIN ME AT TELEGRAM – UPSC PRELIMS WITH NEELESH (AIR 442 UPSC CSE 2021) https://t.me/UPSCPrelimsWithNeelesh
FREE CSAT COURSE ON YOUTUBE – CIVIL SERVICES WITH NEELESH, Best PYQ Document for Prelims – www.neeleshair442.com,
Best test series on – WWW.CSETOPPER.COM ©Copyright Reserved with the Author
2. Planning Commission. Planning Commission was a non-constitutional and non-
3. Zonal Councils. statutory body. It was created by the Govt. of India in 1950 by
Select the correct answer using the codes given a resolution. So, 2 is incorrect.
below:
a. 1 and 2 only Zonal councils are statutory bodies and not the constitutional
b. 2 only bodies. So, 3 is incorrect.
c. 1 and 3 only
d. 1,2 and 3 Instruction - Remember the names of constitutional bodies
given in Laxmikanth.

3 Consider the following statements: (2013) b NDC is listed as an advisory body to the Planning commission.
1. National Development Council is an organ of the Moreover, the plans made by the Planning commission are
Planning Commission. placed before NDC for its acceptance. Planning Commission is
2. The Economic and Social Planning is kept in the abolished now and NDC is non-functional with the creation of
Concurrent List in the Constitution of India. NITI Aayog. The Economic and Social Planning is kept in the
3. The Constitution of India prescribes that Concurrent List in the Constitution of India.The Constitution of
Panchayats should be assigned the task of India prescribes that Panchayats should be assigned the task of
preparation of plans for economic development and preparation of plans for economic development and social
social justice. justice. So, only 2 and 3 are correct.
Which of the statements given above is/are correct?
a. 1 only There is a very rare possibility that UPSC will ask about the
b. 2 and 3 only defunct bodies. Simply read their basics and focus more on
c. 1 and 3 only NITI AAYOG now.
d. 1,2 and 3

JOIN ME AT TELEGRAM – UPSC PRELIMS WITH NEELESH (AIR 442 UPSC CSE 2021) https://t.me/UPSCPrelimsWithNeelesh
FREE CSAT COURSE ON YOUTUBE – CIVIL SERVICES WITH NEELESH, Best PYQ Document for Prelims – www.neeleshair442.com,
Best test series on – WWW.CSETOPPER.COM ©Copyright Reserved with the Author
TOPIC 33 OTHER CONSTITUIONAL DIMENSIONS (Chapter 64 -Co-operative
Societies, Chapter 65 – Official Language, Chapter 66 – Public
Services, Chapter 67 – Rights an Liabilities of the Government,
Chapter 68 – Special Provisions relating to Certain Classes)

Total Number of Questions Asked from 2011-23 0


My Instructions related to these chapters This section is not very important. At best, focus on
a)Co-operative Societies
b)Official Language (esp Classical Language subtopic)
For rest things, just have a brief reading.

TOPIC 34 POLITICAL DYNAMICS (Chapter 69 -Political Parties, Chapter 70 –


Role of Regional Parties, Chapter 71 – Elections, Chapter 72 – Election
Laws, Chapter 73 – Electoral Reforms, Chapter 74 – Voting Behaviour,
Chapter 75 – Coalition Government, Chapter 76 – Anti-Defection Law,
Chapter 77 – Pressure Groups, Chapter 78 – National Integration,
Chapter 79 - Foreign Policy)
Total Number of Questions Asked from 2011-23 7
My Instructions related to these chapters This section has only few important chapters.
The list of most important chapters amongst them are
a) Election
b) Election Laws
c) Electoral Reforms
d) Anti-Defection Law
Even amongst it, ANTI DEFECTION LAW IS MORE IMPORTANT. Rest is
just for brief reading.
JOIN ME AT TELEGRAM – UPSC PRELIMS WITH NEELESH (AIR 442 UPSC CSE 2021) https://t.me/UPSCPrelimsWithNeelesh
FREE CSAT COURSE ON YOUTUBE – CIVIL SERVICES WITH NEELESH, Best PYQ Document for Prelims – www.neeleshair442.com,
Best test series on – WWW.CSETOPPER.COM ©Copyright Reserved with the Author
PREVIOUS YEAR QUESTIONS RELATED TO THESE CHAPTERS OF TOPICS
S.No. Question and Year Answer Explanation
1 With reference to the Delimitation Commission, c The Delimitation commission or Boundary commission of India
consider the following statements: (2012) is a commission established by the Government of India under
1. The orders of the Delimitation Commission cannot the provisions of the Delimitation Commission Act. The main
be challenged in a Court of Law. task of the commission is redrawing the boundaries of the
2. When the orders of the Delimitation Commission various assembly and Lok Sabha constituencies based on a
are laid before the Lok Sabha or State Legislative recent census. The representation from each State is not
Assembly, they cannot affect any modifications in the changed during this exercise. However, the number of SC and
orders. ST seats in a state are changed in accordance with the census.
Which of the statements given above is/are correct? The present delimitation of constituencies has been done on
a. 1 only the basis of 1971 census under the provisions of Delimitation
b. 2 only Act, 2001. The Commission is a powerful and independent body
c. Both 1 and 2 whose orders cannot be challenged in any court of law. So, 1 is
d. Neither 1 nor 2 correct.

The orders are laid before the Lok Sabha and the respective
State Legislative Assemblies. However, modifications are not
permitted. So, 2 is correct.

2 Consider the following statements: (2017) d Election Commission of India is a three-member body and it is
1. The Election Commission of India is a five-member the Election Commission which decides the election schedule
body. for both general elections and bye-elections. So, both 1 and 2
2. Union Ministry of Home Affairs decides the are correct.
election schedule for the conduct of both general
elections and bye-elections. It is also the function of Election Commission to resolve the
3. Election Commission resolves the disputes relating disputes relating to splits/mergers of recognized political
to splits/ mergers of recognized political parties. parties. So, 3 is also correct.
JOIN ME AT TELEGRAM – UPSC PRELIMS WITH NEELESH (AIR 442 UPSC CSE 2021) https://t.me/UPSCPrelimsWithNeelesh
FREE CSAT COURSE ON YOUTUBE – CIVIL SERVICES WITH NEELESH, Best PYQ Document for Prelims – www.neeleshair442.com,
Best test series on – WWW.CSETOPPER.COM ©Copyright Reserved with the Author
Which of the statements given above is/ are correct?
a. 1 and 2 only
b. 2 only
c. 2 and 3 only
d. 3 only

3 Consider of the following statements: (2018) b In the first Lok Sabha election held in 1952, Congress won 364
1. In the first Lok Sabha, the single largest party in the seats followed by Communist Party of India with 16 seats. So,
opposition was the Swatantra Party. it was the Communist Party of India which was the single
2. In the Lok Sabha, a “Leader of the Opposition” was largest party in opposition. So, 1 is wrong.
recognized for the first time in 1969.
3. In the Lok Sabha, if a party does not have a Leader of Opposition was recognised for the first time in 1969.
minimum of 75 members, its leader cannot be First LOP was Ram Subhag Singh, the leader of Indian National
recognized as the Leader of the Opposition. Congress (O). So, 2 is right.
Which of the statements given above is/are correct?
a. 1 and 3 only In order to get formal recognition as Leader of Opposition, the
b. 2 only concerned party must have at least 10 percent of the total
c. 2 and 3 only strength of the House (55 seats in Lok Sabha). Hence,
d. 1,2 and 3 statement 3 is not correct.

4 Which one of the following Schedules of the d Schedules are very important. Remember the list from
Constitution of India contains provisions regarding Laxmikanth.
anti- defection? (2014) For this question, answer is tenth schedule. Hence, the option
a. Second Schedule d.
b. Fifth Schedule
c. Eighth Schedule
d. Tenth Schedule

JOIN ME AT TELEGRAM – UPSC PRELIMS WITH NEELESH (AIR 442 UPSC CSE 2021) https://t.me/UPSCPrelimsWithNeelesh
FREE CSAT COURSE ON YOUTUBE – CIVIL SERVICES WITH NEELESH, Best PYQ Document for Prelims – www.neeleshair442.com,
Best test series on – WWW.CSETOPPER.COM ©Copyright Reserved with the Author
5 With reference to anti-defection law in India, consider b The anti-defection law enshrined through the introduction of
the following statements? (2022) the Tenth Schedule in the Constitution of India comprises 8
1. The law specifies that a nominated legislator paragraphs.
cannot join any political party within six months of Paragraph 2 – Disqualification on grounds of defection – This
being appointed to the House. section deals with the crux of the legislation, specifying factors
2. The law does not provide any time-frame within on which a member cannot be disqualified from the Parliament
which the presiding officer has to decide a defection or the State Assembly. Paragraph 2.3 states that a nominated
case. member shall be disqualified if he/she joins any political party
Which of the statements given above is/are correct? after six months from the date he/she takes his seat. Hence,
a. 1 only statement 1 is not correct.
b. 2 only
c. Both 1 and 2 The law does not provide a time-frame within which the
d. Neither 1 nor 2 presiding officer has to decide a defection case. There have
been many instances when a speaker has not determined the
case of a defecting MLA until the end of the legislature term.
So, statement 2 is correct.

6 Consider the following statements: (2020) d Qualification for a person to become a minister of state
1. According to the Constitution of India, a person legislature are:
who is eligible to vote can be made a minister in a a. He must be a citizen of India
State for six months even if he/she is not a member b. He must take and subscribe to an oath or affirmation before
of the Legislature of that State. the person authorized by the Election Commission for the
2. According to the Representation of People’s Act, purpose. In his oath or affirmation, he swears to bear true faith
1951, a person convicted of a criminal offence and and allegiance to the Constitution of India and to uphold the
sentenced to imprisonment for five years is sovereignty and integrity of India
permanently disqualified from contesting an election c. He must not be less than 25 years of age in case of legislative
even after his release from prison. assembly and not less than 30 years of age in the case of
Which of the statements given above is/are correct? legislative council
a. 1 only
JOIN ME AT TELEGRAM – UPSC PRELIMS WITH NEELESH (AIR 442 UPSC CSE 2021) https://t.me/UPSCPrelimsWithNeelesh
FREE CSAT COURSE ON YOUTUBE – CIVIL SERVICES WITH NEELESH, Best PYQ Document for Prelims – www.neeleshair442.com,
Best test series on – WWW.CSETOPPER.COM ©Copyright Reserved with the Author
b. 2 only d. He must possess other qualifications as prescribed by the
c. Both 1 and 2 Parliament
d. Neither 1 nor 2
Hence, statement 1 is not correct as any person who is eligible
for vote cannot be a minister of the state legislature (minimum
age to vote is 18 years while for a minister it is 25 or 30 years
as per his membership to a house)
According to the Representation of People’s Act, 1951, a person
convicted for any offence and sentenced to imprisonment for
not less than two years shall be disqualified from the date of
such conviction and shall continue to be disqualified for a
further period of six years since his release. Hence statement 2
is not correct.

7 Consider the following statements: (2023) a In India, prisons are governed by the Prisons Act, 1894 which
Statement-I: In India, prisons are managed by State vested the Provincial Governments (now called States) the
Governments with regulations for the day-to-day power to make rules on prisons. Also, till date, the
administration of prisons. management and administration of prisons falls exclusively in
Statement-II: In India, prisons are governed by the the domain of the State Governments, and is governed by the
Prisons Act, 1894 which expressly kept the subject of Prisons Act, 1894 and the Prison Manuals of the respective
prisons in the control of Provincial Governments. State Governments. Hence statements 1 and 2 are correct and
Which one of the following is correct in respect of the statement 2 is the correct explanation of statement 1.
above statements?
(a) Both Statement-I and Statement-II are correct and
Statement-II is the correct explanation for Statement-I
(b) Both Statement-I and Statement-II are correct and
Statement-II is not the correct explanation for
Statement-I
(c) Statement-I is correct but Statement-II is incorrect
JOIN ME AT TELEGRAM – UPSC PRELIMS WITH NEELESH (AIR 442 UPSC CSE 2021) https://t.me/UPSCPrelimsWithNeelesh
FREE CSAT COURSE ON YOUTUBE – CIVIL SERVICES WITH NEELESH, Best PYQ Document for Prelims – www.neeleshair442.com,
Best test series on – WWW.CSETOPPER.COM ©Copyright Reserved with the Author
(d) Statement-I is incorrect but Statement-II is correct

TOPIC 35 Chapter 80 (National Commission to Review the Working of the


Constitution)
Total Number of Questions Asked from 2011-23 0
My Instructions related to these chapters Just remember the name of M N Venkatachaliah

TOPIC 36 APPENDICES (Appendix I, II, III, IV, V, VI)


Total Number of Questions Asked from 2011-23 2
My Instructions related to these chapters Appendices are important. They can be used in one way or the other
for framing question. Focus should be given on
1 Recognizing which subject belong to which list (esp remember the
ones which were transferred to concurrent list)
2 Important Constitutional Amendments
3 Chronology of PRESIDENT, VICE PRESIDENT AND PM
4 First Deputy Prime Minister, first Speaker, first CJI, first CEC, first
Chairman of UPSC, first CAG, first Attorney General, first NHRC
Chairman, first NCW, first NCPCR, first NCBC, FIRST NCM, First NCSC,
First NCST

PREVIOUS YEAR QUESTIONS RELATED TO THESE CHAPTERS OF TOPICS


S.No. Question and Year Answer Explanation
1 The Ninth Schedule was introduced in the a Ninth schedule was introduced through the first Constitutional
Constitution of India during the prime minister ship Amendment Act, 1951. That time, the Prime Minister of India
of: (2019) was Jawaharlal Nehru. So, d

JOIN ME AT TELEGRAM – UPSC PRELIMS WITH NEELESH (AIR 442 UPSC CSE 2021) https://t.me/UPSCPrelimsWithNeelesh
FREE CSAT COURSE ON YOUTUBE – CIVIL SERVICES WITH NEELESH, Best PYQ Document for Prelims – www.neeleshair442.com,
Best test series on – WWW.CSETOPPER.COM ©Copyright Reserved with the Author
a. Jawaharlal Nehru
b. Lal Bahadur Shastri
c. Indira Gandhi
d. Morarji Desai

2 In India, which one of the following Constitutional a During the initial fifteen months of the working of the
Amendments was widely believed to be enacted to Constitution, certain difficulties were brought to light by judicial
overcome the judicial interpretations of the decisions and pronouncements specially in regard to the
Fundamental Rights? (2023) chapter on fundamental rights. The citizen's right to freedom of
(a) 1st Amendment speech and expression guaranteed by article 19(1)(a) was held
(b) 42nd Amendment by some courts to be so comprehensive so as not to render a
(c) 44th Amendment person culpable even if he advocates murder and other crimes
(d) 86thAmendment of violence. The main objects of this First Amendment Act, 1951
was thus enacted, accordingly to amend article 19 for the
purposes indicated above. The Constitution (First Amendment)
Act, 1951, enacted in 1951, made several changes to the
Fundamental Rights provisions of the Indian constitution. It
provided means to restrict freedom of speech and expression,
validation of zamindari abolition laws, and clarified that the
right to equality does not bar the enactment of laws which
provide "special consideration" for weaker sections of society.
Hence option (a) is the correct answer.

JOIN ME AT TELEGRAM – UPSC PRELIMS WITH NEELESH (AIR 442 UPSC CSE 2021) https://t.me/UPSCPrelimsWithNeelesh
FREE CSAT COURSE ON YOUTUBE – CIVIL SERVICES WITH NEELESH, Best PYQ Document for Prelims – www.neeleshair442.com,
Best test series on – WWW.CSETOPPER.COM ©Copyright Reserved with the Author
TOPIC 37 MISCELLANEOUS
Total Number of Questions Asked from 2011-23 24
My Instructions related to these chapters Simply read these, prepare these topics well which are asked and
don’t do mistake if they come. The miscellaneous topic should not
divert you from the original syllabus.

PREVIOUS YEAR QUESTIONS RELATED TO THESE CHAPTERS OF TOPICS


S.No. Question and Year Answer Explanation
1 With reference to the consumers’ rights/ privileges c As per the Consumer Protection Act, consumers are
under the provisions of law in India, which of the empowered to take samples for food testing. Also, in case of
following statements is/are correct? (2012) death of a consumer, his/her legal heir can file a complaint in
1. Consumers are empowered to take samples for the consumer forum on his/her behalf. But during filing of
food testing. complaint, a nominal fee has to be paid. So, c is correct.
2. When a consumer files a complaint in any forum,
no fee is required to be paid.
3. In case of death of a consumer, his/her legal heir
can file a complaint in the consumer forum on his/her
behalf.
Select the correct answer using the codes given
below:
a. 1 only
b. 2 and 3 only
c. 1 and 3 only
d. 1,2 and 3

JOIN ME AT TELEGRAM – UPSC PRELIMS WITH NEELESH (AIR 442 UPSC CSE 2021) https://t.me/UPSCPrelimsWithNeelesh
FREE CSAT COURSE ON YOUTUBE – CIVIL SERVICES WITH NEELESH, Best PYQ Document for Prelims – www.neeleshair442.com,
Best test series on – WWW.CSETOPPER.COM ©Copyright Reserved with the Author
2 The National Green Tribunal Act, 2010 was enacted in a National Green Tribunal Act, 2010 is an Act of the Parliament
consonance with which of the following provisions of of India which enables the creation of a special tribunal to
the Constitution of India? (2012) handle the expeditious disposal of the cases pertaining
1. Right to healthy environment, construed as a part to environmental issues. It draws inspiration from India's
of Right to Life under Article 21. constitutional provision of (Constitution of India/Part III) Article
2. Provision of grants for raising the level of 21 Protection of life and personal liberty, which assures the
administration in the Scheduled Areas for the welfare citizens of India the right to a healthy environment.
of Scheduled Tribes under Article 275(1) So, only 1 is correct.
3. Powers and functions of Gram Sabha as mentioned
under Article 243(A)
Select the correct answer using the codes given
below:
a. 1 only
b. 2 and 3 only
c. 1 and 3 only
d. 1,2 and 3

3 How is the National Green Tribunal (NGT) different b The National Green Tribunal has been established on
from The Central Pollution Control Board (CPCB)? 18.10.2010 under the National Green Tribunal Act 2010 for
(2018) effective and expeditious disposal of cases relating to
1. The NGT has been established by an Act whereas environmental protection and conservation of forests and
the CPCB has been created by an executive order of other natural resources including enforcement of any legal
the Government. right relating to environment and giving relief and
2. The NGT provide environmental justice and helps compensation for damages to persons and property and for
reduce the burden of litigation in the higher courts matters connected therewith or incidental thereto. It is a
whereas the CPCB promotes cleanliness of streams specialized body equipped with the necessary expertise to
and wells, and aims to improve the quality of air in the handle environmental disputes involving multi-disciplinary
country. issues. The Tribunal shall not be bound by the procedure laid
Which of the statements given above is/are correct?
JOIN ME AT TELEGRAM – UPSC PRELIMS WITH NEELESH (AIR 442 UPSC CSE 2021) https://t.me/UPSCPrelimsWithNeelesh
FREE CSAT COURSE ON YOUTUBE – CIVIL SERVICES WITH NEELESH, Best PYQ Document for Prelims – www.neeleshair442.com,
Best test series on – WWW.CSETOPPER.COM ©Copyright Reserved with the Author
a. 1 only down under the Code of Civil Procedure, 1908, but shall be
b. 2 only guided by principles of natural justice.
c. Both 1 and 2
d. Neither 1 nor 2 The Tribunal's dedicated jurisdiction in environmental matters
shall provide speedy environmental justice and help reduce the
burden of litigation in the higher courts. The Tribunal is
mandated to make and endeavour for disposal of applications
or appeals finally within 6 months of filing of the same. Initially,
the NGT is proposed to be set up at five places of sittings and
will follow circuit procedure for making itself more accessible.
New Delhi is the Principal Place of Sitting of the Tribunal and
Bhopal, Pune, Kolkata and Chennai shall be the other four place
of sitting of the Tribunal.

While
The Central Pollution Control Board (CPCB) of India is a
statutory organization under the Ministry of Environment,
Forest and Climate Change (Mo.E.F.C.C.). It was established in
1974 under the Water (Prevention and Control of pollution) Act,
1974. The CPCB is also entrusted with the powers and functions
under the Air (Prevention and Control of Pollution) Act, 1981. It
serves as a field formation and also provides technical services
to the Ministry of Environment and Forests under the
provisions of the Environment (Protection) Act, 1986. It
coordinates the activities of the State Pollution Control Boards
by providing technical assistance and guidance and also
resolves disputes among them. It is the apex organization in
country in the field of pollution control, as a technical wing of
MoEFCC. The board is led by its Chairperson appointed by
JOIN ME AT TELEGRAM – UPSC PRELIMS WITH NEELESH (AIR 442 UPSC CSE 2021) https://t.me/UPSCPrelimsWithNeelesh
FREE CSAT COURSE ON YOUTUBE – CIVIL SERVICES WITH NEELESH, Best PYQ Document for Prelims – www.neeleshair442.com,
Best test series on – WWW.CSETOPPER.COM ©Copyright Reserved with the Author
the Appointments Committee of the Cabinet of
the Government of India.

4 With reference to the ‘Prohibition of Benami Property b The PBPT Act, 1988 defines Benami transactions, prohibit them
Transactions Act, 1988 (PBPT Act)’, and further provides that the violation of PBPT Act is punishable
Consider the following statements: (2017) with fine and imprisonment. The PBPT Act prohibits recovery of
1. A property transaction is not treated as a benami the property held benami from benamidar by the real owner.
transaction if the owner of the property is not aware But on the basis of principle, that “Ignorance of law cannot be
of the transaction. an excuse for violation of law”, So, statement 1 is considered
2. Properties held benami are liable for the wrong.
confiscation by the Government.
3. The Act provides for three authorities for Properties held benami are liable for confiscation by the
investigations but does not provide for any appellate Government without payment of confiscation. So, statement 2
mechanism. is correct.
Which of the statements given above is/are correct?
a. 1 only An appellate mechanism has been provided under the PBPT Act
b. 2 only in the form of adjudicating authority and appellate tribunal
c. 1 and 3 only constituted by the Central Government. Hence, statement 3 is
d. 2 and 3 only not correct.

5 Which of the following is/ are the function/ functions c The Cabinet Secretariat functions directly under the Prime
of the Cabinet Secretariat? (2014) Minister. The administrative head of the Secretariat is the
1. Preparation of the agenda for Cabinet Meetings. Cabinet Secretary who is also the ex-officio Chairman of the
2. Secretarial Assistance to Cabinet Committees. Civil Services Board. The business allocated to Cabinet
3. Allocation of the Financial resources to the Secretariat under Government of India (Allocation of Business)
Ministries. Rules, 1961 includes (i) Secretarial assistance to the Cabinet
Select the correct answer using the code given below: and Cabinet Committees; and (ii) Rules of Business.
a. 1 only
b. 2 and 3 only
JOIN ME AT TELEGRAM – UPSC PRELIMS WITH NEELESH (AIR 442 UPSC CSE 2021) https://t.me/UPSCPrelimsWithNeelesh
FREE CSAT COURSE ON YOUTUBE – CIVIL SERVICES WITH NEELESH, Best PYQ Document for Prelims – www.neeleshair442.com,
Best test series on – WWW.CSETOPPER.COM ©Copyright Reserved with the Author
c. 1 and 2 only The Cabinet Secretariat is responsible for the administration of
d. 1,2 and 3 the Government of India (Transaction of Business) Rules, 1961
and Government of India (Allocation of Business) Rules, 1961
facilitating smooth transaction of business in Ministries/
Departments. The Secretariat assists in decision-making in
Government by ensuring Inter-Ministerial coordination, ironing
out differences amongst Ministries/Departments and evolving
consensus through the instrumentality of the standing/adhoc
Committees of Secretaries. Management of major crisis
situations in the country and coordinating activities of various
ministries in such a situation is also one of the functions of the
Cabinet Secretariat.
Its functions include
a. Allocation and disposal of Government Business

b. Support to Cabinet Committees

The secretarial assistance, provided by Cabinet Secretariat to


the Cabinet and Cabinet committees, includes
• Convening of the meetings of the Cabinet & its Committees
on the orders of the Prime Minister.
• Preparation and circulation of the agenda.
• Circulation of papers related to the cases on the agenda.
• Preparation of record of discussions.
• Circulation of the record of discussions after obtaining the
approval of the Prime Minister.
• Monitoring implementation of decisions taken by the Cabinet
and its Committees.

JOIN ME AT TELEGRAM – UPSC PRELIMS WITH NEELESH (AIR 442 UPSC CSE 2021) https://t.me/UPSCPrelimsWithNeelesh
FREE CSAT COURSE ON YOUTUBE – CIVIL SERVICES WITH NEELESH, Best PYQ Document for Prelims – www.neeleshair442.com,
Best test series on – WWW.CSETOPPER.COM ©Copyright Reserved with the Author
The Cabinet Secretariat is the custodian of the papers of the
Cabinet meetings.

c. Promotion of Inter-Ministerial Coordination


While each Ministry is responsible for acting on its own for
expeditious implementation of Government policies, plans and
programmes, where inter-Ministerial cooperation is involved,
they often seek the assistance of the Cabinet Secretariat. The
inter-Ministerial problems are dealt with in the meetings of the
Committees of Secretaries (COS). Committees are constituted
for discussing specific matters and proposals emanating from
various Secretaries to the Government and meetings are held
under the chairmanship of the Cabinet Secretary. These
committees have been able to break bottlenecks or secure
mutually supporting inter-Ministerial action.

The cabinet secretariat does not allocate financial resources


to the Ministries.
So, only 1 and 2 are correct.

6 Consider the following statements: (2019) a Article 102 of the Constitution provides that a person shall be
1. The Parliament (Prevention of Disqualification) disqualified from being chosen as a Member of Parliament (MP)
Act, 1959 exempts several posts from disqualification if he holds an office of profit under the government of India or
on the grounds of ‘Office of Profit’. the government of a state. However, Parliament can declare by
2. The above-mentioned Act was amended five times. law that the holding of certain offices will not incur this
3. The term “Office of Profit” is well defined in the disqualification. The Parliament (Prevention of
Constitution of India. Disqualification) Act, 1959 lists certain offices of profit under
Which of the statements given above is/are correct? the central and state governments, which do not disqualify the
a. 1 and 2 only holders from being an MP. The Chairperson of the National
JOIN ME AT TELEGRAM – UPSC PRELIMS WITH NEELESH (AIR 442 UPSC CSE 2021) https://t.me/UPSCPrelimsWithNeelesh
FREE CSAT COURSE ON YOUTUBE – CIVIL SERVICES WITH NEELESH, Best PYQ Document for Prelims – www.neeleshair442.com,
Best test series on – WWW.CSETOPPER.COM ©Copyright Reserved with the Author
b. 3 only Commission for Scheduled Castes and Scheduled Tribes is
c. 2 and 3 only exempted from disqualification as per this list.
d. 1,2 and 3 The Act has been amended 5 times, since its formulation, in
the years 1960, 1992, 1993, 2006 and 2013.
The Constitution of India does not clearly define what
constitutes an office of profit, but the definition has evolved
over the years with interpretations made in various court
judgments. Hence, statement 3 is not correct.
Therefore, option (a) is the correct answer.
7 Consider the following statements: (2019) a CAMPA Act or Compensatory Afforestation Fund Act is an
1. As per law, the Compensatory Afforestation Fund Indian legislation that seeks to provide an appropriate
Management and Planning Authority exists at both institutional mechanism, both at the Centre and in each State
National and State levels. and Union Territory, to ensure expeditious utilization in efficient
2. People’s participation is mandatory in the and transparent manner of amounts released in lieu of forest
compensatory afforestation programmes carried out land diverted for non-forest purpose which would mitigate
under the Compensatory Afforestation Fund Act, impact of diversion of such forest land. So, 1 is correct.
2016.
Which one of the following statements above is/are People’s participation is not mandatory in the compensatory
correct? afforestation programmes carried out under the Compensatory
a. 1 only Afforestation Fund Act, 2016. So, 2 is wrong.
b. 2 only
c. Both 1 and 2
d. Neither 1 nor 2

8 In India, which of the following review the a In India, the parliamentary scrutiny of the regulators can take
independent regulators in sectors like place through Question Hour; discussions in Parliament and
Parliamentary Committees. The Parliamentary Departmental

JOIN ME AT TELEGRAM – UPSC PRELIMS WITH NEELESH (AIR 442 UPSC CSE 2021) https://t.me/UPSCPrelimsWithNeelesh
FREE CSAT COURSE ON YOUTUBE – CIVIL SERVICES WITH NEELESH, Best PYQ Document for Prelims – www.neeleshair442.com,
Best test series on – WWW.CSETOPPER.COM ©Copyright Reserved with the Author
telecommunications, insurance, electricity, etc.? related standing committees may do such work and the ad
(2019) hoc committees sometimes set up to review the working of
1. Ad Hoc Committees set up by the Parliament. regulators specific to their respective departments. Hence,
2. Parliamentary Department Related Standing both 1 and 2 are correct.
Committees.
3. Finance Commission. The role played by Finance Commission is related to Fiscal
4. Financial Sector Legislative Reforms Commission. Federalism whereas the role played by NITI Aayog is related
5. NITI Aayog. to Cooperative Federalism. Both these Commissions are of
Select the correct answer using the code given below: advisory nature, so, they do not review the functioning of any
a. 1 and 2 regulatory. Similarly, FSLRC was setup once to review financial
b. 1,3 and 4 legislations and not regulators in the country. So, 3, 4 and 5
c. 3,4 and 5 are wrong.
d. 2 and 5

9 Consider the following statements: (2016) d The Chief Secretary is the top-most executive official
1. The Chief Secretary in a State is appointed by the and senior-most civil servant of the state government. The
Governor of that State. Chief Secretary is the ex-officio head of the state Civil Services
2. The Chief Secretary in a State has a fixed tenure. Board, the State Secretariat, the state cadre Indian
Which of the statements given above is/are correct? Administrative Service and all civil services under the rules of
a. 1 only business of the state government. The Chief Secretary acts as
b. 2 only the principal advisor to the chief minister on all matters of state
c. Both 1 and 2 administration. The Chief Secretary is an officer of the Indian
d. Neither 1 nor 2 Administrative Service. The Chief Secretary is the senior-most
cadre post in the state administration, ranking 23rd on
the Indian order of precedence. The Chief Secretary acts as
an ex-officio secretary to the state cabinet, therefore
called "Secretary to the Cabinet". The status of this post is equal
to that of a Secretary to the Government of India.

JOIN ME AT TELEGRAM – UPSC PRELIMS WITH NEELESH (AIR 442 UPSC CSE 2021) https://t.me/UPSCPrelimsWithNeelesh
FREE CSAT COURSE ON YOUTUBE – CIVIL SERVICES WITH NEELESH, Best PYQ Document for Prelims – www.neeleshair442.com,
Best test series on – WWW.CSETOPPER.COM ©Copyright Reserved with the Author
Chief Secretaries are chosen by the state's Chief Minister.
State Chief Secretaries are IAS officers generally equivalent in
rank to a Secretary to Government of India. So, 1 is wrong.

The chief secretary does not have a fixed tenure. So, 2 is wrong.

10 Democracy’s superior virtue lies in the fact that it calls a Democracy Superior virtue lies in the fact that it calls into
into activity: (2017) activity the intelligence and character of ordinary men and
a. The intelligence and character of ordinary men and women. It allows us to vote by exercising our intellect and it
women. gives us liberty and freedom of expression, and ensures
b. The methods for strengthening executive adherence of moral values (character) such as justice and
leadership. equality. So, a is correct.
c. A superior individual with dynamism and vision.
d. A band of dedicated party workers

11 Which one of the following reflects the most b Since individuals live together in a society, complete absence of
appropriate relationship between law and liberty? restraints would be neither possible nor desirable. It has been
a. If there are more laws, there is less liberty. very aptly said that your liberty to swing your arm ends there
b. If there are no laws, there is no liberty. where my nose begins. For liberty to be enjoyed by everyone,
c. If there is liberty, laws have to be made by the it should have reasonable restraints. The freedom of many
people. requires restraint of law on the freedom of some. Liberty
d. If laws are changed too often, liberty is in danger. Means freedom of thought, expression and action. However, It
does not mean freedom to do anything that one desires or likes.
If that were to be permitted then a large number of people will
not be able to enjoy their freedom. Therefore, freedoms are
defined in such a manner that every person will enjoy her
freedom without threatening freedom of others and without
endangering the law-and-order situation. So, if there are no

JOIN ME AT TELEGRAM – UPSC PRELIMS WITH NEELESH (AIR 442 UPSC CSE 2021) https://t.me/UPSCPrelimsWithNeelesh
FREE CSAT COURSE ON YOUTUBE – CIVIL SERVICES WITH NEELESH, Best PYQ Document for Prelims – www.neeleshair442.com,
Best test series on – WWW.CSETOPPER.COM ©Copyright Reserved with the Author
laws then there will be chaos and no individual can enjoy
his/her liberty. So, b is correct.

12 Which of the following are regarded as the main c The term ‘Rule of Law’ means a government based on the
features of the “Rule of Law”? (2018) principles of law and not of men.
1. Limitation of Powers
2. Equality before law Option c is correct. Among the given choices, only Limitation Of
3. People’s responsibility to the government Powers, Equality Before Law And Liberty And Civil Rights are the
4. Liberty and civil rights features of the Rule of Law.
Select the correct answer using the code given below:
a. 1 and 3 only Limitation of Power – Rule of Law is against any form of
b. 2 and 4 only arbitrary action on the part of the government. The
c. 1,2 and 4 only government is bound to work within the legal framework.
d. 1,2,3 and 4
Equality before Law – Everyone, irrespective of his/her rank or
status, is subject to the ordinary law and to the jurisdiction of
ordinary court. It means universal subjection of all classes to
one law administered by ordinary court.

Liberty and civil rights – rule of law acts as a safeguard for


liberty and civil rights.

Government is responsible to the citizens of its country and not


vice-versa. Hence, 3rd feature is not correct.

13 Right to vote and to be elected in India is a (2017) c Right to vote and to be elected is a Constitutional Right. So, c is
a. Fundamental Right correct.
b. Natural Right
c. Constitutional Right
JOIN ME AT TELEGRAM – UPSC PRELIMS WITH NEELESH (AIR 442 UPSC CSE 2021) https://t.me/UPSCPrelimsWithNeelesh
FREE CSAT COURSE ON YOUTUBE – CIVIL SERVICES WITH NEELESH, Best PYQ Document for Prelims – www.neeleshair442.com,
Best test series on – WWW.CSETOPPER.COM ©Copyright Reserved with the Author
d. Legal Right

14 Which one of the following best defines the term a State is a political organization of society.
‘State’? (2021) The elements of the state are
a. A community of persons permanently occupying a a) People (community of persons permanently living)
definite territory independent of external control and b) Government (A political organization or agency through
possessing and organized government. which the collective will of the population is expressed, i.e.
b. A politically organized people of a definite territory government),
and possessing an authority to govern them, maintain c) Territory (A territory upon which they permanently reside)
law and order, protect their natural rights and d) Sovereignty: Internal sovereignty and independence from
safeguard their means of sustenance. foreign control.
c. A number of persons who have been living in a So, a is correct.
definite territory for a very long time with their own
culture, tradition and government.
d. A society permanently living in a definite territory
with a central authority and an executive responsible
to the central authority and an independent judiciary.

15 In the context of Polity, which one of the following d The term ‘liberty’ means the absence of restraints on the
would you accept as the most appropriate definition activities of individuals, and at the same time, providing
of Liberty? (2019) opportunities for the development of individual personalities.
a. Protection against the tyranny of political rulers. However, liberty does not mean ‘license’ to do what one likes.
b. Absence of restraint. It has to be enjoyed within the limitations mentioned in the
c. Opportunity to do whatever one likes. Constitution itself i.e., it is not absolute but qualified. So, d is
d. Opportunity to develop oneself fully. correct.

16 In the context of India, which one of the following is d It refers to an administrative policy-implementation group,
the characteristic appropriate for bureaucracy? (2020) which is a body of non-elected government officials It is an
agency for implementation of public policy. They are politically
JOIN ME AT TELEGRAM – UPSC PRELIMS WITH NEELESH (AIR 442 UPSC CSE 2021) https://t.me/UPSCPrelimsWithNeelesh
FREE CSAT COURSE ON YOUTUBE – CIVIL SERVICES WITH NEELESH, Best PYQ Document for Prelims – www.neeleshair442.com,
Best test series on – WWW.CSETOPPER.COM ©Copyright Reserved with the Author
a. An agency for widening the scope of parliamentary neutral and their main job is to facilitate effective working of
democracy. different government departments and policy
b. An agency for strengthening the structure of implementations. Other functions are running the
federalism. administration, advisors to political executives, assisting
c. An agency for facilitating political stability and ministers in law making, record keeping, financial
economic growth. administration etc. So, d is correct.
d. An agency for the implementation of public policy.

17 Which one of the following factors constitutes the d The Separation of powers between the legislature, the
best safeguard of liberty in a liberal democracy? executive and the judiciary constitute an important safeguard
(2021) of liberty in a liberal democracy. The separation of powers
a. A committed judiciary. between executive, legislative and judiciary minimizes the
b. Centralization of powers. possibility of arbitrary excesses by the government ->
c. Elected government demarcation prevents the concentration of excessive power by
d. Separation of powers. any branch of the Government, thus helps to safeguard the
liberty and rights of the people in a democracy. So, d is correct.

18 One of the implications of equality in society is the a The term ‘equality’ means the absence of special privileges to
absence of (2017) any section of the society, and the provision of adequate
(a) Privileges opportunities for all individuals without any discrimination.
(b) Restraints Right to equality (Article 18) abolished titular privileges (except
(c) Competition military or academic) granted to citizens of India. So, a is
(d) Ideology correct.

19 Which one of the following statements is correct? c A right is essentially an entitlement or a justified claim. It
(2017) denotes what we are entitled to as citizens, as individuals and
a. Rights are claims of the State against the citizens. as human beings.
b. Rights are privileges which are incorporated in the
Constitution of a State.
JOIN ME AT TELEGRAM – UPSC PRELIMS WITH NEELESH (AIR 442 UPSC CSE 2021) https://t.me/UPSCPrelimsWithNeelesh
FREE CSAT COURSE ON YOUTUBE – CIVIL SERVICES WITH NEELESH, Best PYQ Document for Prelims – www.neeleshair442.com,
Best test series on – WWW.CSETOPPER.COM ©Copyright Reserved with the Author
c. Rights are claims of the citizens against the State. In most cases, the claimed rights are directed towards the state.
d. Rights are privileges of a few citizens against the That is, through these rights, people make demands upon the
many state. When a citizen asserts his right to education, he calls up
the state to make provisions for his basic education. Thus, rights
place an obligation upon the state to act in certain kinds of
ways.

So, combining the above two characteristics of Rights, it can be


concluded that Rights are claims of the citizens against the
state. Hence, option (c) is the correct answer.

20 The national motto of India, ‘Satayamev Jayate’ d Satyameva Jayate is taken from Mundaka Upanishad. So, d is
inscribed below the Emblem of India is taken from: correct.
(2014)
a. Katha Upanishad
b. Chandogya Upanishad
c. Aitareya Upanishad
d. Mundaka Upanishad

21 Consider the following statements in respect of d Statement 1 is not correct. Article 18(1) prohibits the state to
Bharat Ratna and Padma Awards: (2021) confer titles on anybody whether a citizen or a non-citizen.
1. Bharat Ratna and Padma Awards are titles under Military and academic distinctions are however, exempted from
the Article 18(1) of the Constitution of India. the prohibitions. National awards such as Bharat Ratna, and
2. Padma Awards, which were instituted in the year Padma awards do not amount to titles within the meaning of
1954, were suspended only once. Article 18(1) of the Constitution, and hence they cannot be
3. The number of Bharat Ratna Awards is restricted to used as prefix or suffix to names.
a maximum of five in a particular year.
Which of the above statements are not correct? Statement 2 is not correct. Government of India instituted two
a. 1 and 2 only civilian awards – Bharat Ratna and Padma Vibhushan in 1954.
JOIN ME AT TELEGRAM – UPSC PRELIMS WITH NEELESH (AIR 442 UPSC CSE 2021) https://t.me/UPSCPrelimsWithNeelesh
FREE CSAT COURSE ON YOUTUBE – CIVIL SERVICES WITH NEELESH, Best PYQ Document for Prelims – www.neeleshair442.com,
Best test series on – WWW.CSETOPPER.COM ©Copyright Reserved with the Author
b. 2 and 3 only Since then, Padma Awards are announced every year on the
c. 1 and 3 only occasion of Republic Day except for brief interruption(s) during
d. 1,2 and 3 the years 1978 and 1979 and 1993 to 1997. Thus, Padma
awards were suspended more than once.

Statement 3 is not correct. The number of Bharat Ratna Awards


is restricted to a maximum of three in a particular year. It was
awarded to 4 people in the same year only once in 1999.

22 Which one of the following statements best reflects c Constitutionalism’ means limited government or limitation on
the chief purpose of the 'Constitution' of a country? government. The main objective or purpose of the constitution
(2023) of a country is provide a framework for a government by
(a) It determines the objective for the making of deciding the nature and form of the government.
necessary laws.
(b) It enables the creation of political offices and a As a constitution is the basic fundamental law of a State, it also
government. lays down the responsibilities of the different limbs of the
(c) It defines and limits the powers of government. government; the executive, the legislature and the judiciary.
(d) It secures social justice, social equality and social And accordingly, the legislature makes laws in keeping with the
security. directions and limits provided in the constitution. Hence option
(c) is the correct answer.

23 Consider the following statements: (2023) c Statement 1 is correct: Article 16 (4) provides that the State can
Statement-I: The Supreme Court of India has held in enact legislation for the reservation of posts in the government
some judgements that the reservation policies made sector or jobs in favour of the backward classes of citizens,
under Article 16(4) of the Constitution of India would which the State considers to have not been adequately
be limited by Article 335 for maintenance of efficiency represented in the services of the State. In the past seven
of the administration. decades of constitutional jurisprudence on reservations, the
Statement-II: Article 335 of the Constitution of India Supreme Court of India has consistently referred to the notions
defines the term 'efficiency of administration'. of “efficiency” and “merit,” while adjudicating the validity of
JOIN ME AT TELEGRAM – UPSC PRELIMS WITH NEELESH (AIR 442 UPSC CSE 2021) https://t.me/UPSCPrelimsWithNeelesh
FREE CSAT COURSE ON YOUTUBE – CIVIL SERVICES WITH NEELESH, Best PYQ Document for Prelims – www.neeleshair442.com,
Best test series on – WWW.CSETOPPER.COM ©Copyright Reserved with the Author
Which one of the following is correct in respect of the various reservation policies. The Court has held in several
above statements? judgments — Indra Sawhney v Union of India 1993; M Nagaraj
(a) Both Statements-I and Statement-II are correct and v Union of India 2006 — that the reservation policies made
Statement-II is the correct explanation for Statement-I under Article 16(4) of the Constitution would be limited by
(b) Both Statement-I and Statement-II are correct and Article 335, which provides for “maintenance of efficiency of
Statement-II is not the correct explanation for administration,” while considering the claims of the Scheduled
Statement-I Castes (SCs) and the Scheduled Tribes (STs) in the making of
(c) Statement-I is correct but Statement-II is incorrect appointments to public services and posts. This was done while
(d) Statement-I is incorrect but Statement-II is correct the Constitution does not define the term “efficiency of
administration.”
Statement 2 is not correct: Article 335 states that the claims of
the members of the Scheduled Castes and the Scheduled Tribes
shall be taken into consideration, consistently with the
maintenance of efficiency of administration, in the making of
appointments to services and posts in connection with the
affairs of the Union or of a State. This Article, has however, not
defined the expression 'efficiency of administration'. Hence
option (c) is the correct answer.

24 Consider the following statements : (2023) b Statement 1 is correct: According to Article 355 of the Indian
1. According to the Constitution of India, the Central Constitution, it shall be the duty of the Union to protect every
Government has a duty to protect States from State against external aggression and internal disturbance and
internal disturbances. to ensure that the government of every State is carried on in
2. The Constitution of India exempts the States from accordance with the provisions of this Constitution.
providing legal counsel to a person being held for
preventive detention. Statement 2 is correct: According to Article 22 of Constitution
3. According to the Prevention of Terrorism Act, 2002, of India (Protection against arrest and detention in certain
confession of the accused before the police cannot be cases):
used as evidence.
JOIN ME AT TELEGRAM – UPSC PRELIMS WITH NEELESH (AIR 442 UPSC CSE 2021) https://t.me/UPSCPrelimsWithNeelesh
FREE CSAT COURSE ON YOUTUBE – CIVIL SERVICES WITH NEELESH, Best PYQ Document for Prelims – www.neeleshair442.com,
Best test series on – WWW.CSETOPPER.COM ©Copyright Reserved with the Author
How many of the above statements are correct? (1) No person who is arrested shall be detained in custody
(a) Only one without being informed, as soon as may be, of the grounds for
(b) Only two such arrest nor shall he be denied the right to consult, and to
(c) All three be defended by, a legal practitioner of his choice.
(d) None (2) Every person who is arrested and detained in custody shall
be produced before the nearest magistrate within a period of
twenty four hours of such arrest excluding the time necessary
for the journey from the place of arrest to the court of the
magistrate and no such person shall be detained in custody
beyond the said period without the authority of a magistrate
(3) Nothing in clauses (1) and (2) shall apply (a) to any person
who for the time being is an enemy alien; or (b) to any person
who is arrested or detained under any law providing for
preventive detention.

Hence, According to Clause (3) Article 22 of the Constitution of


India, it is not mandatory for states to provide access to legal
counsel to the person held for preventive detention.

Statement 3 is not correct: The Prevention of Terrorism Act,


2002 was an Act passed by the Parliament of India in 2002, with
the objective of strengthening anti-terrorism operations. In
2004, the Act was repealed. As per the act, a confession made
by a person before a police officer (not lower in rank than a
Superintendent of Police) and recorded by that police officer
either in writing or on any mechanical or electronic device shall
be admissible in the trial of such person for an offence under
this Act.

JOIN ME AT TELEGRAM – UPSC PRELIMS WITH NEELESH (AIR 442 UPSC CSE 2021) https://t.me/UPSCPrelimsWithNeelesh
FREE CSAT COURSE ON YOUTUBE – CIVIL SERVICES WITH NEELESH, Best PYQ Document for Prelims – www.neeleshair442.com,
Best test series on – WWW.CSETOPPER.COM ©Copyright Reserved with the Author
Additional Information you should know
1. Concept of written constitution was born in USA
2. Press Council of India is non-constitutional, statutory, quasi-judicial body

CIVIL SERVICES WITH NEELESH


(TELEGRAM CHANNEL – UPSC PRELIMS WITH NEELESH)
OUR INITIATIVE ON YOUTUBE – CIVIL SERVICES WITH NEELESH CHANNEL –
FREE CSAT COURSE with Free tests on www.csetopper.com
JOIN ME AT TELEGRAM – UPSC PRELIMS WITH NEELESH (AIR 442 UPSC CSE 2021) https://t.me/UPSCPrelimsWithNeelesh
FREE CSAT COURSE ON YOUTUBE – CIVIL SERVICES WITH NEELESH, Best PYQ Document for Prelims – www.neeleshair442.com,
Best test series on – WWW.CSETOPPER.COM ©Copyright Reserved with the Author
OUR INITIATIVES on TELEGRAM
1. UPCOMING PRELIMS MARATHON FOR 2024 (TELEGRAM – UPSC PRELIMS WITH NEELESH) (BOTH FREE AND PAID) (FROM DECEMBER 2023)
2. FREE ETHICS AND ESSAY EVALUATION – ER NEELESH AIR 442 UPSC_CSE2021 (https://t.me/UpscWithNeelesh_AIR442)
3. SOCIOLOGY FRAMEWORK- TELEGRAM – SOCIOLOGY WITH NEELESH
4. CSAT MARATHON – CSAT WITH NEELESH
5. UPCOMING CSAT AND SOCIOLOGY MENTORSHIP BATCH
6. YEARLONG MENTORSHIP FOR 2024 (ADMISSIONS CLOSED NOW) (FOR BOTH PRELIMS AND MAINS)
7. YEARLONG MENTORSHIP FOR 2025 STUDENTS

UPCOMING INITIATIVES
1. ETHICS PROGRAM
2. SOCIOLOGY BASED ON PYQ
3. ESSAY COURSE AND MUCH MORE
4. MARATHON FOR 2025

JOIN ME AT TELEGRAM – UPSC PRELIMS WITH NEELESH (AIR 442 UPSC CSE 2021) https://t.me/UPSCPrelimsWithNeelesh
FREE CSAT COURSE ON YOUTUBE – CIVIL SERVICES WITH NEELESH, Best PYQ Document for Prelims – www.neeleshair442.com,
Best test series on – WWW.CSETOPPER.COM ©Copyright Reserved with the Author
1. Don’t watch the clock; do what it does. Keep going
2. Stay dedicated. It won't happen overnight.
3. Be stronger than your excuses
4. Don’t tell people about your dreams! Show them!!!

JOIN ME AT TELEGRAM – UPSC PRELIMS WITH NEELESH (AIR 442 UPSC CSE 2021) https://t.me/UPSCPrelimsWithNeelesh
FREE CSAT COURSE ON YOUTUBE – CIVIL SERVICES WITH NEELESH, Best PYQ Document for Prelims – www.neeleshair442.com,
Best test series on – WWW.CSETOPPER.COM ©Copyright Reserved with the Author

You might also like